CRT 3rd Year New
CRT 3rd Year New
CRT 3rd Year New
INDEX
1. Profit & loss (advance)……….…..……………………………………………………………………………………………………..02 - 05
2. Mensuration…….…………..……………………………………………………………………………………………………………….06 - 07
5. Data Interpretation….………………………..………………………………………………………………………………………….17 - 24
6. Data Sufficiency…………………………..………….…………………………………………………………………………………….25 - 31
7. Seating arrangement…………………………………………………………………………………………………………………….32 - 37
8. Syllogism……………………………….………………………………………………………………………………………………………38 - 50
15. Adjective……….………………………………………………………………………………………………………………………………76
16. Adverb………….……………………………………………………………………………………………………………………………….77
PAGE- 1
PROFIT & LOSS/DISCOUNT
1. A shopkeeper bought 84, Identical shirts priced at Rs. 240 each. He spent a total of Rs. 3200 on transportation and
packaging. He put the label of marked price of Rs. 420 on each shirt. He offered a discount of 15% on each shirt at
the marked price. What is the total profit of the shopkeeper in the whole transaction?
(a)Rs. 6258 (b)Rs. 6528 (c)Rs. 6268 (d)Rs. 6628 (e) None of these
2. The profit earned after selling an article for Rs. 1,516 are the same as loss incurred after selling the article for Rs.
1,112. What is the cost price of the article ?
(a)Rs. 1,314 (b)Rs. 1,343 (c)Rs. 1,414 (d)Rs. 1,434 (e) None of these
3. An article is sold at a loss of 10%. Its cost price is Rs 800. A discount of 20% was offered on the labeled price while
selling. What is the loss per cent at the labeled price?
(a)10% (b)15% (c)20% (d)0.25% (e)None of these
4. Vinod makes a profit of Rs. 110 if he sells a certain number of pencils he has at the price of Rs. 2.5 per pencil and
incurs a loss of Rs. 55, if he sells the same number of pencils for Rs. 1.75 per pencil. How many pencils does Vinod
have ?
(a)220 (b)240 (c)200 (d)Cannot be determined (e)None of these
5. A shopkeeper bought 30 kg. of wheat at the rate of Rs. 45 per kg. He sold forty percent of the total quantity at the
rate of Rs. 50 per kg. Approximately, at what price per kg, should he sell the remaining quantity to make 25 per cent
overall profit ?
(a)Rs. 54 (b)Rs. 52 (c)Rs. 50 (d)Rs. 60 (e)Rs. 56
6. A shopkeeper sells notebooks at the rate of Rs 45 each and earns a commission of 4%. He also sells pencil box at the
rate of Rs 80 each and earns a commission of 20%. How much amount of commission will he earn in two weeks if he
sells 10 notebooks, and 6 pencil boxes a day ?
(a)Rs. 1,956 (b)Rs. 1,586 (c)Rs. 1,496 (d)Rs. 1,596 (e) None of these
7. Meera purchased an item for Rs. 62,000 and sold it at loss of 25 percent. With that amount she purchased another
item and sold it at a gain of 30 percent. What was her overall gain/loss?
(a)Loss of Rs. 1560 (b)Profit of Rs. 1560 (c)Loss of Rs. 1550 (d)Profit of Rs. 1550 (e)None of these
8. Seema purchased an item for Rs. 9,600 and sold it for a loss of 5 percent. From that money she purchased another
item and sold it for a gain of 5 percent, What is her overall gain/loss?
(a)Loss of Rs. 36 (b)Profit of Rs. 24 (c)Loss of Rs. 54 (d)Profit of Rs. 36 (e)None of these
9. A shopkeeper sells two watches for Rs. 308 each. On one he gets 12% profit and on the other 12% loss. His profit or
loss in the entire transaction was ?
(a) 36/25% loss (b)36/25% gain (c)77/25 % loss (d) 77/25 %gain (e)None of these
10. A merchant buys two items for Rs. 7500. One item he sells at a profit of 16% and the other item at 14% loss. In the
deal the merchant makes neither any profit nor any loss. What is the difference between selling price of both the
items? (in rupees)
(a)625 (b)610 (c)620 (d)630 (e)615
11. An Item was bought at Rs. X and sold at Rs. Y, there by earning a profit of 20%. Had the value of X been 15% less and
the value of Y been Rs. 76 less, a profit of 30 would have been earned. What was the value of X?
(a)Rs. 640 (b) Rs.400 (c) Rs.600 (d) Rs.800 (e) Rs.840
12. A' bought a certain quantity of oranges at total cost of Rs. 1200. He sold 1/3rd of those oranges at 20% loss. If A
earns an overall profit of 10%, at what percent profit did A sell the rest of the oranges?
PAGE- 2
(a)16% (b)15% (c)22% (d)25% (e)20%
13. A’ sold an article for Rs. 8000 and incurred a loss. Had he sold the article for Rs. 9800, his gain would have been
twice the amount of loss. At what price should the article be sold to earn 20% profit?
(a)Rs. 10,840 (b)Rs. 9,820 (c)Rs. 10,320 (d)Rs. 9,840 (e)Rs. 10,480)
14. A trader has 400 kg of rice; He sells a part of it at a profit of 36% and remaining part at a loss of 24%. He overall loses
12% in the whole transaction. Find the quantity of rice sold at 24% loss.
(a) 320 kg (b) 330 kg (c) 300 kg (d) 350 kg (e) None of these
15. A trader has 600 kgs of rice, a part of which he sells at 15% profit and the remaining quantity at 20% loss. On the
whole, he incurs an overall loss of 6%. What is the quantity of rice he sold at 20% loss?
(a) 250 kgs (b) 320 kgs (c) 420 kgs (d) 360 kgs (e) 480 kgs
16. A shopkeeper bough a table marked at Rs. 200 at successive discounts of 10% and 15% respectively. He spent Rs. 7
on transport and sold the table for Rs. 208. What will be his profit percentage?
(a)35 (b)40 (c)30 (d)45 (e)32
17. A trader marks up his goods by 50%. However, he could sell only-third of his stock at this price. The half of the
remaining stock was sold at a discount of 7.14% and the remaining at a discount of 16.67%. Find the overall
percentage profit of the trader.
(a)38% (b)42% (c)29% (d)34% (e)40%
18. Two mobile phones were purchased at the same price. One was sold at a profit of 30% and the second was sold at a
price which was Rs. 2500 less than the price at which the first was sold. If the overall profit earned by selling both
the mobile phones was 5%, what was the cost price of one mobile phone?
(a)Rs. 8000 (b)Rs. 5000 (c)Rs. 6000 (d)Rs. 4500 (e)Rs. 5500
19. Mr. X, a businessman had the income in the year 1995, such that he earned a profit of 20% on his investment in the
business. In the year 1996 his investment was less by Rs. 5,000 but still had the same income (Income = Investment
+ Profit) as that in 1995. Thus the per cent profit earned in 1996 increased by 6%. What was his investment in 1995
? (a)Rs. 1,02,000 (b)Rs. 150,500 (c)Rs. 1,05,000 (d)Data inadequate (e)None of these
20. If on selling 12 notebooks any seller makes a profit equal to the selling price of 4 notebooks, what is his per cent
profit?
(a)50% (b)25% (c)50/3 % (d)Data inadequate (e)None of these
21. The value of machine depreciates at the rate of 12 per cent per annum. It was purchased three years ago. Its
present value is Rs 29644.032, what was the purchase price of the machine?
(a)Rs 48700 (b)Rs 43500 (c)Rs 38900 (d)Rs 39000 (e) None of these
22. Profit earned by an organization is distributed among officers and clerks in the ratio of 5 : 3 respectively. If the
number of officers is 45 and the number of clerks is 80 and the amount received by each officer is Rs 25,000, what
was the total amount of profit earned ?
(a)Rs 22 lakhs (b)Rs 18.25 lakhs (c)Rs 18 lakhs (d)Rs 23.25 lakhs (e)None of these
23. A sells an item at 20% profit to B. B sells it to C at 10% profit. C sells it to D at Rs. 116 profit. Difference between the
cost price of D and Cost price of A was Rs. 500. How much did B pay to A for the item ?
(a)Rs. 1,240 (b)Rs. 1,250 (c)Rs. 1,440 (d)Rs. 1,450 (e)Rs. 1,400
24. Ram has two articles A and B. The total cost of both articles is Rs. 8000. Ram sells article A at the profit of 20% and
article B at a loss of 12%. There is neither profit nor loss in the whole transaction. At what price should the article B
be sold to gain 25%?
PAGE- 3
(a)Rs. 6250 (b)Rs. 6350 (c)Rs. 3750 (d)Rs. 3850 (e) None of these
25. If the cost price of 24 articles is equal to the selling price of 21 articles, what is the percentage gain or loss?
(a)100/7 % gain (b)100/7 % loss (c)12.5% loss (d)12.5% gain (e) None of these
26. Of the two varieties of rice available, variety A is bought at Rs. 32 per kg. and variety B at Rs. 80 per kg. Two varieties
of rice are mixed together in the respective ratio of 8:5 and the mixture is sold at Rs. 72 per kg. What per cent of
profit approximately the Seller receives?
(a)43% (b)46% (c)38% (d)39% (e) None of these
27. The income of a broker remains unchanged though the rate of commission is increased from 4 per cent to 5 per
cent. The percentage of slump in business is:
(a)10 percent (b)15 percent (c)20 percent (d)30 percent (e) None of these
28. A watch is sold at a profit of 20%. If both the cost price and the selling price of the watch are decreased by Rs. 100,
the profit would be 5% more. Original cost price of the watch is
(a)Rs. 450 (b)Rs. 500 (c)Rs. 550 (d)Rs. 600 (e) None of these
29. Cost of 4 fans and 3 blowers is Rs. 16,500. Also cost of 2 fans, 2 tables and 2 blowers is Rs. 12.000. Cost of one table
is Rs. 1000.What is the cost of 3 fans and one blower?
(a)Rs. 8,000 (b)Rs. 7,500 (c)Rs. 8,500 (d)Cannot be determined (e)None of these
30. By selling 18 chocolates, a vendor loses the selling price of 2 chocolates. Find his loss percent.
(a)9% (b)10% (c)11% (d)12% (e) None of these
31. Deepa bought a calculator with 30% discount on the listed price. Had she not got the discount, she would have paid
Rs. 82.50 extra. At what price did she buy the calculator ?
(a)Rs. 192.50 (b)Rs. 275 (c) Rs. 117.85. (d)Cannot be determined (e)None of these
32. Naresh purchased a TV set for Rs. 11,250 after getting discount of 10% on the labeled price. He spent Rs. 150 on
transport and Rs. 800 on Installation. At what price should it be sold so that the profit earned would have been 15%
if no discount was offered?
(a)Rs. 12937.50 (b)Rs. 14,030 (c)Rs. 13,450 (d)Rs. 15467.50 (e) None of these
33. Manoj sold an article for Rs. 15,000. Had he offered a discount of 10% on the selling price he would have earned a
profit of 8 %.what is the cost price
(a) Rs. 12,500 (b) Rs. 13,500 (c)Rs. 12,250 (d) Rs. 13,250 (e) None of these
34. A discount of 15% is given on the marked price of an article. The shopkeeper charges sates tax of 6% On the
discounted price. If the selling price be Rs. 1081.20, what is the marked price of the article?.
(a) Rs. 1185.20 (b) Rs. 1250.20 (c) Rs. 302 (d) Rs. 1200 (e) None of these
35. Giridhar purchased 100 sarees Rs. 450 per piece. While selling he offered 10% discount on the labelled price and
earned a profit of 20%. What was the labelled price of each saree? .
(a)Rs. 540 (b)Rs. 650 (c)Rs. 590 (d)Rs. 600 (e) None of these
36. An article Was purchased for Rs. Rs.8.350. Its price was marked up by 30%. It was sold at, a discount of 20% on the
marked up price. What was the profit percent on the cost price?
(a)4% (b)7% (c)5% (d)3% (e)6%
37. A dealer allowed a discount of 25% on the marked price of Rs. 12000 on an article and incurred a loss of 10%. What
discount should he allow on the marked price so that he gains Rs 440 On the article?
(a)11% (b)13% (c)19% (d)15% (e) None of these
PAGE- 4
38. A shopkeeper sells 25 articles at 45/- per article after giving 10% discount and gains 50% profit. If the discount is not
given, what is the profit gained?
(a)60% (b)60.67% (c)66.67% (d)66% (e) None of these
39. While selling a watch, a shopkeeper gives a discount of, 15%. If he gives a discount of 20%. he earns Rs. 51 less as
profit. What is the original price of the watch?
(a) Rs. 920 (b) Rs. 985 (c)Rs. 1125 (d)Rs. 1020 (e) None of these
40. A shopkeeper sold a T.V. set for 17,940/-, with a discount of 8% and gained 19.6% If no discount is allowed, what
will be his gain per cent?
(a)25% (b)26.40% (c)24.80% (d)Cannot be determined (e)None of these
41. Ravi ate in a restaurant and got a membership discount of 30% on the original bill amount but he had to pay 10% as
service tax and 8% service charge on the discounted bill amount. If Ravi paid Rs. 4,743, which included a tip of Rs.
200, how much money did he give as service charge?
(a) Rs. 324 (b) Rs. 314 (c) Rs. 296 (d) Rs. 308 (e) Rs. 272
42. On a Rs. 10,000 payment order, a person has choice between 3 successive discounts of 10%, 10% and 30%, and 3
successive discounts of 40%, 5% and 5%. By choosing the better one he can save (in Rupees) :
(a)200 (b)255 (c)400 (d)433 (e) None of these
43. A shopkeeper marks his goods at such a price that after allowing a discount of 12.5% on the marked price, he still
earns a profit of 10%. The marked price of an article which costs him Rs 4,900 is
(a) Rs. 5,390 (b) Rs. 6,160 (c)Rs. 5,490 (d)Rs. 6,260 (e) None of these
44. A dealer offers a discount of 10% on the marked price of an article and still makes a profit of 20%. If its marked price
is Rs. 800, then the cost price is:
(a) Rs. 600 (b)Rs. 700 (c) Rs. 800 (d) Rs. 900 (e) None of these
45. What is the maximum percentage discount that a merchant can offer on her marked price so that she ends up
selling at no profit or loss, if she had initially marked her goods up by 50% ?
(a)16.67% (b)20% (c)50% (d)33.33% (e) None of these
PAGE- 5
MENSURATION
1. The area of a rectangular field is 3375 m2 and its length and breadth are in the ratio 5:3. Find the breadth ?
(a) 125m (b) 45m (c) 225m (d) 65m (e) None
2. The perimeter of a square is twice the perimeter of a rectangle. If the perimeter of a square is 48cms and the length
of the rectangle is 7cm. Find the breadth of the rectangle?
(a) 5cm (b) 4cm (c) 9cm (d) 6cm (e) None
3. The area of a square is 7sqm less than half of the area of a rectangle. The length of the rectangle is 14m and breadth
is 6m less than its length. What is the perimeter of the square ?
(a) 42cm (b) 32cm (c) 22cm (d) 28cm (e) None
4. The length of a rectangular floor is twice its breadth. If Rs.624 is required to paint the floor at the rate of Rs.8 psm.
What is the breadth of the floor ?
(a) 6.24m (b) 5.10m (c) 7.8m (d) 4.6m (e) None
5. 2 cubes have their volume in the ratio 1:8, find the ratio of their surface area ?
(a) 2 : 5 (b) 4 : 1 (c) 1 : 4 (d) 3 : 7 (e)None
6. A square field has an area of 7396 sq m, find the cost of fencing around it at Rs.22 per meter ?
(a) 6754 (b) 7568 (c) 8765 (d) 7943 (e)None
7. How many marbles of 10cm length and 7cm width are required to pave the floor of room 7m length and 4m breadth ?
(a) 4000 (b) 5100 (c) 2800 (d) 3200 (e) None
10. The length of a rope by which a cow must be tethered in order that she may be able to graze an area of 2826 sq.m is
(a) 20m (b) 17m (c) 27m (d) 30m (e) None
11. The number of rounds that a wheel of diameter14m will make in going 2 km is
(a) 62 rounds (b) 50 rounds (c) 45 rounds (d) 56 rounds (e) None
12. The ratio of length to breadth of a rectangle is 7:6. The perimeter of the plot is 260m. Find the area of the plot ?
(a) 4800sq.m (b) 5600sq.m (c) 4200sq.m (d) 2600sq.m (e) None
13. In a parallelogram the length of adjacent sides are 13m and 11m. If the length of one diagonal is 18m, find the length
of the other diagonal?
(a) 9m (b) 16m (c) 12m (d) 19m (e) None
14. The circumference of the 2 circle is 212cm and 412cm. What is the difference between the area of the 2 circles?
(a) 10048cm2 (b) 10480cm2 (c) 10070cm2 (d) 10140cm2 (e) None
15. If the length of the rectangular field is increased by 10% and the breadth is reduced by 10%, now the area of
rectangle will be 217 cm2. Find the area of the original rectangle?
(a) 210 cm2 (b) 219 cm2 (c) 215 cm2 (d) 221 cm2 (e) None
16. The area of right angle triangle is 100sq cm. The ratio of base to its height is 4:5.Find the length of the hypotenuse?
PAGE- 6
(a) 20cm (b) 22cm (c) 28cm (d) 30cm (e) None
17. The diameter of the driving wheel of a bus is 280 cm. How many revolutions per minute must the Wheel make in
order to keep a speed of 66 kmph?
(a)150 (b) 180 (c) 145 (d) 125 (e) None
18. Find the ratio of the areas of the in circle and circum circle of a square.
(a) 1 : 2 (b) 3 : 2 (c) 2 : 1 (d) 4 : 5 (e) None
19. A cord is in the form of a square enclosing an area of 44 cm^2.If the same cord is bent into a circle, then find the
area of that circle.
(a) 40cmsq (b) 78cmsq (c) 68cmsq (d) 52cmsq (e) 56cmsq
20. The area of a rhombus is 144 cm square. One of its diagonals is twice the other. The length of the shorter diagonal
is?
(a) 12 cm (b) 11 cm (c) 10 cm (d) 14 cm (e) None
21. The base of a parallelogram is twice its height and its area it 128 cm square. Find the difference between its height
and the base.
(a) 6 cm (b) 7 cm (c) 8 cm (d) 9 cm (e) 10 cm
22. Find the distance between the two parallel sides of a trapezium if the area of the trapezium is 250 sq. m. and the
two parallel sides are equal to 15 m and 10 m respectively.
(a) 25 m (b) 20 m (c) 40 m (d) 30 m (e) None
23. In measuring the sides of a rectangle, one side is taken 5% in excess and other 4% in deficit. Find the error per cent
in area calculated from the measurement.
(a) - 0.5% (b) 0.8% (c) 1% (d) -1% (e) None
24. A room is 5 meters long and 3 meters broad; the doors and windows occupy 10 sq. meters, and the cost of colouring
the remaining part of the surface of the walls with paper 25 cm wide, at Rs. 5 per piece of 15 m is Rs. 40. Find the height
of the room.
(a) 2.5 m (b) 2 m (c) 1.5 m (d) 3 m (e) 3.25 m
25. The length of a room floor exceeds its breadth by 20 m. The area of the floor remains unaltered when the length is
decreased by 10 m but the breadth is increased by 5 m. The area of the floor (in square meters) is:
(a) 280 (b) 325 (c) 300 (d) 420 (e) None
26. The in-radius of a triangle is 6 cm, and the sum of the lengths of its sides is 50 cm. The area of the triangle (in sq.
cm.) is
(a) 50 (b) 56 (c) 150 (d) 300 (e) None
27. A piece of wire when bent to form a circle will have a radius of 84 cm. If the wire is bent to form a square, the length
of a side of the square is
(a) 132 cm (b) 225 cm (c) 152 cm (d) 168 cm (e) None
28. The length of a rectangle is reduced by 30%. By what percent would the width have to be increased to maintain the
original area?
(a) 25% (b) 42.86% (c) 32.76% (d) 35.50% (e) None
29. A circular wire of radius 49 cm is cut and bent in the form of a rectangle whose sides are in the ratio of 4:7. The
smaller side of the rectangle is
(a) 56cm (b) 48cm (c) 35cm (d) 42cm (e) None
PAGE- 7
TIME AND DISTANCE
Different cases
By using the basic relationship between time, speed, and distance, the following three different cases are possible:
Case I When S (distance) is constant V α So, V1 /V2 = T2 /T1 (Direct)
Case II When T (time) is constant S α V So, S1 /S2 = V1 /V2
Case III When S (speed) is constant T α D So, T1 /T2 = D1 /D2
TYPES OF QUESTIONS
TYPE – 1: An areoplane covers a certain distance at a speed of 240 km/hour in 5 hours. To cover the same distance in
5/3 hours it must travel at a speed of? Ans: (720 km/hr)
TYPE – 2: If a man walks 20 km at 5 km/hr. he will be late by 40 minutes. If he walks at 8 km/hr, how early from the
fixed time will he reach? Ans: (50 Mins)
TYPE – 3: A train passes a 50 meters long platform in 14 seconds and a man standing on the platform in 10 seconds.
The speed of the train is? Ans: (45 km/hr)
TYPE – 4: A man can reach a certain place in 30 hours. If he reduces his speed by 1/15th, he goes 10 km less in that
time. Find his speed per hour? Ans: (5 km/hr)
TYPE – 5: Two trains, A and B, start from stations X and Y towards each other, they take 4 hours 48 minutes and 3 hours
20 minutes to reach Y and X respectively after they meet if train A is moving at 45 km/hr. , then the speed of the train B
is? Ans: (24 km/hr)
TYPE – 6: A train 270 meters long is running at a speed of 36 km/hr. then it will cross a bridge of length 180 meters in?
Ans: (45 second)
TYPE – 7: A is twice as fast as B and B is thrice as fast as C is. The journey covered by C in 1.5 Hours will be covered by A
in? Ans: (15 min)
TYPE – 8: A thief is noticed by a policeman from a distance of 200m the thief starts running and the policeman chases
him. The thief and the policeman run at the rate of 10km hr. and 11 km./hr. respectively. What is the difference
between them after 6 minutes? Ans: (100 mtr)
TYPE – 9: Two trains 140 m and 160 m long run at the speed of 60 km/hr, and 40 km/hr. respectively in opposite
directions on parallel tracks. The time (in seconds) which they take to cross, each other is? Ans: (10.88 sec)
TYPE – 10: Points 'A' and 'B' are 70 km apart on a highway and two cars start at the same time; If they travel in the
same direction, they meet in 7 hours, but if they travel towards MOCKTIME.COM ONLINE TEST SERIES
CORRESPONDENCE COURSE each other they meet in one hour. Find the speed of the two cars (in km/hr)? Ans: (40
km/hr and 30 km/hr)
TYPE – 11: A and B travel the same distance at speed of 9km/hr and 10 km/hr respectively. If A takes 36 minutes more
than B, the distance travelled by each is? Ans: (54 km)
TYPE – 12: Walking 6/7th of his usual speed a man is 12 minutes late. The usual time taken by him cover that distance
in? Ans: (1 hour 12 minutes)
TYPE – 13: Mr. X goes to his office by scooter at a speed of 30 km/hr and reaches 6 minutes earlier. If he goes at a
speed of 24 km/hr, he reaches 5 minutes late. The distance of his office is? Ans: (22 km)
PAGE- 8
1. A motor starts with the speed of 70 kmph with its speed increasing every two hours by 10 kmph. In how many hours
will it cover 345 kms?
(a) 2.25hours (b) 4.50hours (c)4 hours 5 minutes (d)Cannot be determined (e)None of these
2. A train running at the speed of 20 metres/second crosses a pole in 24 seconds less than the time it requires to cross
a platform thrice its length at the same speed. What is the length of the train?
(a) 270 metres (b)340 metres (c)180 metres (d)Cannot be determined (e)None of these
3. A man takes 6 hours 35 minutes in walking to a certain place and riding back. He would have taken 2 hours less by
riding both ways. What would be the time he would take to walk both ways?
(a) 4 hours 35 minutes
(b) 8 hours 35 minutes
(c) 10 hours
(d) 8 hours 25 minutes
(e) None of these
4. A train travelling at the speed of 60 kmph crosses a platform in 20 seconds. What is the length of the train?
(a)333 metres (b)300 metres (c)336 metres (d)Cannot be determined (e)None of these
5. A boat running downstream covers a distance of 30 kms in 2 hours. While coming back the boat takes 6 hours to
cover the same distance. If the speed of the current is half that of the boat, what is the speed of that boat in kmph?
(a)15 (b)5 (c)10 (d)Cannot be determined (e)None of these
6. Samir drove at the speed of 45 kmph. from home to a resort. Returning over the same route he got stuck in traffic
and took an hour longer, also he could drive only at the speed of 40 kmph. How many kilometres did he drive each
way?
(a) 250 kms (b) 300 kms (c) 310 kms. (d) 275 kms. (e) None of these
7. A boat running at the speed of 34 kmph downstream covers a distance of 4.8 kms. in 8 minutes. The same boat
while running upstream at same speed covers the same distance in 9 minutes. What is the speed of the current?
(a) 2.4 kmph (b) 3 kmph (c) 2 kmph (d) 3.2 kmph (e)' None of these
8. A bus started its journey from Ramgarh and reached Devgarh in 44 minutes with its average speed of 50 km/hour. If
the average speed of the bus is increased by 5 km/hour, how much time will it take to cover, the same distance?
(a) 40 minutes (b) 38 minutes (c) 36 minutes (d) 31 minutes (e) 49 minutes
9. A man walked at a speed of 4 km/hr from point A to B and came back from point B to A at the speed of 6 km/hr.
What would be the ratio between the time taken by man in walking from point A to B to point B to A respectively?
(a) 5 : 3 (b) 2 : 3 (c)2:01 (d)4:03 (e) 3 : 2
10. A train speeds past a pole in 20 seconds and speeds past a platform 100 metres in length in 30 seconds. What is the
length of the train?
(a) 100 metre (b) 150 metre (c) 180 metre (d) 200 metre (e) None of these
11. A 320 metre long train takes 80 seconds more to cross a platform twice its length than it takes to cross a pole at the
same speed. What is the speed of the train in metre/second?
(a)16 (b)10 (c)6 (d)Cannot be determined (e)None of these
12. The respective ratio between the speed of a car, a jeep and tractor is 3 : 5 : 2. The speed of the jeep is 250 percent
of the speed of the tractor which covers 360 km in 12 hours. What is the average speed of car and jeep together?
(a) 60 km/hr (b) 75 km/hr (c)40 km/hr (d)Cannot be determined (e)None of these
13. Amit, Sucheta and Neeti start running around a circular track and complete one round in 18 24 and 32 seconds
PAGE- 9
respectively. In how many seconds will the three meet again at the starting point if they all have started running at
the same time?
(a) 196 sec (b) 288 sec (c)324 sec (d)Cannot be determined (e)None of these
14. A person travels from P to Q at a speed of 40 kmph and returns to Q by increasing his speed by 50%. What is his
average speed for both the trips?
(a) 36 kmph (b) 45 kmph (c) 48 kmph (d) 50 kmph (e) None of these
15. The speed of a boat in still water is 6 kmph and that of current is 3 kmph. The boat starts from point A and rows to
point B and comes back to point A. It takes 12 hours during this Journey. How far is point A from point B?
(a) 27 km (b) 25 km (c) 20 km (d) 30 km (e) None of these
16. The speed of a boat in still water is 9.5 kmph while that of current is 2.5 kmph. If the boat takes 114 minutes in
rowing from point A to B and coming back to point A, what is the distance between A and B?
(a) 8.4 km. (b) 4.8 km. (c) 8.8 km. (d) 7.4 km. (e) None of these
17. A car starts at 11 am from point A towards point B at 36 kmph while another car starts at 1 pm from point B towards
A at 44 kmph. They cover a distance of 592 km till meeting. At what time will they meet each other?
(a) 8 pm (b) 6 : 30 pm (c) 7 : 30 pm (d) 5 : 30 pm (e) None of these
18. A man takes 16/9 times as long to row a distance upstream as to row the same distance downstream. What is the
speed of the boat in still water if it takes 3 hours to travel 38.4 km downstream? (in km/h)
(a)11 (b)10 (c)12.4 (d)12 (e) None of these
19. Prem and Shyam decide to go on a trip to point Y on a particular day from point X. Prem leaves for point Y at 11 : 00
am, at speed of 72 km/hour. Shyam leaves for point Y at 11: 30 same day as Prem left. At what speed should Shyam
travel to catch up with Prem in 4 hours? (in km/hour)
(a)8 5 (b)81 (c)80 (d)82 (e)86
20. A person has to travel from point B in certain time. Travelling at a speed of 5 kmph he reaches 48 minutes late and
while travelling at a speed of 8 kmph he reaches 15 minutes early. What is the distance from point A to point B ?
(a) 15 kms (b) 9 kms (c) 12 kms (d) 18 kms (e) 14 kms
21. Ram and Shyam are travelling from point A to B, which are 60km apart. Travelling at a certain speed Ram takes one
hour more than Shyam to reach point B. If Ram doubles his speed he will take 30 minutes less than Shyam to reach
point B. At what speed was Ram driving from point A to B?
(a) 15 kmph (b) 35 kmph (c) 30 kmph (d) 25 kmph (e) 20 kmph
22. On walking at 3/4 of his usual speed a man reaches his office 20 minutes late. What is the usual time taken by him in
reaching his office?
(a) 75 minutes (b) 60 minutes (c) 40 minutes (d) 30 minutes (e) None of these
23. A man starts going for morning walk every day. The distance walked by him on the first day was 2 kms.
Everyday he walks half of the distance walked on the previous day. What can be the maximum total distance walked
by him in his life time?
(a) 4 kms. (b) 120 kms. (c) 18 kms. (d)Data inadequate (e) None of these
24. The distance between two points is 36 km. A boat rows in still water at 6 kmph. It takes 8 hours less to cover this
distance in downstream in comparison to that in upstream. The rate of stream is
(a) 3 kmph (b) 2 kmph (c) 2.5kmph (d) 4kmph (e) None of these
25. Rohan covered 2/3rd of a certain distance in 2 hours 30 minutes at the rate of x kmph. He covers the remaining
distance at the rate of (x+ 2) kmph in 50 minutes. What is the total distance?
PAGE- 10
(a)21 km (b) 18 km (c)16 km (d) 15 km (e) 20 km
26. Two trains A and B of equal length of 200 metres running in opposite direction cross each other in16 seconds. What
is the speed of train A?
(a) 90 km/hr. (b) 40 km/hr. (c)80 km/hr. (d)Data inadequate (e) None of these
27. An employee may. claim Rs 7.00 for each km when he travels by taxi and Rs 6.00 for each km if he drives his own
car. If in one week he claimed Rs 675 for travelling 90 km, how many kms did he travel, by taxi?
(a) 135 (b)155 (c)162 (d)170 (e)None of these
28. Train-A crosses a pole in 33 seconds and another Train-B crosses a pole in 55 seconds. Length of Train A is three-
fourth of Train-B. What is the respective ratio between the speed of Train-A and Train-B?
(a) 5:11 (b)5:04 (c)11:03 (d)Cannot be determined (e) None of these
29. The cost of fuel running the engine of an army tank is proportional to the square of the speed and Rs. 64 per hour
for a speed of 16 kmph. Other costs amount to Rs. 400 per hour. The tank has to make a journey of 400 km at a
constant speed. - The most economical speed for this journey is?
(a) 20 kmph (b) 30 kmph (c) 35 kmph (d) 40 kmph (e) None of these
30. A can walk a certain distance in 20 days when he rests 8 hours a day. How long will he take to walk twice the
distance, twice as fast and rests twice as long each day?
(a) 40 days (b) 60 days (c) 80 days (d) 100 days (e) None of these
31. An express train travelled at an average speed of 100 km/hr, stopping for 3 minutes after 75 km, A local train
travelled at a speed of 50 km/hr, stopping for 1 minute after every 25 km. If the trains began travelling at the same
time, how many kms did the local train travel in the time it took the express train to travel 600 km?
(a) 307.5 km (b) 900 km (c) 1000 km (d) 1200 km (e)None of these
32. A plane left 30 minutes later than the scheduled time and in order to reach the destination 1500 km away in time, it
had to increase the speed by 250 km/hr from the usual speed. Its usual speed is?
(a) 720km/hr (b) 730 km/hr (c) 740 km/hr (d) 750km/hr (e) None of these
33. The speed of a boat in still water is 17.5 kmph and that of current is 2.5 kmph. The boat goes from X to Y in
downstream and returns to point Z, The whole journey takes 429 minutes. The distance between Z and Y is 2/5th of
the distance between X and Y, Find the total distance covered by the boat. (Approximated to the nearest integer).
(a) 130 km. (b) 140 km. (c) 160 km. (d) 120 km (e) None of these
34. The ratio between the rates of travelling of A and B is 2:3 and therefore A takes 10 minutes more than the time
taken by B to reach a destination. If A had walked at double the speed, he would have covered the distance in
(a) 30 min (b) 25 min (c) 20 min (d) 15 min (e) None of these
35. Two trains running in opposite directions cross a man standing on the platform in 27 seconds and 17 seconds
respectively and they cross each other in 23 seconds. The ratio of their speed is:
(a) 1 : 3 (b) 3 : 2 (c) 3 : 4 (d) CND (e)None of these
36. The speed of the boat in still water is 16 kmph and speed of the current is 2 kmph. It takes a total of 6.5 hours to
row upstream from point A to point B and downstream from point B to point C. If the distance from point A to point
B is two-third the distance between point B and C, what is the total distance travelled by the boat (both upstream
and downstream)?
(a) 112 km (b) 98 km (c) 124 km (d) 90 km (e) 105 km
PAGE- 11
RACE
'A gives B a start of x meters’: This statement implies that, while A starts the race from starting point, whereas, B starts
10 meters ahead of A. To cover a race of 100 meters in this case, A will have to cover 100 meters while B will have to
cover only (100 - x).
'A beats B by x m': This statement implies that in the same time, while A reached the winning point, whereas, B is
behind A by x m. To cover a race of 100 meters in this case, A has covered 100 meters while B has covered only (100 - x )
A can give B a start of t minutes: This statement implies that A will start t minutes after B starts from the starting point.
Both A and B will reach the finishing point at the same time.
A gives B x meters and t minutes: This statement implies that A and B start from the starting point at the same instant,
but while A reaches the finishing point, B is behind by x meters, and, B takes t minutes compared to A to complete the
race. So, B covers remaining x meters in extra t minutes. This gives the speed of B as x/t.
Dead Heat: A dead heat situation is when all participants reach the finishing point at the same instant of time.
1. In a 100 m race, A can give B 10 m and C 28 m. In the same race B can give C:
(a) 18 m (b) 20 m (c) 27 m (d) 9 m
2. A and B take part in 100 m race. A runs at 5 kmph. A gives B a start of 8 m and still beats him by 8 seconds. The
speed of B is:
(a) 5.15km/h (b) 4.14km/h (c) 4.25 km/h (d) 4.4 km/h
3. In a 500 m race, the ratio of the speeds of two contestants A and B is 3 : 4. A has a start of 140 m. Then, A wins by:
(a) 60 m (b) 40 m (c) 20 m (d) 10 m
5. In a race of 200 m, A can beat B by 31 m and C by 18 m. In a race of 350 m, C will beat B by:
(a) 22.75 m (b) 25 m (c) 19.5 m (d) 7.57 m
6. In 100 m race, A covers the distance in 36 seconds and B in 45 seconds. In this race A beats B by:
(a) 20 m (b) 25 m (c) 22.5 m (d) 9 m
7. In a 200 meters race A beats B by 35 m or 7 seconds. A's time over the course is:
(a) 40 sec (b) 47 sec (c) 33 sec (d) None of these
8. Arun is 5/3 times as fast as B. If A gives B a start of 80 m, how far must the winning post be so that A and B might
reach it at the same time?
(a) 200 m (b) 300 m (c) 270 m (d) 160 m
9. In a 100 m race, A can beat B by 25 m and B can beat C by 4 m. In the same race, A can beat C by:
(a) 21 m (b) 26 m (c) 28 m (d) 29 m
10. In a 50 m race, A can give a start of 5 m to B and a start of 14 m to C. In the same race, how much start can B give to
C?
(a) 19 m (b) 20 m (c) 10 m (d) 15 m
PAGE- 12
11. In a 500 m race, A reaches the final point in 28 s and B reaches in 35 s. By how much distance does A beat B?
(a) 90 m (b) 100 m (c) 120 m (d) None of these
12. A runs 4 times as fast as B. If A gives B a start of 60 m, how far must the goal on the race course be so that A and B
reach it at the same time?
(a) 80 m (b) 100 m (c) 90 m (d) None of these
13. In a 10 km race. A, B, and C, each running at uniform speed, get the gold, silver, and bronze medals, respectively. If A
beats B by 1 km and B beats C by 1 km, then by how many meters does A beat C?
(a) 1000 m (b) 1100 m (c) 1900 m (d) None of these
14. In a circular race of 2400m, A and B start from the same point and at the same time with speeds of 27km/hr. and 45
km/hr. Find when will they meet again for the first time on the track when they are running in the same direction
and opposite direction?
(a) 60 sec (b) 50 sec (c) 110 sec (d) 120 sec
15. If Geeta can give a start of 100 m of distance or 20 s of time to her friend Meena in a race of 1000 meters. How
much time Geeta will take to cover the 1000 meters?
(a) 200 sec (b) 160 sec (c) 180 sec (d) 140 sec
16. Sita and Radha participate in a race of 1000 m. If Sita starts, when Radha has covered 100 metres, then Sita finishes
the race 20 seconds before Radha, and if Sita starts, when Radha has covered 125 metres then both finish the race
at the same time. How long does Sita take to run the complete race?
(a) 350 m (b) 700 m (c) 780 m (d) 900 m
17. If Mohan can give a start of 50 m of distance or 10 s of time to Rohan in a race of 1000 meters. How much time
Mohan will take to cover the 1000 meters?
(a) 190 sec (b) 200 sec (c) 240 sec (d) 180 sec
18. Ram and Sita participate in a race of 1000 meters. If Ram gives Sita a start of 200 meters or 20 seconds. In ow much
time Ram completes the race?
(a) 70 sec (b) 120 sec (c) 100 sec (d) 80 sec
19. In a 200m race, if A gives B a start of 25 metres, then A wins the race by 10 seconds. Alternatively, if A gives B a start
of 45 metres the race ends in a dead heat. How long does A take to run 200m?
(a) 100 sec (b) 112.5 sec (c) 77.5 sec (d) 87.5 sec
20. A gives B a start of 10 metres in a 100 metre race and still beats him by 1.25 seconds. How long does B take to
complete the 100 metre race if A runs at the rate of 10 m/sec?
(a) 8 sec (b) 10 sec (c) 16.67 sec (d) 12.5 sec
PAGE- 13
SIMPLE INTREST AND COMPOUND INTREST
1. What will be the compound interest on a sum of Rs. 25,000 after 3 years at the rate of 12 p.c.p.a.?
(a) Rs9000.30 (b) Rs 9720 (c) Rs 10123.20 (d) Rs 10483.20 (e) None
2. At what rate of compound interest per annum will a sum of Rs. 1200 become Rs. 1348.32 in 2 years?
(a) 6% (b) 6.5% (c) 7% (d) 7.5% (e) None
3. The least number of complete years in which a sum of money put out at 20% compound interest will be more than
doubled is:
(a) 3 (b) 4 (c) 5 (d) 6 (e) None
4. Albert invested an amount of Rs. 8000 in a fixed deposit scheme for 2 years at compound interest rate 5 p.c.p.a. How
much amount will Albert get on maturity of the fixed deposit?
(a) Rs. 8600 (b) Rs. 8620 (c) Rs. 8820 (d) Rs 8920 (e) None
5. The effective annual rate of interest corresponding to a nominal rate of 6% per annum payable half-yearly is:
(a) 6.06% (b) 6.07% (c) 6.08% (d) 6.09% (e) None
6. The difference between simple and compound interests compounded annually on a certain sum of money for 2 years
at 4% per annum is Re. 1. The sum (in Rs.) is:
(a) 625 (b) 630 (c) 640 (d) 650 (e) None
7. There is 60% increase in an amount in 6 years at simple interest. What will be the compound interest of Rs. 12,000
after 3 years at the same rate?
(a) Rs. 2160 (b) Rs. 3120 (c) Rs. 3972 (d) Rs. 6240 (e) None
8. What is the difference between the compound interests on Rs. 5000 for 1 year at 4% per annum compounded yearly
and half-yearly?
(a) Rs. 2.04 (b) Rs. 3.06 (c) Rs. 4.80 (d) Rs. 8.30 (e) None
9. The compound interest on Rs. 30,000 at 7% per annum is Rs. 4347. The period (in years) is:
(a) 2 (b) 2.5 (c) 3 (d) 4 (e) None
10. The Simple interest on a certain sum for 2 years at 10% per annum is Rs. 90. The corresponding compound interest
is:
(a) 97 (b) 90 (c) 94.50 (d) 100 (e) None 11
11. Manoj borrowed a certain sum from Anuj at a certain rate of simple interest for 2 years. He lent this sum to Rakesh
at the same rate of interest compounded annually for the same period. At the end of two years, he received Rs. 4200 as
compound interest but paid Rs. 4000 only as simple interest. Find the rate of interest?
(a) 10% (b) 20% (c) 25% (d) 30% (e) None
12. If the difference between the simple interest and compound interests on some principal amount at 20% for 3 years
is Rs. 48, then the principal amount is
(a) 63 (b) 650 (c) 375 (d) 400 (e) None
13. What sum invested for 2 years at 12% compounded annually will grow to Rs. 4390.40…..
(a) 4000 (b) 3875 (c) 3800 (d) 3500 (e) None
14. A sum of money is borrowed and paid back in two annual installments of Rs. 882 each allowing 5% compound
interest. The sum borrowed was:
(a) 1620 (b) 1640 (c) 1680 (d) 1700 (e) None
PAGE- 14
15. Divide Rs. 3903 between A and B, so that A’s Share at the end of 7 years may equal to B’s share at the end of 9 years,
compound interest being at 4 percent.
(a) 2018 and 1885 (b) 2028 and 1875 (c) 2008 and 1895 (d) 2038 and 1865 (e) None
16. If a sum on compound interest becomes three times in 4 years, then with the same interest rate, the sum will
become 27 times in:
(a) 8 y (b) 12 y (c) 24 y (d) 36 y (e) None
17. A certain sum of money amounts to rupees 2900 at 4% per annum in 4 years. In how many years will it amount to
rupees 5000 at the same rate?
(a) 20 (b) 22 (c) 24 (d) 25 (e) None
18. A sum of money becomes four times at simple interest rate of 5%. At what rate it becomes seven times
(a) 8% (b) 10% (c) 12% (d) 14% (e) None
19. Arya borrows rupees 6000 from a bank at SI. After 4 years she paid Rs 2500 to the bank and at the end of 5 years
from the date of borrowing he paid Rs 4560 to settle the account. Find the rate of interest (approx)
(a) 3.25% (b) 3.50% (c) 3.85% (d) 4% (e) None
20.A sum of money lent out at simple interest amounts to rupees 840 in 3 years and to rupees 1200 in 8 years. Find the
rate of interest
(a) 11.8/13% (b) 11.7/13% (c) 12.8/13% (d) 12.7/13% (e) None
21. The simple interest on a certain sum of money at 4% per annum for 5 years is 100 more than the interest on the
same sum for 3 years at 5% per annum. Find the sum
(a) 1000 (b) 1500 (c) 2000 (d) 2500 (e) None
22.A sum of rupees 4800 is lent out in two parts in such a way that the interest on one part at 10% for 4 years is equal to
that on another part at 8% for 7 years. Find the two sums
(a) 2800, 2000 (b) 2400, 2400 (c) 2600, 2200 (d) 2700, 2100 (e) None
23. The simple interest on a sum of money will be rupees 400 after 5 years. In the next 5 years the principal is doubled,
what will be the total interest at the end of the 10th year?
(a) 800 (b) 1000 (c) 1200 (d) 1600 (e) None
24. What annual installment will discharge a debt of rupees 1060 due in 4 years at 4 % simple interest?
(a) 200 (b) 250 (c) 300 (d) 400 (e) None
25. Anil borrowed some money at the rate of 5 percent per annum for the first 2 years, 3 percent for the next 4 years
and 4 percent per annum beyond 6 years. If the total interest paid by him at the end of 9 years is rupees 3400, how
much money did anil borrow.
(a) 5000 (b) 8000 (c) 10000 (d) 12000 (e) None
26. Saroj invested 20000 rupee in fixed deposit at the rate of 10% simple interest. After every 3 rd year he added
interest to principal. Find the interest earned at the end of 6th year.
(a) 7800 (b) 8000 (c) 7600 (d) 8200 (e) None
27. Cost of a Mobile Rs.8000. Sudha bought Mobile in EMI. She paid a Down payment of Rs. 2000 and paid rest in 6
equal installments of Rs.1020 for next 6 months. Then what is the SI rate charged?
(a) 6.5% (b) 6.95% (3) 10.5% (d) 12.5% (e) None
28. What sum of money will amount to Rs. 520 in 5 years and Rs. 568 in 7 years at simple interest?
(a) Rs 300 (b) Rs 350 (c) Rs 400 (d) Rs 450 (e) Rs 650
PAGE- 15
29. If Rs. 12000 is lent at 10% p.a. (compound interest) compounded half yearly for one year. What is the equivalent
simple rate of interest for 1 year for same interest?
(a) 10.25% (b) 9.24% (c) 6.26% (d) 8.42% (e) 12.25%
30. C.I. and S.I. for a certain sum at certain rate of interest for 2 years are Rs. 220 and Rs. 200. Find the principal amount.
(a) Rs. 2200 (b) Rs. 200 (c) Rs. 500 d) Rs. 2000 (e) Rs. 2400
31. A gave some amount at simple rate of interest of 10% p.a to B for 3 years and B gave this amount to C at the rate of
20% p.a for 3 years. If the income of B was Rs. 1560 after 3 years, what amount A had given to B?
(a) Rs. 6200 (b) Rs. 5400 (c) Rs. 5200 (d) Rs. 5600 (e) Rs. 4800
32. The simple interest charged on an amount of Rs. 22,500 at the end of four years is Rs. 10,800. What will be the
compound interest on the same amount at the same rate at the end of two years?
(a) Rs. 14,908 (b) Rs. 5,724 (c) Rs. 26,234 (d) Rs. 8,568 (e) Rs. 9,656
33. A sum of Rs. 91,000 is borrowed at 20% per annum compounded annually for two year Rs. If it were borrowed at the
rate of 100/7% per annum simple interest for four years then, find the difference between C.I and S.I
(a) Rs. 16,910 (b) Rs. 12,800 (c) Rs. 12,960 (d) Rs. 11,960 (e) none
34. A person takes a loan of Rs. 200 at 5% p.a compound interest. He returns Rs. 100 at the end of one year. How much
amount he would require to pay at the end of 2nd year in order to clear his dues.
(a) Rs.125.50 (b) Rs.110 (c) Rs.115.50 (d) Rs.115 (e) Rs.120
35. Find the difference between simple and compound interest on Rs. 6000 for 1 year at 20% per but interest is
calculated on half yearly basis.
(a) Rs.120 (b) Rs.60 (c) Rs.180 (d) Rs.72 (e) Rs.108
36. The simple interest on certain sum at 5% p.a for 9 months is Rs. 10 greater than the simple interest on the same sum
@ 3% p.a for 14 months. What is the sum of interest in both the cases (i.e., total sum of interest)?
(a) Rs. 130 (b) Rs. 290 (c) Rs. 120 (d) Rs. 330 (e) Rs. 430
37. Mr. Lala Ram has lent some money to Aaju at 6% p.a. and to Baaju at 8% p.a. At the end of the year he has gained
the overall interest at 7% per annum. In what ratio has he lent money to Aaju and Baaju?
(a) 2 : 3 (b) 1 : 1 (c) 5 : 6 (d) 4 : 3 (e) 5: 7
38. HariLal and Hari Prasad have equal amounts. HariLal invested all his amount at 10% p.a compounded annually for 2
years and Hari Prasad invested 1/4th amount at 10% p.a compound interest (annually) and rest at r% per annum at
simple interest for the same 2 years period. The amount received by both at the end of 2 year is same. What is the r?
(a) 14% (b) 12.5% (c) 10.5% (d) 11% (e) 8.5%
39. HDFC lends 1 million to HUDCO at 10% simple interest p.a. for 3 years and HUDCO lends the same amount to
SAHARA STATES HOUSING Corporation at 10% p.a. on compound interest for 3 years. What is the earning of HUDCO in
this process?
(a) Rs. 1, 33,100 (b) Rs. 33,100 (c) Rs. 31,000 (d) Rs. 15,000 (e) none
40. A sum at r% p.a compound interest doubles in 3 years. In 9 years it will be k times of the original principal. What is
the value of k?
(a) 10 (b) 9 (c) 6 (d) 8 (e) 12
PAGE- 16
DATA INTERPRETATION – 1
Directions (1 – 5): Study the following graph carefully and answer the given questions. The table shows the discount
rate of four different items in five different shops.
Shops Item – 1 Item – 2 Item – 3 Item – 4
A 25% 15% -- 30%
B 20% -- 16% --
C -- 12% -- 15%
D 10% 30% 20% --
E 30% -- 10% 20%
Note: Selling price for each item is same in all the shops
1. Quantity I: If the ratio of the marked price of item 1 and item 4 in shop E is 15: 14, and the marked price of shop D in
item 1 is Rs. 7000. Find the selling price of item 4 in shop E?
Quantity II: In item 2, if marked price in shop C is 25% more than the cost price and the profit percentage of shop C is
10% which is equal to 280. Find the cost price of shop D if marked price of item 2 in shop D is 10% more than the cost
price?
a) Quantity I > Quantity II
b) Quantity I ≥ Quantity II
c) Quantity I < Quantity II
d) Quantity I ≤ Quantity II
e) Quantity I = Quantity II (or) Relationship cannot be determined
2. Quantity I: Item 3, cost price of all the shops is Rs. 4800 and the marked price is 40%, 50% and 25% more than the
cost price in shop B, D and E respectively. Find the total selling price of shop B, D and E together
Quantity II: If the selling price of item 4 of all the shops is Rs. 9520, find the total marked price of shop A, C and E?
a) Quantity I > Quantity II
b) Quantity I ≥ Quantity II
c) Quantity I < Quantity II
d) Quantity I ≤ Quantity II
e) Quantity I = Quantity II (or) Relationship cannot be determined
3. Quantity I: In shop E, Cost price of item 1 is Rs. 800 more than the cost price of item 4 and the marked price of item
1 and 4 is 60 % and 80 % more than the cost price respectively. Find the selling price of item 4 if the selling price is same
for both the items?
Quantity II: If the marked price of item 3 in shop B is 20% more than the cost price, which is 6000, find the marked price
of item 3 in shop D?
a) Quantity I > Quantity II
b) Quantity I ≥ Quantity II
c) Quantity I < Quantity II
d) Quantity I ≤ Quantity II
e) Quantity I = Quantity II (or) Relationship cannot be determined
4. In shop D, if the ratio of marked price in item 1, item 2 and item 3 is 56: 72: 63 and the marked price of item 1 in
shop B is Rs.6300.
Quantity I: Find the marked price of item 2 in shop A
Quantity II: Find the marked price of item 3 in shop E
a) Quantity I > Quantity II
b) Quantity I ≥ Quantity II
c) Quantity I < Quantity II
d) Quantity I ≤ Quantity II
e) Quantity I = Quantity II (or) Relationship cannot be determined
PAGE- 17
5. Quantity I: Find the marked price of item 4 in shop A. If the marked price of item 4 in shop E is Rs. 6300
Quantity II: Find the marked price of item 2 in shop C. If the marked price of item 2 in shop D is Rs. 4400
a) Quantity I > Quantity II
b) Quantity I ≥ Quantity II
c) Quantity I < Quantity II
d) Quantity I ≤ Quantity II
e) Quantity I = Quantity II (or) Relationship cannot be determined
Directions (6 – 10): Study the following graph carefully and answer the given questions. .
The table shows the number of days taken by Swathi to complete the given percentage of work and the time ratio of
Swathi to Shivani to complete the whole work.
Job Name % of work done by swathi Number of days taken by Ratio of days taken by Swathi
Swathi and Shivani Alone
A 20% 5 5:4
B 50% 15 6:5
C 40% 12 2:1
D 15% 3 5:6
E 30% 6 4:5
6. Find the number of days taken by Sumaya to complete the Job-C alone?
Statement I: Swathi and Shivani started working to complete Job-C, Sumaya also joins with them to complete the work
4 days before the actual time taken by Swathi and Shivani working together.
Statement II: Swathi started working to complete Job-C with 75% of her original efficiency. After some days Sumaya
joins with her and completed Job-C in 30 days.
a) Only statement I alone is sufficient to answer
b) Only statement II alone is sufficient to answer
c) Either statement I or II alone is sufficient to answer the question
d) Both statements I and II alone are sufficient to answer the question
e) Both statements I and II alone are not sufficient to answer the question
PAGE- 18
d) Both statements I and II alone are sufficient to answer the question
e) Both statements I and II alone are not sufficient to answer the question
Directions (11 -15): Study the following information carefully and answer the questions given below:
The pie chart represents percentage wise distribution of total number of students in five schools.
The table represents ratio of number of local students and non-local students.
Total number of students in five schools = 8000
11. Out of the total local students of school P 40% are girls. Find the difference between number of local girls and
number of local boys in school P.
a) 280 b) 420 c) 240 d) 360 e) None of these
12. Find the respective ratio of number of local students in school Q and number of non-local students in school S.
a) 5:4 b) 2:1 c) 3:2 d) 4:3 e) None of these
13. Ratio of number of boys and number of girls in school R is 5:4 respectively. If out of the local students in school R,
60% are girls, find the number of non-local girls in school R.
PAGE- 19
a) 160 b) 540 c) 240 d) 480 e) None of these
14. Number of local students in school T and school S together is what percent more/less than the number of local
students in school P?
a) 40% more b) 35% less c) 40% less d) 35% more e) None of these
15. Find the total number of non-local students in all the schools together.
a) 5220 b) 3840 c) 2460 d) 4220 e) None of these
Directions (16 - 20): Study the following information carefully and answer the given questions:
The following bar graph shows the production of mobiles (In thousands) of different companies in two different years.
100
80
60
2016
40
2017
20
0
A B C D E F
16. The average number of mobiles exported by company D and F in 2017 is 35000 while the mobiles exported by
company D in 2016 is twice the export in 2017 then find the percentage of export by company F out of total production
of company F in 2017?
a) 65 % b) 72 % c) 80 % d) 85 % e) 92 %
17. If company E exported a total of 50000 mobiles in the year 2016 and 2017, then find the percentage of mobiles
exported by company E in 2016?
a) 35 % b) 27.5 % c) 20 % d) 17.5 % e) None of these
18. The total number of mobiles exported by company A in 2016 and 2017 is 2/3 of the mobiles exported by company C
in these years then find the percentage of mobiles that were exported by company C out of total production in 2017?
a) 72 % b) 56.25 % c) 45 % d) 64.75 % e) 75 %
19. The 20% of the mobiles exported by company B in 2016 are defective while the ratio between the defective mobiles
exported by company B in the year 2016 and2017 is 7 : 5 and the percentage of defective mobiles out of total export by
company B in 2017 is 13.5% then find the percentage of mobile exported by company B out of total production?
a) 45.66 % b) 72.77 % c) 48.55 % d) 66.67 % e) 56.33 %
20. Find the difference between the mobiles exported by company A, B and D in 2016 to that of the mobiles exported
by company A and E in 2017?
a) 18500 b) 24200 c) 20600 d) 19700 e) None of these
PAGE- 20
DATA INTERPRETATION – 2
Directions (1-5): Study the table and answer the given questions.
1. The number of male post graduate employees in country H is 1800. If number of female post graduates increase by
50% in the next year, what % of female employees in that particular country is post graduate? (Given that all other data
remain same)
(a) 76.8% (b) 74% (c) 92.5% (d) 90% (e) 80%
2. In which country, is the percentage of women employees to number of employees (both male & female) is ranked
third lowest?
(a) E (b) B (c) H (d) F (e) A
3. What is the ratio between total number of male employees in countries B and H together and total number of post
graduate employees in same countries?
(a) 76 : 65 (b) 86 : 85 (c) 75 : 76 (d)65 : 76 (e)12 : 33
4. What is the difference between average number of post graduate employees in countries A, B and D together and
average number of post graduate employees in countries F, G and H together?
(a) 294 (b) 282 (c) 284 (d) 280 (e) 200
5. Which country has the 2nd highest number of average employees per office?
(a) D (b) H (c) G (d) A (e) F
Directions (6-10): The table given below shows the no. of units produced of six different items by a company, the
mark-up % on each unit and the discount offered on the marked-up price of each unit.
The cost price of all the items is same and fixed at Rs 100.
The line graph shows the estimated percentage of items sold by the company on the normal discounted price.
The bar graph shows the percentage of cost price at which the company sold the remaining no. of items.( means the
company sold the remaining no. of items at a price lower than the cost price)
A B C D E F
Production Unit 200 160 80 140 180 150
Mark UP % 50 60 80 40 60 45
Discount % 20 25 40 15 20 20
PAGE- 21
Series 1,
Estimated sales % out of total Production
100
90
80 75 80
70
50 60
Series 1
0
A B C D E F
100 90
80
80 70
60
60
40
40 30
20
0
6. Find profit % of A and B together in year 2015if it is known that B sold only 90% of goods of what he actually
estimated to sell in year 2015.
(a) 0.33% (b) 0.44% (c) 0.55% (d) 0.60% (e) None of these
7. In year 2016 E has increased his discount % by 10 basis point and which lead to increase in its estimated sales by 20
basis point. Apart from that everything else remains same then what is the difference in profit in year 2015 to 2016.
(a) 504 (b) 508 (c) 512 (d) 516 (e) 520
8. In year 2016 C decreased the discount % by 10 basis point due to which its sales reduced to 80% of total production
of year 2015. By how much % profit will increase or decrease in 2016 compare to year 2015. (Approximately)
(a) Increased by 200% (b) Decreased by 210% (c) Increased by 203%
(d) Increased by 207% (e) Decreased by 207%
9. What is the difference between the absolute profit of A, B and C together and D, E and F together in year 2015
(a) 750 (b) 800 (c) 900 (d) 1000 (e) None of these
10. What is the profit % of all the companies together in year 2015. (Approximately)
(a) 4.9% (b) 4.6% (c) 5.1% (d) 4.7% (e) 5.4%
DIRECTIONS for questions (11 to 13): The following table gives the data about big 12 software exporters in India from
1997-98 to 1999-2000 (Revenue in Rs. crore).
11. What is the lowest rate of growth of 1998-99 to 1999-2000 for any of the companies given in the chart?
(a) 0% (b) 12.7% (c) 19.1% (d) 10.1% (e) 5.3%
12. Which of the following statement is true about the share of exports of Tata Consultancy Services in the total
exports of the 12 companies?
(a) It has almost doubled in 1999 – 00 over 1997 – 98.
(b) It has been steadily increasing
(c) It is more or less constant.
(d) It has dropped by about 10 %.
(e) None of these.
13. If total software exports in 2000 – 2001 were expected to be Rs 10,000 crore and the growth of Infosys is the same
as it was in 1999 – 2000, what would have been Infosys’ share of exports of these 12 companies in 2000 – 2001?
(a) 15% (b) 19% (c) 21% (d) 23% (e) None of these
(DIRECTIONS for questions 14 & 15): Study the following graph and answer the questions that follow.
14. In which year was the value per tin the minimum?
(a) 1987 (b) 1984 (c) 1985 (d) 1986 (e) 1983
15. If in 1986 tins were exported at the same rate per tin as in 1985, then what would be the value of exports in 1986?
(Crores of Rupees)
(a) 400 (b) 420 (c) 375 (d) 330 (e) 352
(Directions for questions 16 to 20): The following diagram shows the percentage share of manufacturing sector in total
employment in small, medium and large establishments individually. The definitions of small, large and medium
establishments are shown below in the diagram
PAGE- 23
Note: Small establishments are defined as those with fewer than 100 employees. Medium-sized establishments are
defined as those with between 100 and 1500 employees. Large establishments are defined as those with more than
1500 employees. Study the diagram carefully and answer the questions given below:
16. In 1981 and 1987 about 50 million and 60 million people were employed in small establishments. What was the
difference between the no. of employees employed in manufacturing sector small establishments in 1981 and 1987?
(a) 4.5 million (b) 13.5 million (c) 5 million (d) 6.3 million
17. In 1981 about 30 million employees were there in the large establishment. Then how many employees were there
in the medium-sized establishments? (Approximately)
(a) 25 m (b) 40 m (c) 35 m (d) Can’t be determined
18. In 1987, about 40 million employees were there in the large establishments. The population of employees from
1987 to 2000 grew by 60% in large establishments. How many employees were there in the manufacturing sector in
large establishments in 2000?
(a) 33.2 m (b) 69.3 m (c) 19.2 m (d) 47.2 m
19. Suppose that in 1996, there were 100,000 employees in manufacturing sector. There were 750 small
establishments. What was the average number of employed persons per small establishment manufacturing unit?
(a) 36 (b) 41 (c) 56 (d) can’t be determined
20. Which of the following is/are correct?
I. Share of manufacturing employment in small establishment is rising steadily.
II. In large establishments there is growing unemployment.
(a) I only (b) II only (c) I and II (d) Neither I nor II
PAGE- 24
DATA SUFFICIENCY
Directions: In each of the following questions, a question is followed by two or three statement. Read all the
statements and find that which statements are required to answer the question and answer accordingly.
1. How much time will Train P take to cross Train Q (from the moment they meet) running in opposite directions
(towards each other) ?
Statement I: The respective ratio of speeds of Train P and Train Q is 3 : 4. The sum of the lengths of Train P and Train
Q is 700 metre.
Statement II: Train P can cross a signal pole in 12 seconds. It can cross 600 metre long station in 25 seconds.
A) Only I
B) Both I and II
C) Only II
D) Either I or II
E) Neither I nor II
4. What is the age of R, in a group of P,Q, R,S and T whose average age is 45 years?
Statement I: Average of the age of S and T is 47 years?
Statement II: Average of the age of P and Q is 53 years?
A) Only II
B) Only I
C) Both I and II
D) Neither I nor II
E) Either I or II
PAGE- 25
C) Only I
D) Neither I nor II
E) Both I and II
6. The ratio between the present ages of the Rohit and Rina is 1 : 3. Find the present age of the Rina.
Statement I: Difference between the present ages of the Pooja and Rohit is 22 years.
Statement II: The present age of Pooja is 4 years less than thrice the present age of Rohit.
Statement III: Difference between the present ages of the Rina and Rohit is 26 years.
A) Only III
B) Either I and II together or III alone.
C) All are together
D) Only I and II
E) None of the statements
9. A,B,C,D and E are five friends. Their mean age is 18. What is the age of C ?
Statement I : A’s age is 18
Statement II : B’s age is 2 years less than E and E’s age is 6 years less than D.
Statement III : C’s age is 6 years more than B’s age and 4 years more than E’s age.
A) Only III
B) Neither I and II nor III
C) Only I and III
D) All statements together
E) Either I and III or II alone
15. Two persons P and Q . P bought some copies and some books from a shopkeeper .The shopkeeper charged extra
Rs.15 ,though he was getting a profit of Rs. 60/copy and Rs 350/book . For what price did P and Q sell the book if
they both earned a profit of 50%?
Statement I: The selling price of 4 copies and 2 books was Rs. 600.
Statement II: P and Q sold equal number of copies and books .
Statement III: They invested a total amount of Rs.10,000 on copies and Rs .15,000 on books.
PAGE- 27
A) Only II
B) None of the above
C) Only II and III
D) Only I and II
E) All of these
16. In how many days will Mohit alone complete the work?
Statement I: Soham alone can do the work in 20 days .Mohit is 10% more efficient than Soham and Rita . 30% of
work is done by Mohit ?
Statement II: Soham and Mohit together can finish the work in 8(2/11) days , Mohit and Rita in 7(1/5) days and,
Rita and Soham can do the same work in 6(2/3) days .
A) Only II
B) Both I and II
C) Either I or II
D) Neither I nor II
E) Only I
18. Raman mixed two types of wheat for selling the mixture in his shop .What is the quantity (in kg) of the first type
wheat in the mixture ?
Statement I: The price of the second type of wheat is Rs. 50/kg . The difference between the quantity of first type
of wheat and second type of wheat in the mixture is 3 kg.
Statement II: The price of the first type of wheat is Rs. 47 /kg. Raman earned a profit of 20% by selling the
mixture at the rate of Rs. 57.60 /kg.
A) Neither I nor II
B) Either I or II
C) Both I and II
D) Only I
E) Only II
24. Find the increase percent in Cost Price to make Marked Price if
Statement I: A trader gain 25% on Cost Price
Statement II: A trader gain 20% on Selling Price
Statement III: A trader gain 20% on Cost Price after giving a discount of 20% on Marked Price
A) I and II Together
B) II and III Together
C) Only III
D) All I, II and III Together
E) None of these
Directions (25-30): In each of the following questions, a question is followed by two statements numbered I
and II. Read both the statements and answer accordingly.
PAGE- 30
31. What is the principal amount?
Statement I: The Simple Interest obtained on the principal after 2 years at 8% rate of interest is Rs 450 less than
the compound interest obtained on the same principal after 2 years at 8% per annum.
Statement II: The sum becomes double in 10 years at 6% per annum rate of simple interest.
Statement III: The compound interest obtained on the principal amount is Rs 4540 after 2 years at the rate of 8%
compounded annually.
A) Only I B) All I, II and III together C) II and III together
D) Only I or III E) I or II and III
33. A person borrowed some money at compound interest for 2 years. What will be the amount required to return
after 2 years?
Statement I: If the amount was borrowed at simple interest, then after 5 years Rs 600 was required to pay as
simple interest.
Statement II: The rate of interest is 6% per annum
Statement III: The sum of money borrowed is 10 times the amount required to be paid as simple interest after 2
years.
A) II and I or III
B) Even all statements together cannot answer the question
C) II and III together
D) I and III together
E) All I , II and III
34. Shopkeeper gained how much by selling his products in November 2016?
Statement I: He earned 40% more profit in December 2016 as compared to October 2016.
Statement II: In December 2016 he earned 10% more profit than in November 2016.
Statement III: The total profit earned in November 2016 and October 2016 was Rs 55,000.
A) Only I and III
B) Only II and III
C) Only I and II
D) All I, II and III
E) Even all statements together cannot answer the question
35. In how many days 6 men and 5 women can complete the work working together?
Statement I: The ratio of efficiency of man to woman is 2 : 1
Statement II: 9 Women can complete three-five of the work in 15 days.
Statement III: 6 Men and a child together can complete 1/4th work in 12 days while 6 women and the child
together can complete two-third of the work in 16 days
A) I and either II or III
B) Any combination of 2 statements can give the required result
C) Only I and II
D) All I, II and III
E) None of these
PAGE- 31
SEATING ARRANGEMENT
Direction 1-5: Read the following information carefully and answer the following questions.
A ,B , C, D ,E ,F and G are sitting in around a circle and are facing the centre. G is the second to the left of C, who is to the
immediate left of F. A is third to the left of E. B is between D and E.
3. Which of the following pair has the first person sitting to the immediate left of the second person?
1) BE 2) CA 3) GD 4) DG 5) None of these
4. Which of the following has the middle person sitting between the remaining two ?
1) FCE 2) EFB 3) DEB 4) GDA 5) None of these
Direction 6-10: Read the following information carefully and answer the following questions.
P, Q, R, S, T, U and V are sitting on a wall facing East. R is on the immediate right of S.Q is at an extreme end and has T
as his neighbor. V is between T and U.S is sitting between R and U.S is sitting third from the south end.
7. Which of the following pairs of people are sitting at the extreme ends ?
1) PQ 2) PT 3) RQ 4) UQ 5) None of these
8. Name the person who should change place with R such that he gets the third place from the North end ?
1) T 2) U 3) V 4) S 5) None of these
Direction 11-15: Read the following information carefully and answer the following questions.
PAGE- 32
A ,B ,C ,D ,E ,F ,G and H are sitting around a circular table with equal distance between them, facing the centre but not
necessarily in the same order. A sits second to the right of G. Only 2 persons sitting between A and H.C sit second to the
left of H.Only 3 persons sit between H and F.B sits on the immediate left of D.
12. If all the persons are made to sit in alphabetical order in clockwise direction, starting from A, the positions of how
many, excluding A would remain unchanged?
1) One 2) Two 3) Three 4) Four 5) None of these
15. Four of the following are alike as per the given arrangement and thus form a group, which of the following does not
belong to that group ?
1)HA 2)GH 3)CG 4)BC 5)FD
Direction 16-20: Read the following information carefully and answer the following questions.
A ,B ,C ,D ,E ,F ,G and H are sitting around a circular table. Two of them are not facing the centre but sit opposite each
other. G is the second to the right of A and third to the right of C. B is second to the left of C and fourth to the right of D.
E is second to the right of H and is facing the centre. The person facing outwards is an immediate neighbor of G and A. D
sits second to the right of C and is not a immediate neighbor of A.
19. Who among the following are not facing the centre ?
1) CA 2) CF 3) GF 4) EB 5) None of these
Direction 21-25: Read the following information carefully and answer the given questions carefully.
Seven people A,B,C,D,E,F and G are sitting in a straight line with equal distance between them, but not necessarily in
the same order. Some of them are facing North and some are facing South. Only two people are sitting to the left of G.
Only two people sit between G and B. A sits second to the left of B. The immediate neighbors of A face opposite
directions. Only one person sits between A and C. F sits third to the left of C. D is not an immediate neighbor of B. Both
the immediate neighbors of C face the same direction. A faces the same direction as that of C. E faces North. B sits to
the immediate left of E.
PAGE- 33
21. Who among the following sits exactly between G and the one who is sitting to the immediate left of B ?
1) F 2) A 3) C 4) B 5) None of these
22. Who among the following sits exactly in the middle of the line ?
1) A 2) C 3) D 4) E 5) F
Direction 26-30: Study the following information carefully to answer the given questions.
P, Q, R, S, T ,V and W are sitting in a straight line facing north. Each of them live on a different floor in the same building,
which is numbered from 1 to 7. Q sits fourth to the left of the person living on the 6th floor. Either Q or the person living
on the 6th floor sits at the extreme at ends of the line. Only one person sits between Q and W. W lives on the 3rd floor.
The person living on 1st floor sits third to right of S. S is not an immediate neighbor of W. Only one person lives between
T and the person who lives on the second floor. P and R are immediate neighbors of each other. P does not live on the
6th floor. One who lives on 5th floor sits third to right of the one who lives on the 7th floor.
27. Four of the following five are alike in a certain way based on the given arrangement, which of the following does not
belong to that group ?
1) P 2) Q 3) S 4) T 5) W
28. How many floors are there between the floors on which V and P live ?
1) Four 2) Three 3) Two 4) One 5) None of these
30. If all the persons are instructed to live in alphabetical order from the bottom-most floor to the top most floor. Then
who would still live on the same floor as the original arrangement ?
1) T 2) S 3) R 4) Q 5) P
Directions (31-35) Study the given information carefully and answer the given questions .
Eight people S, T, U, V, W, X, Y and Z are sitting in a straight line in facing north but not necessarily in the same order.
Each of them likes a different hobby , namely Cooking , Reading , Singing, Banking , Archery , Knitting ,Painting, and
Drawing but not necessarily in the same order .
T sits fourth to the right of X. The one who likes Singing sits on the immediate right of T . Only two people sit between
the one who likes Singing and the one who likes Painting. The one who likes Drawing sits second to the left of the one
who likes Painting . Only two people sit between U and the one who likes Drawing . Y sits on the immediate left of S . Y
does not sit at any of the extreme ends of the line. Y sits at one of the positions to the left of T . Only three people sit
between the one who likes Singing and one who likes Cooking. As many people sit between T and the one who likes
PAGE- 34
Archery as between T and the one who likes Cooking. Z sits second to the left of the one who likes Archery. The one who
likes Knitting sits third to the right of the one who likes Reading . V does not like Knitting .
31. Which of the following pairs represent those who sit at the extreme ends of the line ?
1) U, W
2) U, The one who likes Singing
3) The one who likes Drawing, V
4) T, the one who likes Archery
5) X, the one who likes knitting
33) Four of the following five are alike in a certain way based on their seating positions in the given seating arrangement
and so form a group Which one does not belong to the group ?
1) S-Cooking 2) X-Reading 3) Y-Drawing 4) V-Knitting 5) U-Painting
Directions (36-40)Study the given information carefully and answer the given questions .
Eight friends M, N, O, P, Q, R, S and T are sitting around a circular table with equal distance between them but not
necessarily in the same order. Some of them are facing the centre while some face outward.
O sits second to the right of T. T faces the centre. Only three people sit between O and N . S sits on the immediate right
of N.M sits on the immediate right of P . P and N face the same direction. The immediate neighbors of M face the same
direction. Q sits third to the right of O . Q is not an immediate neighbor of M .Q and O face opposite direction. The
immediate neighbors of N face the same direction. Both R and M face a direction opposite that of P .
PAGE- 35
38) Who sits third to the left of O ?
1) S 2) N 3) P 4) O 5) M
39) Four of the following five are alike in a certain way based on their seating positions in the given seating arrangement
and so form a group Which one does not belong to the group ?
1) S 2) Q 3) M 4) O 5) R
40) How many sit between S and R if counting start from clockwise direction.
1) Three 2) None 3) One 4) More than three 5) Two
Directions (41-45):-Study the following information carefully & answer the given questions.
A, B, C, D, E, F, G & H are sitting around a square table in such a way that four of them sit at four corners of the square
while four sit in the middle of each of the four sides. The ones who sit at the four corners face the centre of the table
while those who sit in the middle of the sides face outside. each of them likes a different subject, Viz, Mathematics,
Hindi, English, Botany, Chemistry, Psychology, History & Geography.
C sits third to the left of the one who likes Geography. The one who likes Geography faces outside. Only two persons sit
between C & H. The one who likes Mathematics sits on the immediate right of H. The one who likes Chemistry sits
second to the right of G. G is neither an immediate neighbour of H nor of C.G does not like Geography. Only one person
sits between A & the one who likes Chemistry. D sits on the immediate left of the one who likes Psychology. G does not
like Psychology. E likes History. E is not an immediate neighbour of A. The one who likes Hindi is an immediate
neighbour of E. The one who likes Botany is an immediate neighbour of F.
41) Who amongst the following sits diagonally opposite the one who likes Mathematics?
1) The one who likes Hindi 2) D 3) A
4) The one who likes English 5) None of these
42) Who among the following represent the immediate neighbours of the one who likes Chemistry?
1) B, F 2) C, E 3) B, E 4) D, F 5) F, H
Directions (46-50):-Study the following information carefully & answer the given questions.
Eight people-A,B,C,D,E,F,G & H – are sitting a circular table facing the centre. Each of them live in different city namely ,
Mumbai, Pune, Lucknow , Chennai, Surat, Kochi ,Jaipur and Aurangabad, but not necessarily in the same order.
The one who lives in Mumbai sits second to the right of H . A is an immediate neighbor of the one who lives in Mumbai.
Only three people sit between A & the one who lives in Chennai. Only two people sit (from either left or right ) between
the one who lives in Chennai and the one who lives in Lucknow. F is an immediate neighbor of one who lives in
Lucknow. As many people sit between F and the one who lives in Aurangabad as between F and the one who lives in
Chennai . D sits second to the left of the one who lives in Aurangabad. C sits to the immediate left of B . The one who
lives in Surat sits second to the right of C. D neither lives in Pune nor Kochi .F does not live in Pune . E does not live in
Lucknow.
46) Who is sitting second to the right of the one who lives in Kochi ?
1) D
2) The one who lives in Pune
3) A
PAGE- 36
4) B
5) The one who lives in Surat
47) Which of the following statements is true according to the given arrangement ?
1) The one who lives in Mumbai is not an immediate neighbor of H
2) All of the given statements are true
3) Only two people sit between C and A when counted form the left of C
4) The one who lives in Aurangabad sits second to the right of one who lives in Jaipur
5) B lives in Chennai
48) Four of the following five are alike in a certain way based on the given arrangement and hence they form a group .
which of the following does not belong to that group ?
1) G-Surat
2) C-Mumbai
3) H-Pune
4) B-Aurangabad
5) F-Jaipur
PAGE- 37
SYLLOGISM
Directions (1-5): In each question below are given three statements followed by two conclusions numbered I and II.
You have to take the given statements to be true even if they seem to be at variance with commonly known facts.
Read all the conclusions and then decide which of the given conclusions logically follows from the given statements,
disregarding commonly known facts. Give answer:
a. If only conclusion I follows
b. If only conclusion II follows
c. If either conclusion I or II follows
d. If neither conclusion I nor II follows
e. If both conclusion I and II follow.
1).Statements:
All rains are rivers
Some rains are waters
Some rivers are seas
Conclusions:
I. All rivers being water is a possibility II. All rains can be water
2).Statements:
All books are papers No book is a chair
Some chairs are tables
Conclusions:
I. At least some papers are books II. All tables being paper is a possibility
3).Statements:
No triangle is a square
No circle is a rectangle
Some squares are circles
Conclusions:
I. Some squares are not rectangle II. Some rectangles are square
4).Statements:
All cups are plates
All plates are boxes No box is spoon
Conclusions:
I. All plates being spoon is a possibility II. Some boxes are not plate
5).Statements:
All bikes are cars
Some cars are buses All buses are trucks
Conclusions:
I. All buses are cars II. Some cars are trucks
Directions (Q.6-10): In each question below are given three statements followed by three conclusions numbered I, II
and III. You have to take the given statements to be true even if they seem to be at variance with commonly known
facts. Read all the conclusions and then decide which of the given conclusions logically follows from the given
statements, disregarding commonly known facts.
6). Statements:
Some books are notes No paper is a pen
All notes are papers
Conclusions:
I. All books being paper is a possibility
PAGE- 38
II. All pens being books is a possibility
III. All notes being books is a possibility
a) Only I follows b) Only II follows c) Only III follows d) Only I and II follows e) None of these
7).Statements:
All pins are threads. Some threads are fingers. All fingers are nails.
Conclusions:
I. Some fingers are pins
II. At least some fingers are threads
III. All nails are fingers
a) Only I follows b) Only II follows c) Only III follows d) None follows e) None of these
8). Statements:
Some additions are subtractions
No subtraction is multiplication
All multiplications are divisions
Conclusions:
I. Some divisions are additions
II. Some multiplications are additions
III. No division is addition
a) None follows b) Only I follows c) Only II follows d) Only either I or III follows
e) Only III follows
9). Statements:
All flowers are leaves, All leaves are trees. Some trees are fruits
Conclusions:
I. Some fruits are leaves
II. Some fruits are flowers
III. Some trees are flowers
a) Only I follows b) Only I and II follows c) Only II and III follows d) Only III follows
e) None of these
10). Statements:
All villages are towns
No town is city
All cities are states
Conclusions:
I. Some states are village
II. all cities are towns
III. no villages are cities
11). Statements:
Some trees are mangoes
No mango is a banana
All bananas are fruits
Conclusions:
I. At least some fruits are trees II. All trees can be bananas
a) a b) b c) c d) d e) e
12). Statements:
All districts are states
Some states are towns
No town is a village
PAGE- 39
Conclusions:
I. All villages can be districts II. Some states are not districts
a) a b) b c) c d) d e) e
13. Statements:
All animals are lions
Some lions are tigers
All lions are fox
Conclusions:
I. All tigers are fox II. Some tigers are not fox
a) a b) b c) c d) d e) e
14. Statements:
All mobiles are boxes.
Some boxes are bulbs.
No bulb is a switch
Conclusions:
I. Some bulbs are boxes II. Some boxes are not switches
a) a b) b c) c d) d e) e
15. Statements:
No car is a bike
All bikes are cycles
No bus is a car
Conclusions:
I. Some cycles can never be cars II. Some buses can be cycles
a) a b) b c) c d) d e) e
16. Statements:
All teas are coffee
No coffee is milk
No milk is water
Conclusions:
I. Some water can be tea II. At least some milk are not coffee
a) a b) b c) c d) d e) e
17. Statements:
All cups are caps
All caps are shirts
No cap is cloth
Conclusions:
I. All cloths are shirts II. Some cloth is not shirt
a) a b) b c) c d) d e) e
18. Statements:
Some books are notes
All books and notes are materials
No book is a test
Conclusions:
I. Some materials are not books II. No note is a test
a) a b) b c) c d) d e) e
Direction (Q. 19-20): In each question below are given four statements followed by four conclusions. You have to
PAGE- 40
take the given statements to be true even if they seem to be at variance with commonly known facts. Read all the
conclusions logically follows from the given statements, disregarding commonly known facts.
19. Statements:
Some pens are papers . No paper is a pencil.
All copies are pencils. All pencils are inks.
Conclusions:
I. Some pens are not copies II. All pencils being ink is a possibility
III. All pens are pencils IV. Some copies are not papers.
a) Only I and II follows b) Only I and IV follows c) Only II, III and IV follows
d) Only I, II and IV follows e) All follows
20. Statements:
All desks are tables. No desk is a chair. All notes are chairs. Some chairs are benches
Conclusions:
I. Some notes are benches II. Some tables are not chairs
III. Some notes are not desks IV. Some tables being bench is a possibility
a) Only I, II and III follows b) Only II, III and IV follows c) Only I, II and III follows
d) None follows e) All follows
Directions (Q. 21-25):In each of the given questions, some statements are followed by two conclusions I and II. You
have to assume everything in the statement to be true even if they seem to be at variance with commonly known
facts, and then decide which of the given conclusions logically follows from the statements, disregarding
commonly known facts.
Give answer:
a) if only conclusion I follows.
b) if only conclusion II follows.
c) if either I or II follows.
d) if neither I nor II follows.
e) if both conclusions I and II follow.
21). Statements :
Some resources are powerful.
All resources are significant.
All significant are organized.
Conclusions:
I. Some organized are not significant. II. All organized being powerful is a possibility.
a) a b) b c) c d) d e) e
22). Statements:
All regular are successful.
Some funds are regular. All exercises are funds.
Conclusions:
I.All exercises being successful is a possibility. II. At least some regular are funds.
a) a b) b c) c d) d e) e
23). Statements:
All regular are successful.
Some funds are regular.
All exercises are funds.
Conclusions:
I. Some successful being exercises is a possibility. II. Some funds can never be exercises.
a) a b) b c) c d) d e) e
PAGE- 41
24). Statements:
No society is standard. No standard is modern. No modern is official.
Conclusions:
I. Some societies being modern is a possibility. II. Some societies can never be official.
a) a b) b c) c d) d e) e
25). Statements:
All attire are balanced. All balanced are critical.
Some critical are delightful.
All delightful are effective.
Conclusions:
I. All critical being attire is a possibility. II. Some balanced are delightful.
a) a b) b c) c d) d e) e
Directions (Q. 26-30):In each of the given questions, some statements are followed by two conclusions I and II. You
have to assume everything in the statement to be true even if they seem to be at variance with commonly known
facts, and then decide which of the given conclusions logically follows from the statements disregarding
Commonly known facts.
Give answer
a) if only conclusion I follows.
b) if only conclusion II follows.
c) if either I or II follows.
d) if neither I nor II follows.
e) if both conclusions I and II follow.
26). Statements:
Some married are educated.
Some educated are men.
All men are qualified.
Conclusions:
I. Some educated if they are men are qualified. II. Some educated if they are qualified are men.
a) a b) b c) c d) d e) e
27). Statements:
All shares are debentures.
No debenture is equity. Many equities are maturities.
Conclusions:
I. No debenture can be a maturity. II. All debentures that are shares may be equities.
a) a b) b c) c d) d e) e
28). Statements:
Some doctors are rich.
All rich who are doctors are honest.
Conclusions:
I. Many doctors are honest. II. Some honest are possibly rich.
a) a b) b c) c d) d e) e
29). Statements:
No ring is a wing.
Some wings are kings. All kings are brave.
Conclusions:
I. Some brave may be ring. II. Kings which are not wings are rings.
a) a b) b c) c d) d e) e
PAGE- 42
30). Statements:
All queens are beautiful. Some princesses are queens. No beautiful is royal.
Conclusions:
I. All beautiful which are princesses will necessarily be queens.
II. All queens are royal.
a) a b) b c) c d) d e) e
Directions (31-35): In each question below are given four statements followed by two conclusions which is
numbered as I, and II. You have to take the given statements to be true even if they seem to be at variance with
commonly known facts. Read all the conclusions and then decide which of the given conclusions logically follows
from the given statements, disregarding commonly known facts.
Mark your answer as –
(a) If only conclusion I follows.
(b) If only conclusion II follows.
(c) If either conclusion I or conclusion II follows.
(d) If neither conclusion I nor conclusion II follows.
(e) If both conclusion I and conclusion II follow.
31. Statements: All A is B. Some B is not C. All C is D. No A is E
Conclusions:
I. All C can be B II. Some B can be D
a) a b) b c) c d) d e) e
Directions (36-40): Question consists of five statements followed by five conclusions. Consider the given statements
to be true even if they seem to be at variance with commonly known facts. Read all the conclusions and then
decide which of the given conclusions does not logically follow from the given statements using all statements
together.
36. Statements: All M are N. All N are C. No D is N. All K are F. Some K are D
Conclusions:
(a) Some K are not N.
(b) Some C can be K.
(c) Some F is C.
(d) No M are D
PAGE- 43
(e) All N can be K.
Directions (41-45): Each of the following questions consists of six statements followed by options consisting of three
statements put together in a specific order. Choose the options that indicate a combination where the third
statement can be logically deduced from the first two statements and that option will be your answer.
41.
i. Some B are D. ii. All B are X.
iii. No Y are X.
iv. No D is X.
v. All Y are D.
vi. Some Y are B
(a) [i,vi,v]
(b) [iv,v,iii]
(c) [ii,vi,iv]
(d) [iii,iv,v]
(e) None is correct
42.
i. All Q are T
ii. Some S are T.
PAGE- 44
iii. Some T are Q.
iv. No T is R.
v. Some S are not R.
vi. All Q are S
(a) [vi,ii,iii]
(b) [iv,v,iii]
(c) [ii,v,iv]
(d) [ii,iv,v]
(e) None is correct
43.
i. All E are X.
ii. All G is H.
iii. Some G are E.
iv. No J is G
v. Some J is H.
vi. Some X is G.
(a) [iv,ii,i]
(b) [iii,vi,i]
(c) [iv,ii,v]
(d) [iii,i,vi]
(e) None is correct
44.
i. Some G are H.
ii. Some I are G.
iii. All H are I
iv. No G is J.
v. Some I are not J.
vi. All H are G.
(a) [iii,ii, i]
(b) [iv,v,i]
(c) None of these
(d) [ii,v,iv]
(e) [ii,iv,v]
45.
i. Some I are K.
ii. All M are N.
iii. A few I are M.
iv. All N are K.
v. All M are K.
vi. Some I are K.
(a) [iv,v,ii]
(b) [i,iii,v]
(c) [ii,v,iv]
(d) [iii,v,vi]
(e) None of these
Directions (Q. 46-55):In each of the given questions, some statements are followed by two conclusions I and II. You
have to assume everything in the statement to be true even if they seem to be at variance with commonly known
facts, and then decide which of the given conclusions logically follows from the statements disregarding
commonly known facts.
Give answer
PAGE- 45
a) if only conclusion I follows.
b) if only conclusion II follows.
c) if either I or II follows.
d) if neither I nor II follows.
e) if both conclusions I and II follow.
46).Statements: All lions are dogs. All dogs are rat. No lions are tiger.
Conclusions:
I. No rat is a tiger II. Some dogs being tiger is a possibility
a) a b) b c) c d) d e) e
47).Statements: Some red are blue. All yellow are blue. No blue are orange.
Conclusions:
I. All blue being red is a possibility II. Some yellow are yellow is a possibility
a) a b) b c) c d) d e) e
48).Statements: All orange are green. No blue is green. All green are red.
Conclusions:
I. No blue is orange II. All red being orange is a possibility.
a) a b) b c) c d) d e) e
49).Statements: All monkey are rat. Some rat are lion. No lion is a dog.
Conclusions:
I. All monkey being lion is a possibility II. No dog is a rat
a) a b) b c) c d) d e) e
50).Statements: No dollar is yen. Some yen are franc. All rupee are dollar.
Conclusions:
I. No dollar is franc II. No yen is rupee
a) a b) b c) c d) d e) e
51).Statements: All pen are eraser. All eraser are pencil. Some eraser are scale.
Conclusions:
I. All scale being pencil is a possibility II. No eraser is scale.
a) a b) b c) c d) d e) e
52).Statements: No table is bench. All bench are chair. Some chair are huts.
Conclusions:
I. No table is chair II. Some bench are huts
a) a b) b c) c d) d e) e
53).Statements: Some image are picture. Some picture are photo. Some photo are clip
Conclusions:
I. All image are clip II. Some clip are picture.
a) a b) b c) c d) d e) e
54).Statements: All apple are grapes. Some apple are mango. No mango is banana.
Conclusion:
I. All banana being apple is a possibility II. All grapes being banana is a possibility
a) a b) b c) c d) d e) e
55).Statements: All png are bmp. Some bmp are jpg. All gif are jpg.
Conclusions:
PAGE- 46
I. Some bmp are gif II. Some jpg are png.
a) a b) b c) c d) d e) e
Directions (Q56-64): In each question below are given two/three statements followed by two Conclusions
numbered I and II. You have to take the given statements to be true even if they seem to be at variance with
commonly known facts and then decide which of the two conclusions logically follows from the given statements
disregarding commonly known facts. Give answer
Give answer
a) if only conclusion I follows. b) if only conclusion II follows. c) if either I or II follows.
d) if neither I nor II follows. e) if both conclusions I and II follow.
56. Statements:
(i) All bags are purses.
(ii) No purse is black.
(iii) All blacks are beautiful.
Conclusions: I. Some bags being black is a possibility.
II. At least some purses are bags.
a) a b) b c) c d) d e) e
57. Statements:
(i) All bags are purses.
(ii) No purse is black.
(iii) All blacks are beautiful
Conclusions: I. All purses being beautiful is a possibility.
II. Some bags are not black.
a) a b) b c) c d) d e) e
58. Statements:
(i) Some fishes are cats.
(ii) Some dogs are cats.
(iii) No fish is black.
Conclusions: I. At least some cats are not black.
II. There is a possibility that some fishes are dogs.
a) a b) b c) c d) d e) e
59. Statements:
(i) Some fishes are cats.
(ii) Some dogs are cats.
(iii) No fish is black
Conclusions: I. No dog is black.
II. Some cats are black
a) a b) b c) c d) d e) e
60. Statements:
(i) Some colours are paints.
(ii) All colours are varnishes.
(iii) No varnish is dye.
Conclusions: I. No paint-is dye.
II. All paints Being varnishes is a possibility.
a) a b) b c) c d) d e) e
61. Statements:
(i) Some colours are paints.
PAGE- 47
(ii) All colours are varnishes.
(iii) No varnish is dye
62. Statements:
(i) All squares are triangles.
(ii) No triangle is circle.
(iii) All circles are rectangles.
Conclusions: I. No rectangle is square.
II. All rectangles being square is a possibility.
a) a b) b c) c d) d e) e
63. Statements:
(i) All squares are triangles.
(ii)No triangle is circle.
(iii) All circles are rectangles.
Conclusions: I. No square is circle.
II. At least some circles are square.
a) a b) b c) c d) d e) e
64. Statements:
(i) No paper is book.
(ii) Some books are libraries.
Conclusions: I. All libraries being books is a possibility.
II. No library is paper.
a) a b) b c) c d) d e) e
Directions (65-74): In the following questions, only one Conclusion is given and five statements are given as a), b),
c), d) and e. From this you have to take the statements to be true even if they seem to be at variance with
commonly known facts and then decide which of the given statement logically follows.
65. Conclusions:
Some mouse is not monitor.
Some keyboards are monitor.
a) Statements I: Some screen is monitor. No monitor is keyboard. All keyboards are mouse.
b) Statements II: All screens are monitor. No monitor is keyboard. Some keyboard is mouse
c) Statements III: No screen is monitor. No monitor is keyboard. All keyboards are mouse
d) Statements IV: All screens are monitor. All monitors are keyboard. No keyboard is mouse
e) Statements V: Some keyboards are mouse. All monitor is screen. No keyboard is a screen.
66. Conclusions:
Some walls are not brick.
Some cement is not water.
a) Statements I: Some brick are cement. No cement is wall. All walls are water.
b) Statements II: All bricks are cement. Some Cement is wall. All walls are water.
c) Statements III: No brick is cement. All cement is wall. No wall is water.
d) Statements IV: No brick is cement. No cement is wall. All walls are water.
e) Statements V: All walls are bricks. Some wall is cement. All waters are bricks.
67. Conclusions:
Some silver are platinum.
PAGE- 48
Some diamonds are silver.
a) Statements I: All silver are gold. Some gold are diamond. No diamond is platinum.
b) Statements II: Some silvers are gold. All gold are diamond. All diamond is platinum.
c) Statements III: All silvers are diamond. No gold is silver. Some diamond is platinum.
d) Statements IV: All silvers are gold. No gold is diamond. All diamond is platinum
e) Statements V: All platinum are silver. Some gold is silver. No gold is diamond.
68. Conclusions:
Some crows are not parrot.
Some crows are owl.
a) Statements I: All parrot are owl. No owl is crow. Some crows are Dove.
b) Statements II: Some parrots are owl. Some owls are crow. All crows are dove.
c) Statements III: No parrot is owl. All owls are crow. Some crows are dove.
d) Statements IV: All parrots are owl. Some owl is Dove. Some crows are dove.
e) Statements V: All owls are Parrot. All Parrots are crow. All crows are dove.
69. Conclusions:
Some white are not brown.
Some maroon is brown.
a) Statements I: All white are black. All black is brown. No brown is maroon.
b) Statements II: Some white are black. All black is brown. All brown is maroon.
c) Statements III: Some black are white. No black is brown. No brown is maroon.
d) Statements IV: All white is black. No black is brown. All brown is maroon
e) Statements V: No white is black. No black is brown. No brown is maroon
70. Conclusions:
All Donald being Micky is a possibility.
All Tom being Jerry is a possibility.
a) Statements I: All Donald is Jerry. All Jerry is Micky. No Micky is Tom.
b) Statements II: Some Donald is Jerry. No Jerry is Micky. Some Micky is Tom.
c) Statements III: Some Donald is Jerry. Some Jerry is Micky. No Micky is Tom.
d) Statements IV: All Donald is Jerry. No Jerry is Micky. All Micky is Tom.
e) Statements V: No Jerry is Tom. Some Donald is Tom. No Micky is Donald.
71. Conclusions:
Some Table is plastic.
Some plastic are bench
a) Statements I: All Table are Chair. All Chairs is bench. Some benches are plastic.
b) Statements II: All Table is Chair. Some Chair is bench. All benches are plastic.
c) Statements III: No Table is Chair. All Chairs are bench. Some benches are plastic.
d) Statements IV: Some Table is Chair. All Chairs are bench. No bench is plastic
e) Statements V: All Table is Chair. All Chairs are bench. All benches are plastic.
72. Conclusions:
Some dates are day.
Some years are day.
a) Statements I: All day are month. All month are year. Some years are date.
b) Statements II: Some days are month. All month are year. Some days are date.
c) Statements III: All day are month. Some month is year. All year are date.
d) Statements IV: All day are month. All month are date. Some years are date.
e) Statements V: No year is day. Some day is date. Some date is month.
PAGE- 49
73. Conclusions:
Some teachers are not student.
Some lessons are classroom.
a) Statements I: Some classroom is not student. All classrooms are teacher. All teachers are lesson.
b) Statements II: All classrooms are teachers. Some student is not classroom. Some teacher is lesson.
c) Statements III: All classrooms are student. No student is teacher. No teachers are lesson.
d) Statements IV: some classrooms are student. No student is teacher. Some teachers are lesson
e) Statements V: All students are classroom. All classrooms are lessons. All lessons are teachers.
74. Conclusions:
All windows being cot is a possibility.
Some doors are not pillow.
a) Statements I: Some pillows are window. All windows are door. No door is cot.
b) Statements II: All pillows are window. All windows are door. Some door is cot.
c) Statements III: All pillows are window. No window is door. Some door is cot.
d) Statements IV: Some pillows are window. Some window is door. All doors are cot.
e) Statements V: No window is door. Some door is pillow. Some pillow is cot.
DIRECTIONS Q. 75 to 77: In each question, a set of six statements is given, followed by five answer choices. Each of
the answer choices has a combination of three statements from the given set of six statements. You are required to
identify the answer choice in which the third statement is logically follows the first two in the same order.
75. Statements:
A. All red is green.
B. All red is white.
C. All red is black.
D. All black is white.
E. All green is yellow.
F. All green is white.
A. ABF
B. AEF
C. CDB
D. CBE
E. None of these
76. Statements:
A. All cows are goats.
B. All goats are dogs.
C. No goats are cows.
D. No goats are dogs.
E. All cows are dogs.
F. All dogs are cows.
A. FAB
B. ABE
C. AFB
D. ABF
E. None of these
PAGE- 50
CLOCK / CALENDAR
1: It is 4 hrs. 40 min in a clock. Find the angle between both the needles of a clock.
(A) 1000 (B) 800 (C) 1200 (D) 2000
2: What will be the angle formed between needles of a clock at 8 hrs.20 min.?
(A) 1300 (B) 1400 (C) 1200 (D) 700
3: It is 2:40 hours in a clock. What will be the angle between both needles of a clock?
(A) 1800 (B) 1600 (C) 3200 (D) None of these
4: It is 5:10 hours in a clock. What will be the angle between the hour needle & the minute needle of that clock?
(A) 2650 (B) 800 (C) 950 (D) 1050
5: At 3:40, the hour hand and the minute hand of a clock form an angle of:
(A) 125 (B) 1200 (C) 1350 (D) 1300
7: The angle between the minute hand and the hour hand of a clock when the time is 4:20, is:
(A) 100 (B) 00 (C) 200 (D) 50
8: At what time between 5 and 6 will the hands of a clock be at right angles?
7 5
(A) 4311min. past 5 (B) 4311min. past 5 (C) 45 min. past5 (D) 40 min. past 7
9: At what time between 7 and 8 o'clock will the hands of a clock be in the same straight line but, not together?
2 5 3
(A) 5 min.past 7 (B) 5 min. past 7 (C) 5 min. past 7. (D) 5 min. past 7
11 11 11
10: The mirror image of a clock shows 1:20 hours time. Find the actual time in the clock.
(A) 1:10 (B) 1:40 (C) 10:40 (D) 6.30
11: An accurate clock shows 8 o'clock in the morning. Through how may degrees will the hour hand rotate when the
clock shows 2 o'clock in the afternoon?
(A) 144° (B) 150° (C) 168° (D) 180°
12: The reflex angle between the hands of a clock at 10.25 is:
(A) 180° (B) 192.5 (C) 195° (D) 197.5
13: A clock is started at noon. By 10 minutes past 5, the hour hand has turned through:
(A) 145° (B) 150° (C) 155° (D) 160°
14: At what time between 7 and 8 o'clock will the hands of a clock be in the same straight line but, not together?
2 3 5
(A) 5 min. past 7 (B) 5 11 min. past 7 (C) 5 11 min. past 7 (D)5 11 min. past 7
15: At what time between 5 and 6 will the hour hands of a clock be at right angles?
5 7
(A) 4311min. past 5 (B) 4311 min. past 5 (C) 40 min. past 5 (D) 45 min. past 5
16: The angle between the minute hand and the hour hand of a clock when the time is 4:20, is:
(A)0° (B)10° (C)5° (D)20°
PAGE- 51
17: At 3:40, the hour hand and the minute hand of a clock form an angle of:
(A) 120° (B) 125° (C) 130° (D) 135°
18: How many times are the hands of a clock at right angle in a day?
(A) 22 (B) 24 (C) 44 (D) 48
19: The angle between the minute hand and the hour hand of a clock when the time is 8.30 is:
(A) 80° (B) 75° (C) 60° (D) 105°
20: How many times in a day, are the hands of a clock in straight line but opposite in direction?
(A) 20 (B) 22 (C) 24 (D) 48
21: At what time between 4 and 5 o'clock will the hands of a watch point in opposite directions?
6
(A) 45 min. past 4 (B) 40 min. past 4 (C) 50min. past 4 (D) 5411min. past 4
22: If the day before yesterday was Thursday, when will Sunday be?
(A) Tomorrow (B) Today (C) Two days after today (D) Day after tomorrow
23: If 3rd December, 2000 was Sunday, what day was 3rd January 2001?
(A) Friday (B) Thursday (C) Wednesday (D) Sunday
24: If 30th January 2003 was Thursday, what was the day on 2nd March, 2003?
(A) Sunday (B) Tuesday (C) Wednesday (D) Friday
25: The year next to 1990 will have the same calendar as that of the year 1990?
(A) 1995 (B) 1997 (C) 1996 (D) 2001
26: January 1, 2004 was a Thursday, what day of the week lies on Jan 2005?
(A) Monday (B Thursday (C) Saturday (D) Sunday
27: On 8th march, 2005, Wednesday falls what day of the week was it on 8th march, 2004?
(A) Monday (B) Tuesday (C) Wednesday (D) Friday
29: If the second day of a month is a Friday, which of the following would be the last day of the next month which
has 31 days?
(A) Monday (B) Thursday (C) Data inadequate (D) Sunday
30: If the 25th of August in a year is Thursday, the number of Mondays in that month is
(A) 3 (B) 4 (C) 5 (D) 6
31: If day after tomorrow is Saturday, what day was three days before yesterday?
(A) Wednesday (B) Tuesday (C) Sunday (D) Thursday
32: If the day before yesterday was Saturday, what day will fall on the day after tomorrow?
(A) Monday (B) Tuesday (C) Wednesday (D) Thursday
33: If 18th February, 2005 falls on Friday then what will be the day on 18th February, 2007?
(A) Monday (B) Wednesday (C) Friday (D) Sunday
PAGE- 52
34: If Tuesday falls on the fourth of the month, then which day will fall three days after the 24th?
(A) Monday (B) Thursday (C) Tuesday (D) Wednesday
35: It was Sunday on Jan 1, 2006. What was the day of the week Jan 1, 2010?
(A) Sunday (B) Saturday (C) Friday (D) Wednesday
36: What was the day of the week on 28th May, 2006?
(A) Thursday (B) Friday (C) Saturday (D) Sunday
37: What was the day of the week on 17th June, 1998?
(A) Monday (B) Tuesday (C) Wednesday (D) Thursday
38: What will be the day of the week 15th August, 2010?
(A) Thursday (B) Friday (C) Saturday (D) Sunday
40: On 8th Feb, 2005 it was Tuesday. What was the day of the week on 8th Feb, 2004?
(A)Tuesday (B) Monday (C) Sunday (D) Wednesday
PAGE- 53
DATA SUFFICIENCY
Direction (1-19 ) In each of the questions below consists of a question and two statements numbered I and II given
below it. You have to decide whether the data provided in the statements are sufficient to answer the question.
Read both the statements and Give answer.
(A) If the data in statement I alone are sufficient to answer the question, while the data in statement II alone are not
sufficient to answer the question
(B) If the data in statement II alone are sufficient to answer the question, while the data in statement I alone are not
sufficient to answer the question
(C) If the data either in statement I alone or in statement II alone are sufficient to answer the question
(D) If the data given in both statements I and II together are not sufficient to answer the question and
(E) If the data in both statements I and II together are necessary to answer the question.
3). Who among A, B, C, D and E each having a different age, is definitely the youngest?
I.B is younger than E but not the youngest
II.C is younger than only E and A
(1) A (2) B (3) C (4) D (5) E
4). Among 5 people M, N, O, P and Q having different heights, who is the tallest?
I.O is taller than only one person. Only 1 people is taller than only Q. M is not the Shortest.
II.O is shorter than 3 persons. Only one person is taller than Q. M is neither the taller nor the shortest in the group.N is
the shortest in the group.
(1) A (2) B (3) C (4) D (5) E
5). What is the number of girls in the second half of the row?
I. In a row of 36 students there are 8 boys and 28 girls.
II. In a row of 36 students the first boy is followed by girl, the second is followed by 2 girl
(1) A (2) B (3) C (4) D (5) E
10). How many pages of the Book did Divya read on Sunday ?
I. Divya read the last 50 pages of the book on Monday morning
II. The book has 300 pages out of which two-thirds were read by Divya before Sunday.
(1) A (2) B (3) C (4) D (5) E
11). A, B, C, D, E and F are sitting in a circular table facing the centre. How many persons are there between E and F (If
counted clockwise from F)
I.B and E sit adjacent to each other and B is on the immediate left of F.
II.A and C sit adjacent to each other. There is one person who sits between D and E. A is not an immediate neighbor
of E.
(1) A (2) B (3) C (4) D (5) E
12). Among the four friends A, B, C and D, who works for the most hours (each works for different no of hours?
I.A works less than B, D works less than both A and C
II.C works less than B but more than D, A works more than C but not the most.
(1) A (2) B (3) C (4) D (5) E
14). How many people are standing between R and U in a straight line of 15 people facing North?
I.T stands fifth from the left end of the line. R and O are immediate neighbors of T. Only two people stand between T
and U.
II.Q stands third from the right end of the line. Only 6 people stand between O and Q. Only one person stands between
R and O. O stands exactly between R and U
(1) A (2) B (3) C (4) D (5) E
16). Who among P, Q, T, V and M is exactly in the middle when they are arranged in ascending order of their heights?
I. V is taller than Q but shorter than M.
II. T is taller than Q and M but shorter than P.
(1) A (2) B (3) C (4) D (5) E
17). Which code word stands for ‘good’ in the coded sentence ‘sin co bye’ which means ‘He is good’ ?
I. In the same code language, ‘co mot det’ means ‘They are good’.
II. In the same code language, ‘sin mic bye’ means ‘He is honest’.
(1) A (2) B (3) C (4) D (5) E
18). Gaurav ranks eighteenth from the top in a class. What is his rank from the last?
I. There are 47 students in the class.
II. Jatin who ranks 10th in the same class, ranks 38th from the last.
(1) A (2) B (3) C (4) D (5) E
PAGE- 55
Directions (19-23): In each of the following problems, there is one question and three statements I, II and III given
below the question. You have to decide whether the data given in the statements is sufficient to answer the
question. Read all the statements carefully and find which of the statements is/are sufficient to answer the given
question. Choose the correct alternative in each question.
Directions (24-33): Each of the following questions below consists of a question and two statements numbered I and
II given below it. You have to decide whether the data provided in the statements are sufficient to answer the
question. Read both the statements and give the answer.
(A) if the data in statement I alone are sufficient to answer the question, while the data in statement II alone are not
sufficient in answer the question.
(B) if the data in statement II alone are sufficient to answer the question, while the data in statement I alone are not
sufficient to answer the question.
(C) if the data in either in statement I alone or in statement II alone are sufficient to answer the question.
(D) if the data in both the statements I and II together are not sufficient to answer the question.
(E) if the data in both the statements I and II are together necessary to answer the question.
28). Among A, B, C, D and E, each has scored different marks in an exam, who has scored the lowest marks?
I. D has scored more marks than only three of them.
II. A has scored more marks than the only E.
(1) A (2) B (3) C (4) D (5) E
29). What is the code for ‘time’ in the code language in which, ‘he made third centuries’ is written as ‘gift fat fit gat’?
I. In that code language ‘third time he pleased us’ is written as ‘kat kit mit git fit’.
II. In that code language ‘he made double centuries is written as ‘gat fat zat git’ and ‘give us time’ is written as ‘mat mit
kit’.
(1) A (2) B (3) C (4) D (5) E
30). What is the position of P with respect to Q in a round table in seating arrangement (If all of them facing towards
the center)?
I. Seven persons P, Q, R, S, T, U and V sat around a round table where P sat between U and V, S sat adjacent to V and R
did not sit adjacent to S.
II. T sat on the immediate right of U.
(1) A (2) B (3) C (4) D (5) E
PAGE- 58
STATEMENT ASSUMPTION
Directions (1 – 26): In each question below is given a statement followed by two assumptions numbered I and II. An
assumption is something supposed or taken for granted. You have to consider the statements and the following
assumption and decide which of the assumption(s) is / are implicit in the statement.
Give answer
a) If only Assumption I is implicit
b) If only Assumption II is implicit
c) If either Assumption I or II is implicit
d) If neither Assumption I nor II is implicit
e) If both Assumption I and II are implicit
1). Statement : The General Administration Department has issued a circular to all the employees informing them that
hence forth the employees can avail their lunch break at any of the half hour slots between 1 : 00 pm and 2 : 30pm.
Assumptions
I. The employees may welcome the decision and avail lunch break at different time slots.
II. There may not be any break in the work of the organization as the employees will have their lunch break at
different time slots.
2). Statement: The Government has decided against reduction of prices of petroleum products though there is
asignificant drop in the crude oil prices in the international market.
Assumptions
I. The prices of crude oil in the international market may again increase in the near future.
II. The present price difference of petroleum products will help the government to with stand any possible price rise
in future.
3). Statement : The Government has made an appeal to all the citizens to honestly pay income tax and file
returnsreflecting the true income level to help the Government to carry out development activities
Assumptions
I. People may now start paying more taxes in response to the appeal.
II. The total income tax collection may considerably increase in the near future.
4). Statement: The State Government has decided to appoint four thousand primary school teachers during the next
financial year.
Assumptions
I. There are enough schools in the state to accommodate four thousand additional primary school teachers.
II. The eligible candidates may not be interested to apply as the Government may not finally appoint such a large
number of primary school teachers.
5). Statement: The school authority has decided to increase the number of students in each classroom to seventy
from the next academic session to bridge the gap between income and expenditure to a larger extent.
Assumptions
I. The income generated by way of fees of the additional students will be sufficient enough to bridge the gap.
II. The school will get all the additional students in each class from the next academic session.
6). Statement : The driver of the huge truck pulled the emergency brakes to avoid hitting the auto rickshaw which
suddenly came in front of the truck.
Assumptions
I. The auto rickshaw driver may be able to steer his vehicle away from the oncoming truck.
II. The truck driver may be able to stop the truck before it hits the auto rickshaw.
7). Statement: The doctor warned the patient against any further consumption of alcohol, if he desired to get cured
PAGE- 59
from the ailment and live a longer life.
Assumptions
I. The patient may follow the doctor’s advice and stop consuming alcohol
II. The doctor may be able to cure the patient from the ailment, if the patient stops consuming alcohol
8). Statement: The chairman of the company urged all the employees to refrain from making long personal calls
during working hours in order to boost productivity.
Assumptions
I. Majority of the employees may respond positively to the chairman‟s appeal
II. Most of the employees may continue to make long personal calls during working hours
9). Statement: The local cultural club decided to organize a musical event to raise money for the construction of the
club building.
Assumptions
I. The local residents may not allow the club to organize the musical event in the locality.
II. The money allocated by organizing the musical event may be substantial enough for the club to start construction.
10). Statement : The traffic police department has put up huge notice boards at all major junctions of the city,
warning drivers to refrain from using cell phones while driving or else their licenses will be impounded
Assumptions
I. The drivers of the vehicles may ignore the warning and continue using cell phones while driving.
II. The traffic police department may be able to nab most of the offenders and impound their licenses.
11). Statement: “Repeat your recruitment ads on Sunday for just 60 per sq cm.” – An advertisement in a newspaper
Assumption:
I. People want ads at Zero cost
II. There are some people who want to repeat their recruitment ads.
12). Statement: “Quit drinking before it destroys you.” – A notice issued in public interest by XYZ department.
Assumptions:
I. People fear being destroyed
II. Drinking breaks the drinker’s family.
13).Statement: “The government’s decision to supply ARV drugs in six selected states free of cost is commendable,
but the patients should be adequately educated before the drugs are administrated.” – View of Mr.X‟
Assumptions:
I. The patients can develop drug resistance if they are not regular in their medication
II. The patients if not counselled properly may spread the drug – resistant disease.
14). Statements: “It is alarming to note that only 48 to 50 percent of the voters in State „X‟. The government must
make it mandatory for all to exercise their franchise in order to make the democracy more meaningful.” – View of a
voter
Assumption:
I. Increase in the participation of voters will ensure increase in the rationality of the democratic set – up.
II. Making it „Mandatory for all to exercise their franchise‟ may increase the number of voters.
15). Statements : The Food and Agriculture Organization (FAO) has declared 2004 the International year of Rice in an
effort to highlight the increasing demands for rice among growing populations and production constraints for the
commodity.
Assumptions:
I. There is an impending crisis of rice in several countries of the world
II. The year of Rice may act as a catalyst for country – driven programmes throughout the world.
PAGE- 60
16). Statement: “Policies and programmes and seminars cannot change the women’s status. There is a greater need
for attitudinal change in the society towards working women. “– View of Mr. X
Assumption:
I. It is possible to make change in the attitude of the society towards working women.
II. Status of the working women is not satisfactory.
17). Statement: “Government of State X‟s state, according to which it has been decided to provide cooked meal to
the students of all primary schools of the states, is not a proper and judicious step.” – Criticism by a person
Assumption:
I. Student may hesitate to eat the cooked meal provided by the Government.
II. The cooked meal served to the students may be hazardous for the health of students.
18). Statement : ”Although no war was witnessed during the tenure of prime minister ship of Mr. X, a large number of
top bravery medals were conferred upon many cops in the name of curbing terrorism and eliminating terrorists of
different organizations belonging to our own states.” – View of a citizen
Assumption:
I. It is disgraceful of cops to name the act of killing of terrorists of our own country as an act of bravery
II. A war is less harmful for a country than menace of terrorism.
19). Statement: “Completely eliminating the say of executive is not acceptable; merit, ability, competence, integrity
and suitability of the candidate alone are not enough for appointment of High Court Judges. “ – A journalist
Assumption:
I. A person‟s social outlook, concern for public interest and promotion of equality and his / her political outlook are
also equally important.
II. Executive consultation will ensure greater transparency of the appointment.
20). Statement: “The potential of knowledge as a creator of wealth is gaining currency all around the world. “ –View
of an economist
Assumption:
I. Only usable knowledge that is protectable may have the potential of wealth creation.
II. Only usable knowledge that is protected may have the potential of wealth creation.
21). Statement : 1‟s advice to 2‟s-If you want to study Marketing, join institute XZ”.
Assumptions:
I. 2 listens 1‟s advice.
II. Institute XZ provides good Marketing education.
23).Statements: “please drop this letter in the letter box”. An officer tells his assistant.
Assumptions: I. The assistant would follow the instructions.
II. The assistant knows the address where the letter is to be sent.
24).Statement: “Buy pure and natural milk of company Z”-An advertisement in a Newspaper.
Assumptions:
I. Artificial milk can be prepared.
II. People do not mind paying more for pure and natural milk.
27).Statement: we must be prepared to face any eventually and all the assignments must be completed as per their
schedule-Director told the faculty members.
Assumptions:
I. There is a possibility of any serious eventuality.
II. Dates are fixed for all the assignments.
III. Faculty members are supposed to complete all assignments.
a) All b) Only II and III c) None d) Only III e) Only I
28).Statement: Prakash decided to get the railway reservation in May for the journey he wants to make in July to
Madras.
Assumptions:
I. The railways issues reservations two months in advance
II. There are more than one train to madras.
III. There will be vacancy in the desired class.
a) Only II and III
b) Only I
c) All
d) Only I and III
e) None of these
29).Statement: “X- chocolate is deal as a gift for someone you love”- an advertisement.
Assumptions:
I. People generally gifts to loved ones.
II. Such advertisement generally influences people.
III. Chocolate can be considered as a gift item.
a) Only I and II
b) Only I and III
c) All
d) Only II and III
e) None of these
30).Statement: In view of the recent spurt in Sugar prices in the open market the government has asked the dealers
to release a vast quantity of imported sugar in the open market.
Assumptions:
I. The dealers will follow the government directive.
II. The sugar price will come down.
III. The price of indigenous sugar will remain unchanged.
a) None
b) Only I and II
c) Only I and III
d) All
e) Only II and III
PAGE- 62
STATEMENT CONCLUSION
Direction: In each question below is given a statement followed by two conclusions numbered I and II. You have to
assume everything in the statement to be true, then consider the two conclusions together and decide which of them
logically follows beyond a reasonable doubt from the information given in the statement.
Give answer:
(a) If only conclusion I follows
(b) If only conclusion II follows
(c) If either I or II follows
(d) If neither I nor II follows and
(e) If both I and II follow.
1. Statements: In a one day cricket match, the total runs made by a team were 200. Out of these 160 runs were made
by spinners.
Conclusions:
I. 80% of the team consists of spinners.
II. The opening batsmen were spinners.
3. Statements: Government has spoiled many top ranking financial institutions by appointing bureaucrats as Directors
of these institutions.
Conclusions:
I. Government should appoint Directors of the financial institutes taking into consideration the expertise of the
person in the area of finance.
II. The Director of the financial institute should have expertise commensurate with the financial work carried out by
the institute.
4. Statements: Population increase coupled with depleting resources is going to be the scenario of many developing
countries in days to come.
Conclusions:
I. The population of developing countries will not continue to increase in future.
II. It will be very difficult for the governments of developing countries to provide its people decent quality of life.
5. Statements: Prime age school-going children in urban India have now become avid as well as more regular viewers
of television, even in households without a TV. As a result there has been an alarming decline in the extent of
readership of newspapers.
Conclusions:
I. Method of increasing the readership of newspapers should be devised.
II. A team of experts should be sent to other countries to study the impact of TV. on the readership of newspapers.
6. Statements: In Japan, the incidence of stomach cancer is very high, while that of bowel cancer is very low. But
Japanese immigrate to Hawaii, this is reversed - the rate of bowel cancer increases but the rate of stomach cancer is
reduced in the next generation. All this is related to nutrition - the diets of Japanese in Hawaii are different than those
in Japan.
Conclusions:
I. The same diet as in Hawaii should be propagated in Japan also.
II. Bowel cancer is less severe than stomach cancer.
PAGE- 63
7. Statements: The Government run company had asked its employees to declare their income and assets but it has
been strongly resisted by employees union and no employee is going to declare his income.
Conclusions:
I. The employees of this company do not seem to have any additional undisclosed income besides their salary.
II. The employees union wants all senior officers to declare their income first.
8. Statements: Monitoring has become an integral part in the planning of social development programmes. It is
recommended that Management Information System be developed for all programmes. This is likely to give a
feedback on the performance of the functionaries and the efficacy with which services are being delivered.
Conclusions:
I. All the social development programmes should be evaluated.
II. There is a need to monitor the performance of workers.
9. Statements: The T.V. programmes, telecast specially for women are packed with a variety of recipes and household
hints. A major portion of magazines for women also contains the items mentioned above.
Conclusions:
I. Women are not interested in other things.
II. An average woman's primary interest lies in home and specially in the kitchen.
10. Statements: The distance of 900 km by road between Bombay and Jafra will be reduced to 280 km by sea. This will
lead to a saving of Rs. 7.92 crores per annum on fuel.
Conclusions:
I. Transportation by sea is cheaper than that by road.
II. Fuel must be saved to the greatest extent .
11. Statements: The manager humiliated Sachin in the presence of his colleagues.
Conclusions:
I. The manager did not like Sachin.
II. Sachin was not popular with his colleagues.
12. Statements: Women's organizations in India have welcomed the amendment of the Industrial Employment Rules
1946 to curb sexual harassment at the work place.
Conclusions:
I. Sexual harassment of women at work place is more prevalent in India as compared to other developed countries.
II. Many organizations in India will stop recruiting women to avoid such problems.
13. Statements: Nation X faced growing international opposition for its decision to explode eight nuclear weapons at
its test site.
Conclusions:
I. The citizens of the nation favored the decision.
II. Some powerful countries do not want other nations to become as powerful as they are.
14. Statements: In a highly centralized power structure, in which even senior cabinet ministers are prepared to reduce
themselves to pathetic countries or yes men airing views that are primarily intended to anticipate or reflect the Prime
Minister's own performances, there can be no place for any consensus that is quite different from real or contrived
unanimity of opinion, expressed through a well orchestrated endorsement of the leader's actions.
Conclusions:
I. The Ministers play safe by not giving anti-government views.
II. The Prime Minister does not encourage his colleagues to render their own views.
15. Statements: National Aluminium Company has moved India from a position of shortage to self-sufficiency in the
metal.
Conclusions:
PAGE- 64
I. Previously, India had to import aluminium.
II. With this speed, it can soon become a foreign exchange earner.
16. Statements: Reading makes a full man, conference a ready man and writing an exact man.
Conclusions:
I. Pointed and precise expression comes only through extensive writing.
II. Extensive reading makes a complete man.
17. Statements: Jade plant has thick leaves and it requires little water.
Conclusions:
I. All plants with thick leaves require little water.
II. Jade plants may be grown in places where water is not in abundance.
18. Statements: Use "Kraft" colours. They add colour to our life. - An advertisement.
Conclusions:
I. Catchy slogans do not attract people.
II. People like dark colours.
19. Statements: All those political prisoners were released on bail who had gone to jail for reasons other than political
dharnas. Bail was not granted to persons involved in murders.
Conclusions:
I. No political - prisoner had committed murder.
II. Some politicians were not arrested.
20. Statements: Modern man influences his destiny by the choice he makes unlike in the past.
Conclusions:
I. Earlier there were fewer options available to man.
II. There was no desire in the past to influence the destiny.
21. Statements: Water supply in wards A and B of the city will be affected by about 50% on Friday because repairing
work of the main lines is to be carried out.
Conclusions:
I. The residents in these wards should economize on water on Friday.
II. The residents in these wards should store some water on the previous day.
22. Statements: People who speak too much against dowry are those who had taken it themselves.
Conclusions:
I. It is easier said than done.
II. People have double standards.
23. Statements: The national norm is 100 beds per thousand populations but in this state, 150 beds per thousand are
available in the hospitals.
Conclusions:
I. Our national norm is appropriate.
II. The state's health system is taking adequate care in this regard.
24. Statements: Our securities investments carry market risk. Consult your investment advisor or agent before
investing.
Conclusions:
I. One should not invest in securities.
II. The investment advisor calculates the market risk with certainty.
27. Statements: The serious accident in which a person was run down by a car yesterday had again focused attention
on the most unsatisfactory state of roads.
Conclusions:
I. The accident that occurred was fatal.
II. Several accidents have so far taken place because of unsatisfactory state of roads.
28. Statements: In a recent survey report, it has been stated that those who undertake physical exercise for at least
half an hour a day are less prone to have any heart ailments.
Conclusions:
I. Moderate level of physical exercise is necessary for leading a healthy life.
II. All people who do desk-bound jobs definitely suffer from heart ailments.
30. Statements: This world is neither good nor evil; each man manufactures a world for himself.
Conclusions:
I. Some people find this world quite good.
II. Some people find this world quite bad.
31. Statements: The eligibility for admission to the course is minimum second class Master's degree. However, the
candidates who have appeared for the final year examination of Master's degree can also apply.
Conclusions:
I. All candidates who have yet to get their Master's degree will be there in the list of selected candidates.
II. All candidates having obtained second class Master's degree will be there in the list of selected candidates.
32. Statements: Any student who does not behave properly while in the school brings bad name to himself and also
for the school.
Conclusions:
I. Such student should be removed from the school.
II. Stricter discipline does not improve behaviour of the students.
33. Statements: A Corporate General Manager asked four managers to either submit their resignations by the next
day or face termination orders from service. Three of them had submitted their resignations by that evening.
Conclusions:
I. The next day, the remaining manager would also resign.
II.The General Manager would terminate his services the next day.
34. Statements: Only good singers are invited in the conference. No one without sweet voice is a good singer.
Conclusions:
I. All invited singers in the conference have sweet voice.
II. Those singers who do not have sweet voice are not invited in the conference.
PAGE- 66
35. Statements: To cultivate interest in reading, the school has made it compulsory from June this year for each
student to read two books per week and submit a weekly report on the books.
Conclusions:
I. Interest in reading can be created by force.
II. Some students will eventually develop interest in reading.
36. Statements: Applications of applicants who do no fulfil eligibility criteria and/or who do not submit applications
before last date will be summarily rejected and will not be called for the written test.
Conclusions:
I. Those who are called for the written test are those who fulfil eligibility criteria and have submitted their applications
before last date.
II. Written test will be held only after scrutiny of applications.
37. Statements: Recent trends also indicate that the number of child migrants in large cities is increasing. These
children leave their families to join the ranks of urban poor doing odd jobs in markets, workshops, hotels or in service
sectors.
Conclusions:
I. Migration to big cities should be checked.
II. The plight of poor children should be thoroughly studied.
39. Statements: The percentage of the national income shared by the top 10 per cent of households in India is 35.
Conclusions:
I. When an economy grows fast, concentration of wealth in certain pockets of population takes place.
II. The national income is unevenly distributed in India.
40. Statements: Players who break various records in a fair way get special prizes. Player X broke the world record but
was found to be under the influence of a prohibited drug.
Conclusions:
I. X will get the special prize.
II. X will not get the special prize.
PAGE- 67
COURSE OF ACTION
Direction: In each question below is given a statement followed by two courses of action numbered I and II. You have
to assume everything in the statement to be true and on the basis of the information given in the statement, decide
which of the suggested courses of action logically follow(s) for pursuing.
Give answer
(a) If only I follows
(b) If only II follows
(c) If either I or II follows
(d) If neither I nor II follows
(e) If both I and II follow.
1. Statement: A large number of people in ward X of the city are diagnosed to be suffering from a fatal malaria type.
I. The city municipal authority should take immediate steps to carry out extensive fumigation in ward X.
II. The people in the area should be advised to take steps to avoid mosquito bites.
2. Statement: Severe drought is reported to have set in several parts of the country.
I. Government should immediately make arrangement for providing financial assistance to those affected.
II. Food, water and fodder should immediately be sent to all these areas to save the people and cattle.
3. Statement: Since its launching in 1881, Vayudoot has so far accumulated losses amounting to Rs 153 crore.
I. Vayudoot should be directed to reduce wasteful expenditure and to increase passenger fare.
II. An amount of about Rs 300 crore should be provided to Vayudoot to make the airliner economically viable.
4. Statement: Exporters in the capital are alleging that commercial banks are violating a Reserve Bank of India
directive to operate a post shipment export credit denominated in foreign currency at international rates from
January this year.
I. The officers concerned in the commercial banks are to be suspended.
II. The RBI should be asked to stop giving such directives to commercial banks.
5. Statement: A large number of people die every year due to drinking polluted water during the summer.
I. The government should make adequate arrangements to provide safe drinking water to all its citizens.
II. The people should be educated about the dangers of drinking polluted water.
6. Statement: Footpaths of a busy road are crowded with vendors selling cheap items.
I. The help of police should be sought to drive them away.
II. Some space should be provided to them where they can earn their bread without blocking footpaths.
7. Statement: Some serious blunders were detected in the Accounts section of a factory.
I. An efficient team of auditors should be appointed to check the Accounts.
II. A show cause notice should be issued to all the employees involved in the irregularity.
8. Statement: Researchers are feeling agitated as libraries are not equipped to provide the right information to the
right users at the right time in the required format. Even the users are riot aware about the various services available
for them.
I. All the information available to the libraries should be computerized to provide faster services to the users.
II. Library staff should be trained in computer operations.
9. Statement: Many medical and engineering graduates are taking up jobs in administrative services and in banks.
I. All the professionals should be advised to refrain from taking up such jobs.
II. The government should appoint a committee to find out the reasons for these professionals taking up such jobs and
to suggest remedial measures.
PAGE- 68
10. Statement: Due to substantial reduction in fares by different airline services, large number of passengers so far
travelling by upper classes in trains have switched over to airline services.
I. The railways should immediately reduce the fare structure of the upper classes substantially to retain its
passengers.
II. The railways should reduce the capacity of upper classes in all the trains to avoid loss.
11. Statement: Next day, millions of pilgrims are expected to take dip the Ganga at the holy place.
I. The state government should put a restriction on the number of people that can come to take dip each day.
II. During the next whole day, the state government should deploy personnel to maintain law and order.
12. Statement: During the workers laying cables underground in ABC locality, a main pipe supplying water to the
houses burst and the locality was flooded.
I. The concerned authority should immediately take an action and start repairing the damage.
II. The concerned authority should take an action against the company whose workers were laying the cables and ask
them for the compensation.
13. Statement: Some college students were found guilty of travelling in a local train without tickets.
I. The parents of these students should be contacted and asked to counsel their wards.
II. The students found guilty in such cases should be held behind bars.
14. Statement: Though private ferry systems have made going to holy places easy, today many pilgrims died in a
stampede while boarding a private ferry.
I. The state government should cancel the licenses of all such private ferry operators.
II. The state government should deploy some people to guide the pilgrims on their journey to the holy place.
15. Statement: Many of the people invited for a marriage at ABC Hotel fell ill due to food poisoning and were
admitted to the hospitals.
I. The government should keep on hold marriages till further notice.
II. The hospitals should be advised to give good services to the effected people.
16. Statement: In the coastal area, a huge wave swept away the homes of fishermen and others living along the
coastline.
I. The people should not be allowed to build their homes along coastline.
II. The administration should send its officials to find the extent of damages taken place and suggest remedial
measures.
17. Statement: Some workers of ABC diamond company were held by the police, due to found smuggling small pieces
of diamonds when they were leaving the company after company hours.
I. The company should shut its business for some time until full proof security systems are installed in the company
premises.
II. Before the workers leave the company, they should all be checked from now onwards.
18. Statement: A huge truck and a car met with an accident and this incident has blocked a major part of the main
road leading to a huge traffic jam.
I. The traffic department should take an action by immediately deploying its people to divert traffic on other roads.
II. The traffic department should take an action by immediately sending its people and equipment to move the truck
and clear the jam.
19. Statement: During a fight between two student groups of a college last night, many vehicles were damaged by
throwing stones on each other.
I. The local police should arrest the ones found guilty of doing these acts.
II. The local police should call the college authority to bring the situation under control.
PAGE- 69
20. Statement: The inflation rate in increasing since the last 6 months and there is no sign of it coming down in next 3-
4 months.
I. Government should make an effort by reducing government taxes on essential commodities with immediate effect.
II. Government should ask the farmers to sell their crops at low prices.
21. Statement: The sale of a particular product has gone down considerably causing great concern to the company.
I. The company should make a proper study of rival products in the market.
II. The price of the product should be reduced and quality improved.
22. Statement: The Asian Development Bank has approved a $285 million loan to finance a project to construct coal
ports by Paradip and Madras Port Trusts.
I. India should use financial assistance from other international financial organizations to develop such ports in other
places.
II. India should not seek such financial assistance from the international financial agencies.
23. Statement: Doordarshan is concerned about the quality of its programmes particularly in view of stiff competition
it is facing from STAR and other satellite TV channels and is contemplating various measures to attract talent for its
programmes.
I. In an effort to attract talent, the Doordarshan has decided to revise its fee structure for the artists.
II. The fee structure should not be revised until other electronic media also revise it.
24. Statement: The Minister said that the teachers are still not familiarised with the need, importance and meaning
of population education in the higher education system. They are not even clearly aware about their role and
responsibilities in the population education programme.
I. Population education programme should be included in the college curriculum.
II. Orientation programme should be conducted for teachers on population education.
25. Statement: A recent study shows that children below five die in the cities of the developing countries mainly from
diarrhea and parasitic intestinal worms.
I. Governments of the developing countries should take adequate measures to improve the hygienic conditions in the
cities.
II. Children below five years in the cities of the developing countries need to be kept under periodic medical check-up.
26. Statement: The kharif crops have been affected by the insects for consecutive three years in the district and the
farmers harvested less than fifty percent of produce during these years.
I. The farmers should seek measures to control the attack of insects to protect their crops next year.
II.The Government should increase the support price of kharif crops considerably to protect the economic interests of
farmers.
27. Statement: The car dealer found that there was a tremendous response for the new XYZ's car-booking with long
queues of people complaining about the duration of business hours and arrangements.
I. People should make their arrangement of lunch and snacks while going for car XYZ's booking and be ready to spend
several hours.
II. Arrangement should be made for more booking desks and increased business hours to serve more people in less
time.
28. Statement: The State Government has decided to declare 'Kala Azar' as a notifiable disease under the Epidemics
Act. Family members or neighbours of the patient are liable to be punished in case they did not inform the State
authorities.
I. Efforts should be made to effectively implement the Act.
II. The cases of punishment should be propagated through mass media so that more people become aware of the
stern actions.
PAGE- 70
29.Statement: The Chairman stressed the need for making education system more flexible and regretted that the
curriculum has not been revised in keeping with the pace of the changes taking place.
31.Statement: The Indian electronic component industry venturing into the West European markets faces tough
competition from the Japanese.
I. India should search for other international markets for its products.
II. India should improve the quality of the electronic components to compete with the Japanese in capturing these
markets.
32.Statement: Orissa and Andhra Pradesh have agreed in principle to set up a joint control board for better control,
management and productivity of several inter-state multipurpose projects.
I. Other neighboring states should set up such control boards.
II. The proposed control board should not be allowed to function as such joint boards are always ineffective.
33. Statement: The Government has decided not to provide financial support to voluntary organizations from next
Five Year Plan and has communicated that all such organizations should raise funds to meet their financial needs.
I. Voluntary organizations should collaborate with foreign agencies.
II. They should explore other sources of financial support.
34.Statement: The availability of imported fruits has increased in the indigenous market and so the demand for
indigenous fruits has been decreased.
I. To help the indigenous producers of fruits, the Government should impose high import duty on these fruits, even if
these are not of good quality.
II. The fruit vendors should stop selling imported fruits. So that the demand for indigenous fruits would be increased.
35.Statement: There has been an unprecedented increase in the number of successful candidates in this year's School
Leaving Certificate Examination.
I. The government should make arrangements to increase number of seats of intermediate courses in existing
colleges.
II. The government should take active steps to open new colleges to accommodate all these successful candidates.
36.Statement: On an average, about twenty people are run over by trains and die every day while crossing the railway
tracks through the level crossing.
I. The railway authorities should be instructed to close all the level crossings.
II. Those who are found crossing the tracks, when the gates are closed, should be fined heavily.
37.Statement: Majority of the students in many schools do not pass in the final examination.
I. These schools should be closed down as these have become unproductive.
II. The teachers of these schools should immediately be retrenched.
38.Statement:In spite of the Principal's repeated warnings, a child was caught exploding crackers secretly in the
school.
I. All the crackers should be taken away from the child and he should be threatened not to do it again.
II. The child should be severely punished for his wrong act.
PAGE- 71
ARGUMENT
Directions: Each question given below is followed by two arguments numbered I and II. You have to decide which of
the argument a ‘strong’ argument is and which a ‘weak’ argument is.
Give answer:
(a) If only argument I is strong,
(b) if only argument II is strong,
(c) If either I or II is strong
(d) if neither I nor II is strong and
(e) if both I and II are strong.
1. Statement: Should one close relative of a retiring government employee be given a job in government in India?
Arguments: I. Yes, where else will the relatives get a job like this?
II. No, it will close doors of government service to competent and needy youth.
2. Statement: Should purchase of gold by individuals be restricted in India to improve its foreign exchange positions?
Arguments: I. Yes, interference on customer's right and freedom is desirable.
II. No, business interest has to be guarded first.
4. Statement: Should all education be made free for girls and women of all ages of India?
Arguments: I. No, this will weaken our present social structures.
II. Yes, this is the only way to bring back glory to Indian womanhood.
5. Statement: Should private colleges offering professional courses like Engineering Medical, Management be banned
in India?
Arguments: I. Yes, such courses should be run by Government Colleges only
II. Yes, no other country allows private colleges to run professional courses.
7. Statement: Should adult education programme be given priority over compulsory education programmes?
Arguments: I. No, It will also help in success of compulsory education programme.
II. Yes, it will help to eliminate the adult illiteracy.
10. Statement: Should higher education be completely stopped for some time?
Arguments: I. No, It will hamper the country’s future progress.
II. Yes, It will reduce the educated unemployment.
PAGE- 72
11. Statement: Should slum dweller be provided free houses in big cities and metropolises?
Arguments: I. No, most of the slum dwellers are poor and illiterate.
II. Yes, providing food and shelter to every citizen is the responsibility of any welfare state.
13. Statement: Should military service for short duration be made compulsory to all eligible youth in India?
Arguments: I. yes, Indian defense forces are badly in need of jawans and officers.
II. Yes, it will inculcate discipline and national pride in youth.
17. Statement: Should non- vegetarian food be totally banned in our country?
Arguments: I. Yes. It is expensive and therefore it is beyond the means of most people in our country.
II. No. Nothing should be banned is a democratic country like ours.
18. Statement: Should workers be allowed to participate in the management of factories in India?
Arguments: I. Yes. It is the present management theory.
II. No. Many workers are illiterate and so their contributions will not be of any value.
20. Statement: Should those who receive dowry, despite the law prohibiting it, be punished?
Arguments: I. Yes. Those who violate the law must be punished.
II. No. Dowry system is firmly rooted in the society since time immemorial.
21. Statement: Should selection tests be of the objective rather than of the descriptive type?
Arguments: I. Yes. The assessment of answers to objective type questions is fair and impartial.
II. No. The descriptive type test is certainly a better tool than the objective type test.
27. Statement: Should loyalty be the only criterion for promotion in any organization?
Arguments: I. Yes. With out loyal men, no organization can function.
II. No. It leads to hypocrisy and partiality.
31. Statement: Should election expenses to central and state legislatures be met by the Government?
Arguments: I. Yes. It will put an end to political corruption.
II. No. It is not good in any country.
33. Statement: Should all the remote parts of a country be connected by road?
Arguments: I. No. It will disturb peaceful simple life of the villagers.
II. Yes. It must be done immediately.
35. Statement: Should India encourage exports, when most things are insufficient for internal use itself.
Arguments: I. Yes. We have to earn foreign exchange to pay for our imports.
II. No. Even selective encouragement would lead to shortages.
37. Statement: Should retirement age for the employees be reduced to 56 years?
PAGE- 74
Arguments: I. yes. It will provide employment opportunity to many unemployed.
II. Yes. It will provide good output because average age of employee is reduced.
38. Statement: Should India sign comprehensive Test Ban Treaty (CTBT)?
Arguments: I. No. India will not be able to protect her border if it does so.
II. Yes. This is the only way to reduce tension in the Asian sub-continent.
39. Statement: Should Hindi language we made compulsory at school level in all the states in India?
Arguments: I. No. Students should not be forced to learn any language.
II. Yes. It is the national language, so everybody must learn.
PAGE- 75
ADJECTIVES
1. (A) Anurag is eclipsed by his wife, / (B) who is cleverer and / (C) amusing than he is. /(D) No error
2. (A) The two first t o arrive / (B) were the lucky recipients / (C) of a surprise gift. /(D) No error
3. (A) Of the billions of stars in the galaxy/ (B) how much are/ (C) suitable for life? / (D) No error
4. (A) A few word of / (B) gratitude are enough / (C) to express your / (D) feelings sincerely. /(e) No error.
5. (A) He feels his troubles / (B) as much or / (C) even more than they. / (D) No error
6. (A) I like reading / (B) more than / (C) to play games. / (D) No error
7. (A) There is not many traffic / (B) along the street / (C) where I live. / (D) No error
8. (A) The police arrived and discovered / (B) a large number of hoarded sugar/ (C) in his shop. / (D) No error
9. (A) As a dramatist / (B) Shaw is superior than / (C) any other twentieth century writer. / (D) No error
10. (A) Of the two great cities/ (B) the former is / (C) biggest. / (D) No error
11. (A) A non banking financial company is a / (B) financial institution similarly to a bank / (C) but it cannot issue
cheque books to customers. / (D) No error
12. (A) Everyone agrees that / (B) the Ganga is the holiest / (C) of all other rivers of India. /(D) No error.
13. (A) Krishna ran to the / (B) nearing grocery store to / (C) buy biscuits as his parents were expecting guests. / (D) No
error
14. (A) While giving a loan/ (B) you must check / (C) if the borrower has sufficiently collateral to repay it. / (D) No error
15. (A) Although his speech / (B) was not very clearly everyone understood / (C) the underlying meaning. / (D) No
error
16. (A) He is the most / (B) intelligent and also / (C) the very talented student of the college. / (D) No error
17. (A) Having been found / (B) guilty of the theft / (C) Sunny was sentenced to five year’s imprisonment. / (D) No
error
18. (A) The Railways have made / (B) crossing the tracks / (C) a punished offence. / (D) No error
19. (A) Alms / (B) are given / (C) to the poors. / (D) No error
20. (A) Lasers are / (B) indispensable tools / (C) for the delicate eyes surgery. / (D) No error
21. (A) The number of applications has risen / (B) this year by / (C) as many as 50%. / (D) No error
22. (A) Two lakh of people / (B) attended the meeting / (C) held in parade ground. / (D) No error
23. (A) These sort of men / (B) attain success by hook or by crook / (C) so they do not deserve any applause. (D) No
error
24. (A) Can you tell me how / (B) many eggs and / (C) milk he has brought / (D) No error
25. (A) Whole the chapter / (B) is full of printing errors which are the outcome / (C) of the proof reader’s carelessness.
(D) No error
26. (A) Her black long / (B) hair adds / (C) glamour to her looks. (D) No error
27. (A) I trembled when I saw / (B) a sharp long knife / (C) in his hand. / (D) No error
28. (A) I saw / (B) an anxious pale girl / (C) who was fidgetting near the ICU / (D) No error
29. (A) Mumbai is / (B) further from / (C) Delhi than / (D) Patna. (e) No error
30. (A) This book is / (B) undoubtedly preferable than / (C) that and its printing / (D) is also comparatively good.(e) No
error
31. (A) You can trust / (B) this channel/ (C) for the last news of this hour. (D) No error
32. (A) Everybody knows / (B) that Radha is the most unique / (C) singer of this college / (D) No error
33. (A) The faster he completes / (B) the work given to / (C) him, the largest will be his profit. / (D) No error.
34. (A) He does not have / (B) some money to buy a new machine so he is / (C) anxious / (D) No error
35. (A) This young lady is / (B) more beautiful but not so cultured / (C) as her sister. (D) No error
36. (A) Now-a-days, / (B) the weather / (C) is getting cold and colder. / (D) No error
37. (A) This photograph / (B) was comparatively better / (C) than that which he had kept in his purse. / (D) No error
38. (A) Ram is as good, / (B) if not better than / (C) they./ (D) No error
39. (A) Gopal felt happily / (B) to learn that I / (C) had got a job in the Bank./ (D) No error
40. (A) He is comparatively / (B) weaker / (C) in maths / (D) no error.
PAGE- 76
ADVERBS
PAGE- 77
TIME AND TENSE
1.When you will try to (A)/ understand the rules of the game (B)/you will become able (C)/ to play properly.(D)/No
error (E)
2. Whenever (A)/ he is coming here (B)/ he brings a lot of (C)/ problems for me.(D)/ No Error (E)
3. It is appearing to me (A)/ that you are complaining (B)/ against my (C)/ friends and relatives. (D)/ No Error (E)
4. The old man who is sitting on the bench (A)/ lived here (B)/ for more than (C)/ a month (D)/ No Error (E)
5. A misogynist is a person (A)/ who hates woman (B)/ but a philogynist is a person (C)/ who loves woman. (D)/ No
Error (E)
6. The secret of his sound (A)/ health lies in the fact that (B)/ he is getting up before sunrise(C)/ and has fresh air.(D)/
No Error(E)
7. As soon as he will (A)/ reach home, he will (B)/ send you the books (C)/ you are in need of.(D)
8. He seems to be happy (A)/ because his family (B)/ had come from village (C)/ to live with him. (D)/ No Error (E)
9. “It is high time (A)/ you are reading (B)/ this work (C)/” my friend said to me. (D)/ No Error(E)
10. I have got (A)/ the information that your (B)/ father has died (C)/ last week. (D)/No Error(E)
11. Many of my friends (A)/ told to come here (B)/ today but (C)/ none of them arrived yet. (D)/No Error (E)
12. Ruchika says (A)/ that she will take (B)/ her umbrella (C)/ in case it will rain. (D)/ No Error(E)
13. I will let you know (A)/ as soon as I will get (B)/ any information (C)/ about your job. (D)/ No Error(E)
14. When I will be thirty (A)/ most of my friends (B)/ will be above thirty (C)/ five, be sure. (D)/No Error (E)
15. This is, indeed, (A)/ the first time in my life that (B)/ I hear(C)/ such an interesting story(D)./ No Error(E)
16. Before the alarm (A) had stopped ringing (B) Veena had pulled up (C) the shade. (D) No Error(E)
17. I have been knowing (A)/ him for ten years (B) /but I don’t know where he lives.(C)/No Error(D)
18. He uses to study (A)/ at least till ten o’clock (B) /at night and then (C) /he goes to bed. (D)/ No Error(E)
19. If he had (A)/ time to spare (B) /he would spend (C)/ them with his family. (D)/ No Error(E)
20. All the flowers (A)/ in the park will die (B)/ before the rains(C)/will fall. (D)/No Error(E)
21. If he would have done this, (A) /he would done wrong (B)/ and would have disappointed(C) /many of his friends.
(D)/ No Error(E)
22. He reached (A)/the station after(B)/ the train had(C)/ started.(D)/ No error (E)
23. I wish (A)/ he saw you (B)/when you were (C)/ living in Delhi.(D)/ No Error(E)
24. The judge had not given (A)/ any decision (B)/ until he had studied the case.(C) /No Error (D)
25. He lived (A)/ here since 1980 (B)/, therefore he knows everything(C)/ his whereabouts. (D)/. No Error(E)
26. I have been (A)/ working for this (B)/ institution for (C)/ the last five years.(D)/ No Error (E)
27. If we had Ram (A)/ in our team, we (B)/ would have won the match(C) /against Bangladesh.(D)/ No Error(E)
28. My secretary came (A) to the meeting (B) later (C) than I expect. (D) No Error
29. If I was you (A)/ I would have told (B)/ the Director to keep (C)/ his mouth shut.(D)/ No Error(E)
30. I want you to (A)/ pick up the box (B)/ of clothes and(C) /kept it in hall .(D)/No Error(E)
31. My friend tried to tell us (A)/ what has happened with him (B) /but his words (C) /were not clear to us. (D)/No
Error(E)
32. My wife got (A)/ everything ready for all (B)/ of them long (C)/ before they arrived. (D) /No Error(E)
33. She was with (A) /me uptil now, (B)/ so please don’t scold her(C)/ for being late. (D)/No Error
34. I have been adoring (A)/ her for (B) /the beauty of her eyes.(C)/No Error (D)
35. Had he come(A)/ even a moment earlier(B) /he would have(C)/ found me there. (D)/ No Error(E)
36. Many observations indicate (A)/ that the number of the drug(B)/ addicts is grew(C)/ day by day. (D)/ No Error
37. Newspapers told (A)/ us about the event (B)/ that happen (C)/all over the world. (D)/ No Error(E)
38. “Here is coming (A)/ my friends!” (B)/ said Manju when (C)/she was anxious(D)/ no error (E)
39. He advised me (A)/ to do as he did (B)/ but I did not pay any attention(C)/ to his advice. (D) /No Error(E)
40. By this time next year he has had (A)/ settled himself (B)/ in London .(C)/ No Error(D)
PAGE- 78
CLOZE TEST
Directions (1-10): In the following passage there are blanks, each of which has been numbered. These numbers are
printed below the passage and against each, five words are suggested, one of which fits the blank appropriately.
Find out the appropriate word in each case.
Visual experiences can (1)children, teenagers and even adults learn and absorb more due to its highly stimulating
and (2)engaging impact. It is for this reason that we are seeing an increase in schools across the globe (3) content
provider programmes into their class curriculum to (4) lessons through video. Visual excursions and school
collaborations are (5) by advances in high definition video, high fidelity audio and content sharing allowing students to
experience a richer and more stimulating learning experience. Schools that have previously transported students to
excursions in (6), now face increased transportation costs, higher insurance premiums, attendance costs for the
families and strict duty of care policies for students while (7) school property/ Virtual excursions (8)students to
improve their presentation, research, learning and speaking skills while they engage in a live learning session.
Students also now have the ability to meet peers from many cultures, speak to subject-matter (9) like scientists or
authors practise a foreign language with students from another country, and learn about global issues from the (10)of
their own classrooms.
1. (a) help (b) aiding (c) prescribe (d) feature (e) present
2. (a) plus (b) lonely (c) ably (d) many (e) deeply
3. (a) incorporating (b) pressing (c) following (d) parting (e) leaving
4. (a) make (b) demand (c) impart (d) vision (e) need
5. (a) dissolved (b) enhanced (c) measured (d) failed (e) blasted
6. (a) deed (b) total (c) parent (d) person (e) lieu
7. (a) involving (b) saving (c) away (d) off (e) vacating
8. (a) let (b) enable (c) present (d) pressure (e) collect
9. (a) clauses (b) dictionaries (c) books (d) experts (e) partners
10. (a) vacancy (b) availability (c) safety (d) comfortable (e) gap
Directions (11-20): In the given passage there are blanks, each of which has been numbered. Against each five
words are suggested, one of which fits the blank appropriately. Find out the appropriate word in each case.
If China’s state owned commercial banks seem burdened by bad debts, the Country’s rural financial sector is even
worse. In the villages, the only formal banking institutions are what are known as rural credit co-operatives.
These (11)the distinction in China of having been officially declared insolvent. The rural credit co-operatives are ill
named. They are often reluctant to (12) and they are not run as co-operatives as they do not (13) any profits and their
customers have no say in their operations. Until 1996,they were offshoots of the Agricultural Bank of China. Since
then they have been (14) by the Central Bank, though they are in reality run by country government. Even the word
‘rural’ is misleading. (15)of their deposits are sucked up and put in the urban banking system. Farmers usually find it
easier to (16) from friends or relatives or black market moneylenders. Yet the co-operatives remain a big part of
China’s financial system. Last year, they (17) for 12 percent of deposits and 11 percent of loans. In recent years,
commercial banks (including the Agricultural bank) have closed down (18) in the countryside. Yet some 40,000 credit
co-operatives remain in place with one in almost every township as the larger villages or smaller rural towns are (19).
If as the government claims, the credit co-operatives are beginning to turn a profit after six years of losses, it is not
because they are any better run. In an effort to (20) a stagnant rural economy, the central bank has pumped more
than $9 billion into them hoping that they will lend more to farmers. But the root causes of their problems remain
and the real solution may have to involve a mix of approaches from commercial banking to real cooperatives.
11. (a) awarded (b) enjoy (c) worry (d) making (e) trouble
12. (a) sanctions (b) apply (c) part (d) provide (e) giving
13. (a) function (b) eligible (c) claims (d) declared (e) share
14. (a) own (b) govern (c) regulations (d) ran (e) supervised
15. (a) Such (b) Partly (c) Whole (d) Most (e) Entire
PAGE- 79
16. (a) visit (b) help (c) borrow (d) loan (e) advice
17. (a) include (b) accounted (c) fulfilled (d) achieved (e) taking
18. (a) branches (b) all (c) operating (d) staff (e) factory
19. (a) thinking (b)known (c) creating (d) cross (e) develop
20. (a) make (b)release (c) boosting (d)stall (e) revitalize
Directions (21-25): In the following passage there are blanks, each of which has been numbered. Against each, five
words are suggested, one of which fits the blank appropriately. Find out the appropriate word in each case.
Primary school enrolment in India has been a success story,(21)due to various programmes and drives to increase
enrolment even in remote areas. With enrolment reaching at least 96 percent since 2009, and girls (22) up 56 percent
of new students between 2007 and 2013, it is clear that many (23) of access to schooling have been (24).
Improvement in infrastructure has been the (25) behind achieving this and now in India 98 percent habitations have a
primary school within one kilometre and 92 percent have an upper primary school within a three kilometre walking
distance.
21. (a) most (b) properly (c) totally (d) optionally (e) largely
22. (a) coming (b) reaching (c) counting (d) making (e) touching
23. (a) issue (b)opportunities (c) problems (d) efforts (e) exertions
24. (a) accustomed (b) addressed (c) met (d) forwarded (e) dissolved
25. (a) main (b)forced (c)force (d)compulsion (e) awareness
Directions (25-30): In the following passage there are blanks, each of which has been numbered. Against each, five
words are suggested one of which fits the blank appropriately. Find out the appropriate word in each case.
Education has been a problem in our country for (26). The lack of it has been blamed for all (27) of evil for hundreds of
years. Even scholars have written lengthy articles about how the Indian education system needs to change. The funny
thing is that fro colonial times, things have (28) changed. We have established reputed business schools, law schools
and other institutions of excellence. Students, now, so routinely score 90% marks that even with this percentage they
find it (29) to get into the colleges of their choice. The problem thus lies with us doing more of the same old staff. This
needs to change by bringing about (30) in education.
26. (a) time (b) take (c) ever (d) long (e) decade
27. (a) possession (b) abundance (c)typical (d) much (e) sorts
28. (a) bare (b) hardly (c) little (d) much (e) highly
29. (a) simple (b) easy (c) irregular (d) noble (e) difficult
30. (a) innovation (b) dreams (c)creating (d) foreign (e) choice
Directions (30-35): In the following passage there are blanks, each of which has been numbered. Against each, four
words are suggested one of which fits the blank appropriately. Find out the appropriate word in each case.
Today most businessmen are very worried. To begin with, they are not used to competition.In the past they sold
whatever ...(31)... produced at whatever prices they chose. But ...(32)... increasing competition, customers began to
...(33)... and choose. Imports suddenly became ...(34)... available and that too at cheaper ...(35)...
Directions (35-40): In the following passage there are blanks, each of which has been numbered. Against each, four
words are suggested one of which fits the blank appropriately. Find out the appropriate word in each case.
PAGE- 80
As a rule of thumb, a manned mission costs from fifty to a hundred times more than a comparable unmanned mission.
Thus, for scientific exploration alone, ...(1)... missions, employing machine intelligence, are ...(2)... However, there may
well be ...(3)... other than scientific for exploring ...(4)... social, economic, political, cultural or ...(5)...
PAGE- 81
SUBJECT VERB AGREEMENT
1. (A) Neither of them/ (B) are going to attend/ (C) the party on 10th October./ (D) No error
2. (A) He walked five miles which are really a great distance/ (B) for a man like him who is not only old but also ill./ (D) No
3. (A) Either my colleague/ (B) or a peon are coming home/ (C) with the material today./ (D) No error
4. (A) The rise and fall/ (B) of the tide are due/ (C) to lunar influence./ (D) No error
5. (A) Many a man/ (B) have succumbed/ (C) to this temptation./ (D) No error
6. (A) The introduction of tea, coffee/ (B) and such other beverages/ (C) have not been without some effect./ (D) No error
7. (A) The newer type of automatic machines/ (B) wash/ (C) clothes faster./ (D) No error
8. (A) Each of the students in the computer class/ (B) has to type/ (C) their own research paper this semester./ (D) No error
9. (A) Everyone of the films/ (B) you suggested/ (C) are not worth seeing./ (D) No error
10. (A) The Secretary and Principal of the college/ (B) are attending/ (C) the District Development Council Meeting at
the Collectorate./ (D) No error
11. (A) There is/ (B) only one of his novels/ (C) that are interesting./ (D) No error
12. (A) Knowledge of/ (B) at least two languages/ (C) are required to pass the examination./ (D) No error
13. (A) It is I/ (B) who is to blame/ (C) for this bad situation./ (D) No error
14. (A) Romanticism of melancholy/ (B) in art and literature are the reason/ (C) for insensitivity to those suffering from
depression./ (D) No error
15. (A) Patience as well as perseverance (B) are necessary/ (C) for success./ (D) No error
16. (A) In Singapore/ (B) my brother-in-law with his wife/ (C) were present at the function./ (D) No error
17. (A) A hot and/ (B) a cold spring/ (C) was found near each other./ (D) No error
18. (A) Either of the roads/ (B) lead/ (C) to the park/ (D) No error
19. (A) One of my desires/ (B) are to become/ (C) a doctor./ (D) No error
20. (A) The whole block of flats/ (B) including two shops were/ (C) destroyed in fire./ (D) No error
21. (A) The sum and substance/ (B) of his poem/ (C) are as follows./ (D) No error
22. (A) Neither of the/ (B) five accused/ (C) could be convicted./ (D) No error
23. (A) The strain of all/ (B) the difficulties, vexations and anxieties/ (C) were more than he could beat./(D) No error
24. (A) Everybody/ (B) it must be admitted/ (C) has their ups and downs./ (D) No error
25. (A) Every woman in the world/ (B) fervently hopes that their child/ (C) will be a normal and healthy
baby/ (D) No error
26. (A) Neither of them/ (B) sent their papers/ (C) in time for the last seminar/ (D) No error
27. (A) This is a strange world/ (B) where each one pursues their own golden bubble/ (C) and laughs at others for doing the
same./ (D) No error
28. (A) If it were possible to get near when/ (B) one of the volcanic eruptions take place/ (C) we would see a grand sight./
(D) No error
29. (A) A rise in rents and wages/ (B) have been found/ (C) to go together./ (D) No error
30. (A) He is one of those few post-colonial writer who believes/ (B) that this talk about colonialism has gone too far/ (C) and
has turned into a cliché./ (D) No error
31. (A) One of the peculiarities/ (B) which distinguishes the present age/ (C) is the multiplication of books/ (D) No error
32. (A) Neither of them/ (B) are going to attend/ (C) the party on 10th October/ (D) No error
33. (A) Ten miles are/ (B) a long distance / (C) to cover on foot./ (D) No error
34. (A) If Mahatma Gandhi/ (B) was alive, he would feel sorry for the poor and downtrodden who/(C) still struggle everyday
to make both ends meet. / (D) No Error
35. (A) Having acquired some experience/ (B) she is no longer one of those who believes/ (C) every explanation they are
given./ (D) No error
36. (A) With regard to implementation of the/ (B) details of the proposal, the committee was divided/ (C) in their opinion./
(D) No error
37. (A) Most of the funds/ (B) we get from/ (C) America is used to build roads and bridges./ (D) No error
38. (A) The tiger was not/ (B) the only dangerous animal/ (C) in the forest there was hyenas too./ (D) No Error
39. (A) She immediately quit/ (B) the job in which/ (C) neither skill nor knowledge were required./ (D) No Error
40. (A) The type of qualities you acquire/ (B) depend upon your company/ (C) and so you associate yourselves with simple
and good natured people./(D) No error
PAGE- 82
CRITICAL REASONING
Directions: Each question has a small passage followed by a question. Choose the best answer from the five options
given after each question.
ASSUMPTION
1. The local university recently hired a new soccer coach. Although she has several years' worth of coaching
experience and is a diligent student of the game, she was never a member of a collegiate soccer team. For this reason,
the new coach will be unable to build a successful program.
The argument above is based on which of the following assumptions?
(A) The local university should have hired a former collegiate soccer player as its new coach.
(B) Coaching experience is one of the most crucial factors for coaching success.
(C) The previous coach at the university was dismissed due to her lack of success,
(D) To build a successful soccer program as a coach, one must be a former collegiate soccer player.
(E) The university does not plan to provide the new coach with the resources necessary to build a successful program.
2. Methyl tetrachloride (MTC) chemical found in some pesticides, glues and sealants. Exposure to MTC can cause
people to develop asthma. In order to halve the nation's asthma rate, the government has announced that it will ban
all products containing MTC.
The government's plan to halve the nation's asthma rate relies on which of the following Assumptions?
(A) Exposure to MTC is responsible for no less than half of the nation's asthma cases.
(B) Products containing MTC are not necessary to the prosperity of the American economy.
(C) Asthma has reached epidemic proportions.
(D) Exercise and proper nutrition are helpful in maintaining respiratory health.
(E) Dust mites and pet dander cause asthma.
3. Traffic Safety Expert: In order to achieve a substantial reduction in the extremely high number of car accidents in
our country, we should implement a radical new safe driving plan. Seat belts and air bags would be eliminated from
the driver's side of all automobiles to provide the driver with the strongest possible incentive to drive safely. Further,
an electroshock system would administer shocks to the driver if he or she exceeds the speed limit or engages In other
unsafe driving practices.
In declaring that the radical new safe driving plan will help to reduce accident rates, the author assumes which of
the following?
(A) Many car accidents are caused, at least partially, by naturally occurring conditions such as rain and fog.
(B) Accidents in which one or both participants exceed 60 miles per hour account for a majority of all fatal car
accidents.
(C) A significant number of accidents are the result of negligence or other unsafe driving practices on the part of the
driver.
(D) To alleviate safety concerns, citizens should be allowed to reinstall their drivers' side seat belts and air bags at their
own expense.
(E) If not implemented properly, electroshock systems can cause heart attacks and other health problems.
4. Two genes, BRCA1and BRCA2, are linked to hereditary breast cancer. Therefore, in order to decrease the annual
number of mammogram tests administered across a population and to more accurately assess woman's individual risk
of breast cancer, all women should be tested for these genes. -
PAGE- 83
basis than they used to.
(B) The majority of breast cancer patients have no family history of the disease. '
(C) Researchers may have identified a third breast cancer gene that is linked with hereditary breast cancer.
(D) Women who have these genes have an 80 percent chance of getting breast cancer, while women who don't have
these genes have only a 10 percent chance of getting breast cancer.
(E)The presence of BRCA1and BRCA2 can explain up to 50 percent of hereditary cases.
5. Editorial: The Intercontinental Bank should reallocate the voting shares of its members in order to more effectively
shape global economic policy. For example, China comprises about 15 percent of the world's gross domestic product
but has only a 3 percent voting share, whereas Belgium, with less than 1 percent of the global economy, has a 2
percent share.
Which of the following is an assumption upon which the editorial's authors depend in suggesting a way to more
effectively shape global economic policy?
(A) The United States has a larger voting share of the Intercontinental Bank than does China or Belgium.
(B) The specific allocation of voting shares factors into the Intercontinental Bank's effectiveness in shaping global
economic policy.
(C) Only voting shares that are precisely proportional to each country's contribution to the global economy are
appropriate for the Intercontinental Bank.
(D) The Intercontinental Bank is necessary to the maintenance of a prosperous global economy.
(E)As one of the fastest growing economies, China should have a larger voting share in the Intercontinental Bank.
6. Advertisement: According to a recent research study, daily use of Fresh Start, a new toothpaste, reduces the risk of
developing dental cavities by over 20 percent. In addition, as a result of a new formula, the use of Fresh Start results in
whiter, healthier-looking teeth. Clearly, Fresh Start not only gives your teeth a beautiful look but also provides the
most reliable protection against dental cavities.
7. Student Advisor: One of our exchange students faced multiple arguments with her parents over the course of the
past year. Not surprisingly, her grade point average (GPA) over the same period showed a steep decline. This is just
one example of a general truth: problematic family relationships can cause significant academic difficulties for our
students.
Which of the following is an assumption underlying the general truism claimed by the Student Advisor?
(A) Last year, the exchange student reduced the amount of time spent on academic work, resulting in a lower GPA.
(B) The decline in the GPA of the exchange student was not the reason for the student's arguments with her parents.
(C) School GPA is an accurate measure of a student's intellectual ability.
(D) If proper measures are not taken, the decline in the student's academic performance may become irreversible.
(E) Fluctuations in academic performance are typical for many students,
8. Country B's oil production is not sufficient to meet its domestic demand in order to sharply reduce its dependence
on foreign sources of oil, Country B recently embarked on a program requiring all of its automobiles to run on ethanol
in addition to gasoline. Combined with its oil production, Country B produces enough ethanol from agricultural by-
products to meet its current demand for energy.
PAGE- 84
Which of the following must be assumed in order to conclude that Country B will succeed in its plan to reduce its
dependence on foreign oil?
(A) Electric power is not a superior alternative to ethanol in supplementing automobile gasoline consumption.
(B) ln Country B domestic production of ethanol is increasing more quickly than domestic oil production.
(C) Ethanol is suitable for the heating of homes and other applications aside from automobiles.
(D) In Country B, gasoline consumption is not increasing at a substantially higher rate than domestic oil and ethanol
production.
(E) Ethanol is as efficient as gasoline in terms of mileage per gallon when used as fuel for automobiles.
9. An industry analyst asserted in his recent report that the relative scarcity of housing in a particular market leads to
larger than normal increases in price. During the late 1990s, according to the analyst's report, occupancy rates---a
measure of the percentage of housing occupied at a given time-in crowded urban markets such as New York and San
Francisco hovered around 99.5%. During the same period, housing prices increased by as much as 100% per year,
compared to more normal past increases in the range of 5%to 15% per year.
10. Studio executives carefully examine how a film performs on its opening weekend in order to determine whether-
and how-to invest more in that film. Many decisions, such as increasing the number of screens that show the film and
expanding the marketing campaign, are best made after reactions can be gathered from audiences who actually
purchased tickets. Therefore, to maximize returns on their marketing investments, studios should initially release all
their films on a small number of screens and with a limited advertising campaign.
The plan to maximize returns by initially releasing films on only a small number of screens and limiting advertising
depends upon which of the following assumptions?
(A) Large marketing investments made before the opening weekend never eventually yield greater profits than
small initial marketing investments.
(B) New advertising techniques, such as Web-based viral marketing, have not substantially reduced the average
marketing costs for films.
(C) A film's prior performance in noncommercial settings, such as festivals, is not well correlated with how the
general public tends to react to that film.
(D) Across the movie industry, marketing investments do not influence the eventual financial returns of films in
predictable ways.
(E) How a film performs during its opening weekend is a strong indicator of the film's financial performance over its
lifetime.
CONCLUSION
11. Both enrollment and total tuition revenue at Brownsville University have increased during each of the last four
years. During the same period of time, enrollment at Canterbury University has steadily decreased, while total tuition
revenue has remained constant.
12. A detailed lab analysis of a meteorite recently discovered in Antarctica revealed that the meteorite has geological
characteristics common to the planet Mars. To date, scientists have not found these characteristics anywhere other
than on Mars. Using a technique called "acid-etching," scientists found that the meteorite contained fossilized remains
of single-cell life forms.
The statements above, if true, best support which of the following as a conclusion?
(A) The fossilized remains indicate that life exists on Mars.
(B) The scientists have evidence to support a hypothesis that the meteorite came from Mars.
(C) The meteorite represents a substantial step forward in human knowledge of the development of life in the solar
system.
(D) Undiscovered meteorites currently exist in Antarctica.
(E) "Acid-etching" is necessary to confirm the existence of fossilized remains in meteorites.
13. The average fuel efficiency of vehicles sold nationwide during the period 2000-2004 was 25 miles per gallon; the
corresponding figure during the period 1995-1999 was 20 miles per gallon. The national average price of gasoline
during the period 2000-2004 was $2 per gallon; the corresponding figure during the period 1995-1999 was $1.60 per
gallon.
The statements above, if true, best support which of the following conclusions?
(A) The average fuel efficiency of vehicles sold nationwide should reach 30 miles per gallon for the period 2005 - 2009.
(B) The national average price of gasoline during 1997 was lower than the corresponding price during 2003.
(C) Rising gasoline prices lead consumers to purchase more fuel-efficient cars.
(D) Between the two described time periods, the national average fuel efficiency and the national average gasoline
price both increased at roughly the same rate.
(E) Consumers spent more money on gasoline during the period 2000-2004 than during the period 1995-1999.
14. Museum A will henceforth display only undamaged objects of proven authenticity. Doubts have been raised about
the origins of a supposedly Mycenaean vase currently on display in the museum's antiquities wing. The only way to
establish this vase's authenticity would be to pulverize it, then subject the dust to spectroscopic analysis.
The claims above, if true, most strongly support which of the following conclusions?
(A) Authentic Mycenaean vases are valuable and rare.
(B) Museum A has been beset with questions about the provenance of many of the items in its antiquities wing.
(C) The vase in question will no longer be displayed in Museum A.
(D) Spectroscopic analysis has revolutionized the forensic investigation of art forgery.
(E) Knowingly or not, many of the world's museums display some forgeries.
15. Of all the high schools in the United States, Judd Academy is the one with the largest number of alumni serving in
the Air Force. Knox worth High School, however, is the school with the most graduates serving in the military as a
whole, including the Army, Navy, Air Force, Marines, and Coast Guard.
Which of the following, if true, is most clearly supported by the statements above?
(A) Knox worth is the most patriotic town in the nation.
(B) Judd Academy has fewer graduates serving in the Navy than does Knox worth High School.
(C) Judd Academy has a higher percentage of alumni serving in the Air Force than does Knox worth High School.
(D) Some graduates of Knox worth High School are serving in the military but not in the Air Force.
(E) Knox worth High School is the school with the second-highest number of alumni serving in the Air Force.
PAGE- 86
STRENGTHEN THE CONCLUSION
16. Brand X designs and builds custom sneakers, one sneaker at a time. It recently announced plans to sell "The Gold
Standard," a sneaker that will cost five times more to manufacture than any other sneaker that has been ever been
created.
Which of the following, if true, most supports the prediction that The Gold Standard shoe line will be profitable?
(A) Because of its reputation as an original and exclusive sneaker, The Gold Standard will be favored by urban
hipsters willing to pay exceptionally high prices in order to standout.
(B) Of the last four new sneakers that Brand X has released, three have sold at a rate that was higher than projected.
(C) A rival brand recently declared bankruptcy and ceased manufacturing shoes. '
(D) The market for The Gold Standard will not be more limited than the market for other Brand X shoes.
(E) The Gold Standard is made using canvas that is more than five times the cost of the canvas used in most sneakers.
17. The Farmsley Center for the Performing Arts, designed by a world-renowned architect, was built ten years ago in
downtown Metropolis. A recent study shows that, on average, a person who attends a performance at the Farmsley
Center spends eighty-three dollars at downtown businesses on the day of the performance. Citing this report, the
chairman of the Farmsley Center's Board of Trustees contends that the Farmsley Center has been a significant source
of the economic revitalization of downtown Metropolis.
Which of the following, if true, most strongly supports the chairman's contention?
(A) The Metropolis Chamber of Commerce honored the Farmsley chairman this year for his contributions to the city.
(B) Restaurants near the Farmsley Center tend to be more expensive than restaurants in outlying areas.
(C) The Farmsley Center is the only building in Metropolis designed by a world-renowned contemporary architect.
(D) For major theater companies on national tours, the Farmsley Center is the first choice among venues in downtown
Metropolis.
(E) Many suburbanites visit downtown Metropolis on weekends primarily in order to see performances at the
Farmsley Center.
18. John was flying from San Francisco to New York with a connecting flight in Chicago on the same airline. Chicago's
airport is one of the largest in the world, consisting of several small stand-alone terminals connected by trams. John's
plane arrived on time. John was positive he would make his connecting flight thirty minutes later, because _
19. Advertising Executive: More than 10 million American households now own digital video recorders which can fast-
forward over television commercials; approximately 75% of these households fast-forward over at least one
commercial per 30-minute program. Television commercials are now much less cost-effective, as they are not as
widely watched as they used to be.
Which of the following, if true, strengthens the claim that television commercials are less cost-effective than they
used to be?
(A) Product placement within television programs is a viable alternative to traditional television commercials.
(B) The television programs preferred by consumers without digital video recorders are similar to those preferred
by consumers with the devices.
PAGE- 87
(C) Prior to the advent of digital video recorders, very few television viewers switched channels or left the room
when commercials began
(D) The cost-effectiveness of television advertising is based less upon how many people watch a particular
commercial and more upon the appropriateness of the demographic.
(E) Due to an imperfect sampling system used to measure the number of viewers, many companies find it difficult
to determine the return on investment for television commercials.
20. Company Management: The most recent advertising campaign for our leading brand of low calorie soft drinks,
Sunrise Splash, has obviously been a success. Since this campaign was conducted in several magazines a year ago, our
unit sales of Sunrise Splash have increased by 10%, reaching a record level in our corporate history. In addition,
consumer surveys indicate that the proportion of customers who recognize this brand has nearly doubled over this
period.
Which of the following statements would most strongly support the claim made about the campaign's success?
(A) Over the past year, the price of Sunrise Splash has been reduced by nearly 20%.
(B) Over the past year, unit sales of Sunrise Splash have increased by nearly 1.5 million bottles.
(C) As a result of a shift in consumer preferences towards low-calorie soft drinks, the consumption of these drinks
has grown at a double-digit rate over the past several years.
(D) The majority of new sales of Sunrise Splash made over the past year Involved one of the coupons distributed
during the last advertising campaign.
(E) Over the past year the company has experienced a dramatic increase in sales of many other soft drinks.
WEAKEN THE CONCLUSION
21. The united States is considering a ban on the importation of salmon from Country B in order to protest poor
protection of intellectual property rights in Country B. An economist counters that such a ban would be ineffective,
since Country B would circumvent. it by selling the extra salmon in Europe. Indeed, last year, six European nations
each imported more salmon than Country B exported to the United States.
Which of the following, if true, would most severely weaken the economist's argument?
(A) Salmon is the chief export of Country B, accounting for a substantial proportion of its export earnings over
each of the last three years.
(B) The supply of native salmon has become increasingly limited in certain parts of North America in the past
decade, including many parts of the United States.
(C) Salmon from Country B is considered a delicacy in all of the European nations that imported salmon last year.
(D) The economic value to U.S. companies of the adoption of intellectual property regulations in Country B is
greater than the value of salmon exports from Country B.
(E) Costs for the transportation of salmon from Country B to Europe would make salmon from Country B more
expensive for European consumers than salmon imported from other countries.
22. At Hospital A, there has been an upsurge in emergency room visits by children under twelve with stomach
disturbances. Patient tracking revealed that the vast majority of the children had eaten candy labeled "Coconut Fun
Snax" shortly before the onset of symptoms. The emergency room physicians concluded that it is unsafe for children
under the age of twelve to eat products containing coconut.
Which of the following, if true, would be the best reason to doubt the warning?
(A) Coconuts contain saturated fats.
(B) Some pathogens are not detectable by any medical tests.
(C) No coconuts or coconut products are used in the manufacture of "Coconut Fun Snax."
(D) The patient tracking at the hospital contacted the parents of all of the children concerned and received full
cooperation from them.
(E) Coconuts are a favorite food of many children.
23. The Smithtown Theatre, a town theatre that stages old Broadway shows, has announced a new expansion that will
PAGE- 88
substantially increase both the capacity and the costs of operating the theatre. Attendance at the Smithtown Theatre
is currently just enough for the theatre to cover its present operating costs. In addition, all of the current patrons of
the theatre live in Smithtown, and the population of the town is not expected to increase in the next several years.
Thus, it seems certain that the expansion of the Smithtown Theatre will prove unprofitable.
Which of the following, if true, would most seriously weaken the argument?
(A) A large movie chain plans to open a new multiplex location in Smithtown later this year.
(B) Concession sales in the Smithtown Theatre comprise a substantial proportion of the theatre's revenues.
(C) Many recent arrivals to Smithtown are students that are less likely to attend the Smithtown Theatre than are
older residents.
(D) The expansion would allow the Smithtown Theatre to stage larger, more popular shows that will attract
patrons from neighboring towns. .
(E) The Board of the Smithtown Theatre responsible for choosing which shows to stage regularly solicits input
from residents of the town.
24. The respective owners of a book store and a coffee shop that are next door to one another have decided to
combine their businesses. Both owners believe that this merger will help increase the number of customers, and
therefore the gross revenue. They reason that customers who come for a cup of coffee might find themselves glancing
at the book titles for sale, and those who come for books might like to sit down and start reading with a cup of coffee.
Which of the following, if true, most weakens the owners' conclusion that a merger will increase revenue?
(A) Books and drinks can both be considered impulse purchases; often, they are purchased by customers without
forethought.
(B) Because of the way in which the two stores are currently positioned relative to one another, many customers
who come to the coffee shop never see or notice the book store.
(C) If books are damaged in a bookstore before purchase, the customers responsible for the damage are
generally not held financially accountable.
(D) The coffee shop is primarily frequented by local high school students whose parents make up a large
percentage of the book store's customer base; the teenagers use the coffee shop as a place to mingle with
their friends without family involvement.
(E) A combination book store and coffee shop that opened in a neighboring city last year has already earned
higher than expected profits.
25. A recent article asserted that, from 2002 to 2006, Band 1 generated the most revenue of any band in the world.
The article based this claim on the fact that in each of those years, Band 1 sold 'the most albums, and Band 1's highly
anticipated first concert tour was the highest grossing concert tour in the industry in 2006.
MINOR QUESTIONS
26. Nitrogen triiodide is a highly explosive chemical that is easy to make from only two ingredients: ammonia and
concentrated iodine. However, no terrorists are known to have ever used nitrogen triiodide in an attack.
Which of the following, if true, is the most likely explanation for the discrepancy described above?
(A) Ammonia can be bought in a grocery store, but concentrated iodine must be obtained from somewhat more
restricted sources, such as chemical supply houses.
(B) Nitrogen triiodide is only one of several powerful explosives that can be made from ammonia.
PAGE- 89
(C) Many terrorists have been more focused on acquiring weapons of mass destruction, such as nuclear or
biological weapons, than on developing conventional. chemical explosives..
(D) Airport security devices are typically calibrated to detect nitrogen compounds, such as ammonia and
ammonium compounds.
(E) Nitrogen triiodide is extremely shock sensitive and can detonate as a result of even slight movement.
27. With information readily available on the Internet, consumers now often enter the. Automobile retail environment
with certain models and detailed specifications in mind. In response to this trend, Car Store has decided to move
toward a less aggressive sales approach. Despite the fact that members of its sales pers9"nel have an average of ten
years of experience each.
Car Store has implemented a mandatory training program for all sales personnel, because
(A) the sales personnel in CarStore have historically specialized in aggressively selling automobiles and add-on
features
(B) the sales personnel in CarStore do not themselves use the Internet often for their own purposes
(C) CarStore has found that most consumers do not mind negotiating over price
(D) information found on the Internet often does not reflect sales promotions at individual retail locations
(E) several retailers that compete directly with CarStore have adopted ."customer-centered" sales approaches
28. Editorial: In order to preserve the health of its local economy, Metropolis should not permit a
Costmart warehouse department store to open within city limits. It has been demonstrated
that when Costmart opens a warehouse department store within a city, the bankruptcy rate of
local retailers increases in that city by twenty percent over the next several years.
Which of the following questions would be most useful for evaluating the conclusion of the editorial?
(A) Does the bankruptcy rate of local retailers in a city generally stabilize several years after a Costmart
warehouse department store opens?
(B) Do most residents of Metropolis currently do almost all of their shopping at stores wlthin the city limits of
Metropolis?
(C) Have other cities that have permitted Costmart warehouse department stores within city limits experienced
any economic benefits as a result?
(D) Is the bankruptcy rate for local retailers in Metropolis higher than in the average city that has permitted a
Costmart warehouse department store within city limits?
(E) Does Costmart plan to hire employees exclusively from within Metropolis for the proposed warehouse department
store?
29. Scientists recently documented that influenza spreads around the world more efficiently in the modern era due to
commercial air travel. Symptoms of a pandemic-level flu are severe enough that the ill would likely cancel or
reschedule air travel, but an infected person can travel across the globe before the first signs appear. Further, if
symptoms develop while someone is still on a plane, the infected person's cough can spread the virus easily in the
enclosed and closely packed environment.
Which of the following would best minimize the role air travel can play in the spread of influenza during a
pandemic?
(A) installing air filtration systems in the planes to kill any flu virus particles flowing through the filters
(B) requiring air travelers to receive flu vaccinations far enough in advance of the trip to provide protection
against the disease
(C) refusing to allow children, the elderly, or others who are especially vulnerable to flu to travel by air during a
pandemic
(D) requiring all air travelers to wash their hands before boarding a plane
(E) conducting medical examinations during the boarding process to weed out passengers with flu symptoms
30. In an attempt to explain the cause of malaria, a deadly infectious disease common in tropical areas, early
European settlers in Hong Kong attributed the malady to poisonous gases supposedly emanating from low-lying
PAGE- 90
swampland. Malaria, in fact, translates from the Italian as "bad air." In the 1880s, however, doctors determined that
Anopheles mosquitoes were responsible for transmitting the disease to humans after observing that the female of the
species can carry a parasitic protozoan that is passed on to unsuspecting humans when a mosquito
feasts on a person's blood.
What function does the statement in boldface fulfill with respect to the argument presented above? .
(A) It provides support for the explanation of a particular phenomenon.
(B) It presents evidence which contradicts an established fact
(C) It offers confirmation of a contested assumption;
(D) It identifies the cause of an erroneous conclusion.
(E) It proposes a new conclusion in place of an earlier conjecture.
31. Due to the increase in traffic accidents caused by deer in the state, the governor last year reintroduced
a longer deer hunting season to encourage recreational hunting of the animals. The governor expected the longer
hunting season to decrease the number of deer and therefore decrease. the number of accidents. However, this year
the number of accidents caused by deer has increased substantially since the reintroduction of the longer deer
hunting season.
Which of the following, if true, would best explain the increase in traffic accidents caused by deer?
(A) Many recreational hunters hunt only once or twice per hunting season, regardless of the length of the season.
(B) The deer in the state have become accustomed to living in close proximity to humans and are often easy prey
for hunters as a result.
(C) Most automobile accidents involving deer result from cars swerving to avoid deer, and leave the deer in question
unharmed.
(D) The number of drivers in the state has been gradually increasing over the past several years.
(E) A heavily used new highways recently built directly through the ,state’s largest forest, which is the primary
habitat of the state's deer population.
32. The law of demand states that, if all other factors remain equal, the higher the price of a good, the less people will
consume that good. In other words, the higher the price, the lower the quantity demanded.
33. In past decades, private equity investors used to compete for exclusive participation in investments. Now, in
response to both the growing scale of investments and increased competition to participate, private equity funds
often form syndicates or "dubs" and jointly take positions in large investments. Clearly the reason investors do this is
to allow them to spread some of the risk and also gain access to a broader range of investments and opportunities.
Which of the following statements by a private equity investor best defines the changing attitude described in the
argument above? .
(A) "We would rather beat our competition by working with them in order to find out their strengths and
weaknesses,"
(B) "In order to keep up with our competition, we should stop investing small and only invest in very large
opportunities,"
(C) "In order to make sure that we can participate in certain investments, we should expect to cooperate with our
competition on occasion,"
PAGE- 91
(D) "To avoid taking any risks, it is necessary to stop competing with our former competitors,"
(E) "In response to changing market conditions, we should participate only in investments that allow us to take
better positions than our competitors,"
34. Media Critic: Network executives have alleged that television viewership is decreasing due to the availability of
television programs on other platforms, such as the internet, video-on demand, and mobile devices. These executives
claim that declining viewership will cause advertising revenue to fall so far that networks will be. unable to spend the
large sums necessary to produce programs of the quality now available. That development, in turn, will lead to a
dearth of programming for the very devices which cannibalized television's audience.
However, technology executives point to research which indicates that users of these platforms Increase the number
of hours per week that they watch television because they are exposed to new programs and promotional spots
through these alternate platforms. This analysis demonstrates that networks can actually increase their revenue
through higher advertising rates, due to larger audiences lured to television through other media.
The portions in boldface play which of the following roles in the media critic's argument?
(A) The first is an inevitable trend that weighs against the critic's claim; the second is that claim.
(B) The first is a prediction that is challenged by the argument; the second is a finding upon which the argument
depends.
(C) The first clarifies the reasoning behind the critic's claim; the second demonstrates why that claim is flawed.
(D) The first acknowledges a position that the technology executives accept as true; the second is a consequence
of that position.
(E) The first opposes the critic's claim through an analogy; the second outlines a scenario in which that claim will not
hold.
35. As a percentage of the total population in the United States, the foreign-born population increased from 4.7
percent in 1970 to 11.1 percent in 2000. However, given historical immigration patterns, this trend is unlikely to
continue in the 21st century.
Which of the following is most like the argument above in its logical structure?
(A) The birth rate in Town T increased dramatically between 1970 and 2000. However, between 2000 and 2005,
the birth rate decreased slightly.
(B) The gray wolf population in Minnesota grew nearly 50 percent between 1996 and 2005. However, during the
same time period, the gray wolf population in Montana only increased by around 13 percent.
(C) Company A's sales have decreased over the past two quarters. However, as sales typically increase during the
fourth quarter, Company A predicts that sales will not continue to go down.
(D) Per capita soft drink consumption in the United States has increased by nearly 500% over the past 50 years. In
order to combat the affiliated health risks, some soft drink manufacturers are developing carbonated milk
drinks to be sold in schools.
(E) The number of televisions sold in Country Q decreased by 20% between 2005 and 2006. However, the average
number of hours spent watching television in Country Q more than doubled.
36. Diamond Enterprises is a store in Apisville that sells specialty luxury items. For several years, Diamond reaped
substantial profits and was considering building branch stores in nearby counties. Stibium Industries, for several
years the single largest employer in Apisville and the surrounding region, abruptly closed its plant last year, causing
widespread unemployment. Only a fraction of the former Stibium workers have found new jobs, and many of these at
much lower wages. Early this year, Diamond Enterprises has filed for Chapter 11 bankruptcy, citing the closure of
Stibium as one of the primary causes.
37. Public Health Official: After several years of vaccinating all of the citizens of this state for Tacitus’ Disease, a highly
infectious virus, state hospitals have cut costs by no longer administering this vaccine, starting at the beginning of this
year. A state senator defended the position, arguing that after several years with zero incidence of the disease in the
state, its citizens were no longer at risk. This is a flawed argument. Our state imports meats and produce from
countries with high incidences of diseases for which our country has vaccines. Three years ago, when we reduced the
use of the Salicetiococcus vaccines, a small outbreak of Salicetiococcus among young children, fortunately without
fatalities, encouraged us to resume use of the previous vaccines.
The public health official’s statements, if true, best support which of the following as a conclusion?
(A) Young children of the state will be at risk for Tacitus’ Disease.
(B) Some of the meats imported to this state do not have adequate refrigeration during the shipping process.
(C) Tacitus’ Disease is a much deadlier disease than Salicetiococcus, and has a correspondingly higher fatality rate.
(D) No food products produced within the state bear any contaminants that could lead to either Tacitus’ Disease
or Salicetiococcus.
(E) The cost of providing all citizens of the state with the Tacitus’ Disease vaccine places an undue burden on the
budget of state health agencies.
38. Randall: Many of the productions of my plays by amateur theater groups are poorly done, and such
interpretations do not provide a true measure of my skills as a dramatist.
Which one of the following can be properly inferred from Randall’s statement?
(A) Some amateur theater groups’ productions of Randall’s plays provide a true measure of his skills as a dramatist.
(B) All amateur theater group productions of Randall’s plays that are not poorly done provide a true measure of
his skills as a dramatist.
(C) All of the productions of Randall’s plays by amateur theater groups that do not provide a true measure of his
skills as a dramatist are poorly done.
(D) If a production of a dramatist’s play is well done, then it provides a true measure of his or her skills as a dramatist.
(E) At least some amateur theatrical groups’ productions of Randall’s plays fail to provide a true measure of his
skills as a dramatist.
39. XYZ Corporation has two divisions, both of which performed consistently over the last five years. The Interment
Services Division accounted for approximately 30% of the corporation’s transactions and 50% of the corporation’s
profits; the Toxic Household Products Division accounts for the balance.
The statements above support which of the following inferences about XYZ Corporation over the last five years?
(A) Measured in dollars, the total profits for XYZ Corporation have remained stable over the last five years.
(B) Interment Services is an increasingly competitive field, while Toxic Household Products are a largely untapped
market.
(C) The Toxic Household Products Division yields a lower average profit per transaction than does the Interment
Services Division.
(D) XYZ Corporation’s Toxic Household Products line has remained consistent over the past five years.
(E) Most families will, over a given five-year period, spend more money on Interment Services than on Toxic
Household Products.
PAGE- 93
READING COMPREHENSION
Directions (1- 10): Read the following passage carefully and answer the questions given below them. Certain
words/phrases have been printed in bold to help you locate them while answering some of the questions.
Last year the idea took hold that Mark Zuckerberg might run for president in 2020 and seek to lead the world’s most
powerful country. Today, Facebook’s founder is fighting to show that he is capable of leading the World’s eighth-
biggest listed company or that any of its 2.1bn users should trust it. News that Cambridge Analytica (CA), a firm linked
to President Donald Trump’s 2016 campaign, got data on 50m Facebook users in dubious, possibly illegal, ways has lit
a firestorm. Mr. Zuckerberg took five days to reply and, when he did, he conceded that Facebook had let its users
down in the past but seemed not to have grasped that its business faces a wider crisis of confidence. After months of
talk about propaganda and fake news, politicians in Europe and, increasingly, America see Facebook as out of control
and in denial. Congress wants him to testify. Expect a roasting.
Since the news, spooked investors have wiped 9% off Facebook’s shares. Consumers are belatedly waking up to the
dangers of handing over data to tech giants that are run like black boxes. Already, according to the Pew Research
Centre, a think tank, a majority of Americans say they distrust social-media firms. Mr. Zuckerberg and his industry
need to change, fast.
The addiction game
Facebook’s business relies on three elements: keeping users glued to their screens, collecting data about their
behavior and convincing advertisers to pay billions of dollars to reach them with targeted ads. The firm has an
incentive to promote material that grabs attention and to sell ads to anyone. Its culture melds a ruthless pursuit of
profit with a Panglossian and narcissistic belief in its own virtue. Mr. Zuckerberg controls the firm’s voting rights.
Clearly, he gets too little criticism.
In the latest fiasco, it emerged that in 2013 an academic in Britain built a questionnaire app for Facebook users,
which 270,000 people answered. They in turn had 50m Facebook friends. Data on all these people then ended up with
CA. Facebook says that it could not happen again and that the academic and CA broke its rules; both deny doing
anything wrong. Regulators in Europe and America are investigating. Facebook knew of the problem in 2015, but it did
not alert individual users. Although nobody knows how much CA benefited Mr. Trump’s campaign, the fuss has been
amplified by the left’s disbelief that he could have won the election fairly. But that does not give Facebook a defense.
The episode fits an established pattern of sloppiness towards privacy, tolerance of inaccuracy and reluctance to admit
mistakes. In early 2017 Mr. Zuckerberg dismissed the idea that fake news had influenced the election as “pretty
crazy”. In September Facebook said Kremlin-linked firms had spent a mere $100,000 to buy 3,000 adverts on its
platform, failing at first to mention that 150m users had seen free posts by Russian operatives. It has also repeatedly
misled advertisers about its user statistics.
2. Which of the following given statements is correct in the context with the passage?
(A) Mark Zuckerberg will be the president in 2020 and seek to lead the world's most powerful country US.
(B) Facebook's founder showed that he is capable of leading the world's 8th largest company.
(C) After months of talks about propaganda, America sees Facebook as out of control and in denial.
(D) Investors have invested 9% more in the company after the campaign Cambridge Analytica.
(E) Mark Zuckerberg took 5 days to reply and when he did, he proved that Facebook had never let down the
confidence of its users.
PAGE- 94
(B) collecting data about their users' behaviour
(C) convincing advertisers to pay billions of dollars to reach them with targeted ads
(D) All of the above
(E) None of the above
9. Choose the word which is MOST SMILAR to the word given in passage
FUSS
(A) Joy
(B) Happiness
(C) Sorrow
(D) Worry
(E) Tiring
10. Choose the word which is MOST SMILAR to the word given in passage
RELUCTANCE
(A) Willingness
PAGE- 95
(B) Unwillingness
(C) Disapproval
(D) Acceptance
(E) Restoration
(Direction 11 – 20): We find that today the unity and integrity of the nation is threatened by the divisive forces of
regionalism, linguism and communal loyalties which are gaining ascendancy in national life and seeking to tear apart
and destroy national integrity. We tend to forget that India is one nation and we are all Indians first and Indians last. It
is time we remind ourselves what the great visionary and builder of modern India Jawaharlal Nehru said, “Who dies if
India lives, who lives if India dies?” We must realise, and this is unfortunately what many in public life tend to
overlook, sometimes out of ignorance of the forces of history and sometimes deliberately with a view to promoting
their self interest, that national interest must inevitably and forever prevail over any other considerations proceeding
from regional, linguistic or communal attachments. The history of India over the past centuries bears witness to the
fact that India was at no time a single political unit. Even during the reign of the Maurya dynasty, though a large part
of the country was under the sovereignty of the Mauryan kings, there were considerable portions of the territory
which were under the rule of independent kingdoms. So also during the Mughal rule which extended over large parts
of the territory of India, there were independent rulers who enjoyed political sovereignty over the territories of their
respective kingdoms.
It is an interesting fact of history that India was forged into a nation, neither on account of a common language nor on
account of the continued existence of a single political regime over its territories but on account of a common culture
evolved over the centuries. It is cultural unity—something more fundamental and enduring than any other bond
which may unite the people of a country together which has welded this country into a nation. But until the advent of
the British rule, it was not constituted into a single political unit. There were, throughout the period of history for
which we have fairly authenticated accounts, various kingdoms and principalities which were occasionally engaged in
conflict with one another. During the British rule, India became a compact political unit having one single political
regime over its entire territories and this led to the evolution of the concept of a nation. This concept of one nation
took firm roots in the minds and hearts of the people during the struggle for independence under the leadership of
Mahatma Gandhi. He has rightly been called the Father of the Nation because it was he who awakened in the people
of this country a sense of national consciousness and instilled in them a high sense of patriotism without which it is
not possible to build a country into nationhood. By the time the Constitution of India came to be enacted, insurgent
India, breaking a new path of non-violent revolution and fighting to free itself from the shackles of foreign
domination, had emerged into nationhood and “the people of India” were inspired by a new enthusiasm, a high and
noble spirit of sacrifice and above all, a strong sense of nationalism and in the Constitution which they framed. They
set about the task of a strong nation based on certain cherished values for which they had fought.
11. The author has quoted Jawaharlal Nehru to emphasize the point that
(A) national interest must enjoy supreme importance
(B) India is going to survive even if the world is under the spell of destruction
(C) the world will be destroyed if India is on the threshold of destruction
(D) the survival of the world depends only upon the well-being of India
(E) None of these
12. What, according to the author, is the impact of the divisive forces on our nation?
(A) They promote a sense of regional pride.
(B) They help people to form linguistic groups.
(C) They separate groups of people and create enmity among them.
(D) They encourage among people the sense of loyalty to their community.
(E) They remind us of our national pride.
PAGE- 96
(D) a threat to the solidarity of the nation
(E) None of these
14. Which of the following was instrumental in holding the different people of India together?
(A) A common national language
(B) A common cultural heritage
(C) The endurance level of the people
(D) Fundamentalist bent of mind of the people
(E) None of these
15. The passage appears to have been written with the purpose of
(A) giving a piece of advice to politicians of free India
(B) assessing the patriotic values and sacrifices made by people for India’s freedom
(C) justifying the teaching of Mahatma Gandhi and its impact on the people
(D) giving a historical account of how India evolved as a nation
(E) None of these
16. History shows that India, which was not a political unit earlier, became so
(A) during the reign of Maurya dynasty
(B) during the Mughal rule
(C) after one-national-language policy was adopted
(D) during the regime of independent rulers
(E) during the British rule
17. The “people of India”, as highlighted by the author in the last sentence of the passage, refer to
(A) the people of one unified nation
(B) the subjects of several independent rulers
(C) the patriots who sacrificed themselves in the freedom struggle
(D) the people who were instrumental in writing the Constitution
(E) None of these
19. Which of the following statements is/ are definitely true in the context of the passage?
(I) The people of India had fought for certain values.
(II) The fight of the Indian people was for one common culture.
(II) The Indian people lacked sense of nationalism until they gained freedom.
(A) Only (I)
(B) Only (II)
(C) Only (III)
(D) Both (I) and (II)
(E) Both (I) and (III)
20. Choose the word which is most opposite in meaning to the word ‘attachments’ as used in the passage.
(A) predicaments
(B) hatred
(C) harmony
(D) mistrust
(E) loyalty
PAGE- 97
(Direction 21 – 25): The stubborn persistence of child malnutrition in India is one of the tragedies of our time. Many of
us have long
agonized over this preventable problem, and we continue to ask: why do half of our children not get enough or the
right food or adequate care? Even in sub-Saharan Africa, only 30 per cent of the children are malnourished, versus 50
per cent in South Asia. And this gap exists despite our much higher levels of per capita income, education and even
safer water access. One-third of the babies in India are born with low birth weight compared to one-sixth in sub-
Saharan Africa. This is heartbreaking given the dramatic improvements in our agriculture, advances in literacy, and
great strides in economic growth. For more than 20 years India has even sustained the greatest effort in history to
improve nutritional standards, according to UNICEF, through its Integrated Child Development Services (ICDS)
Programme. So it is not for lack of effort. Nor is it due to poverty, which has been steadily declining by one per cent a
year for two decades. What accounts for this puzzle? In 1996, India’s famous physician nutritionist wrote aground-
breaking article on this called ‘The Asian Enigma’. After considering different factors, including access to food and
income and our vegetarianism, he concluded that the lower status of women might be the reason. The link between
women’s status and child nutrition seems plausible. In many Indian homes, men eat first; women have to make do
with leftovers. This is perhaps why 83 per cent of women in India suffer from iron deficiency-anaemia versus 40 per
cent in sub-Saharan Africa. A malnourished mother will give birth to a baby with low birth weight. Moreover, domestic
work often forces a mother to delegate the chore of feeding solid food to her baby to older siblings. If women had
more control over family income and decisions, they would devote them to better pre and post-natal care and to their
children.
So far this was the theory. But now a study by the International Food Policy Research Institute and Emory University
seems to confirm this hypothesis. It brought together data from 36 developing countries, spanning over one hundred
thousand children under the age of three and an equal number of women. It measured a woman’s position in the
home—whether she works for cash, her age at marriage, and the difference in age and education between spouses.
The study concludes that the lowly position of women in the family is the single most important reason for the gap in
children’s nutrition between South Asia and sub-Saharan Africa, followed by sanitation (lack of latrines) and
urbanisation (slum living).
I wonder why the position of women in India is worse than that of women in other societies. The report seemed to
suggest that South Asian women were not so far behind African women as their inferior status too limited their ability
to nurture children. I also wonder whether children’s well being is only a woman’s issue or a family concern where
men play a crucial role. I suspect there are no easy answers. Women everywhere suffer from lower status, but in India
it appears to have devastating consequences. The policy implications are clear: if we want to reduce child
malnutrition, we must combine our child programmes with efforts to improve the situation of women. To succeed, we
need healthy children who’ll become tomorrow’s innovative adults. If we ignore gender inequality, we will continue to
produce stunted children, wasted lives, and untold misery.
21. A hypothesis related to low birth weight has now been confirmed. According to this, the major reason for this
state is
(A) Vegetarianism
(B) Illiteracy
(C) Illiteracy of women
(D) Status of women
(E) Slum living
22. Which type of scheme indicates that there was no lack of efforts in India for the last two decades to improve the
situation?
(A) Literacy
(B) Rural Development
(C) Child Development
(D) Family Planning
(E) Poverty Alleviation
23. In which of the following areas is South Asia’s performance better than that of sub-Saharan Africa?
(A) Safer drinking water
PAGE- 98
(B) Lower infant mortality rate
(C) Higher status of women
(D) Higher birth weight of children
(E) None of these
25. Which of the following was one of the measures of women’s position in the home?
(A) Number of children
(B) Difference in husband’s and wife’s income.
(C) Weights of child at birth
(D) Age of marriage
(E) None of these
(Direction 26 – 36): The internet has become the integral part of today’s generation of people; from communicating
through instant messages and emails to banking, travelling, studying and shopping, internet has touched every aspect
of life. With the growing use of the internet by people, protecting important information has become a necessity. A
computer that is not having appropriate security controls can be infected with malicious logic and thus any type of
information can be accessed in moments. Number of infected Web Pages and malicious websites can be seen every
day that infects the computer and allow hackers to gain illegal access to other computer systems.
Hacking of important data, network outages, computer viruses and other cyber related threats affect our lives that
range from minor inconvenience to serious incidents. Cyber threats can be caused due to negligence and
vulnerabilities, or unintentional accidents. The main objectives of such type of system attackers or hackers are to steal
confidential information, to make illegal monetary transactions, to destroy or to change data and the like. System
attackers can be terrorists, crackers or recreational hackers. They have a variety of tools that can harm or infect the
computer; usually they use malicious logic or virus to gain unauthorized access to a computer. Opening email
attachments that carry the virus, clicking malicious links or websites or unintentionally downloading a dangerous
program are common ways through which a computer can be infected and data can be stolen.
As the number of data networks, digital applications, as well as internet and mobile users are growing, so do the
chances of cyber exploitation and cyber crimes. Even a small mistake in securing data or bad social networking can
prove to be extremely dangerous. If accounts are not properly secured, it makes easier for hackers or unauthorized
users to spread viruses or social engineered attacks that are designed to steal data and even money. Such types of
issues highlight the need for cyber security as an essential approach in protecting and preventing data from being
used inappropriately.
In simple language, Cyber Security or Information technology security means protecting data, networks, programs and
other information from unauthorized or unintended access, destruction or change. It encompasses all the
mechanisms and processes that protect digital equipment, information and records from illegal or unintended
access, manipulation or destruction.
In today’s dynamic environment, cyber security has become vital for individuals and families, as well as organizations
(such as military, government, business houses, educational and financial institutions, corporations and others) that
collect and store a wide range of confidential data on computers and transmit that to other computers across
different networks. For families, protection of children and family members from cyber crime has become
substantially important. For an individual, protecting information
that could impact social life as well as personal finance is essential. The internet has provided a wide array of learning
opportunities, but there are risks too. Photos, videos and other personal information shared by an individual on social
networking sites such as Facebook, Twitter can be inappropriately used by others may lead to serious and even life-
PAGE- 99
threatening incidents. Social networking sites have become the most popular medium for sharing information and
connecting with other people. But these sites have created varied opportunities for cybercrimes, compromised
personal identities and information leakage. Therefore, it is important for individuals to understand how to protect
against cyber threats, and must also comprehend the difference between virtual and real world. One should learn
how to protect computers and personal information from being hacked and should engage in appropriate online
behaviour in order to eliminate changes of cyber threats and thereby creating a safer online environment.
26. According to the author what does the cyber security means?
(A) Cyber Security means the dispersion of important data and devise a structural engineering that allow easy flow of
information
(B) Cyber Security means protecting data, networks, programs and other information from unauthorized or
unintended access, destruction or change.
(C) Cyber Security means the blocking of all the websites that are available in free domain
(D) Cyber Security means the Hacking of important data, network outages, computer viruses and other cyber related
threats affect our lives that range from minor inconvenience to serious incidents.
(E) None of the above
27. According to the author, what is the main reasons behind cyber threats?
(A) the unknown person with malice
(B) due to intelligence, veneration and intentional threats.
(C) due to negligence and vulnerabilities, or unintentional accidents.
(D) by communicating through instant messages and emails to banking, travelling, studying and shopping on internet
(E) None of the above
29. According to this passage, how the virus gets into the computers?
(A) by saving data and clicking on the desktop
(B) by opening a document file in the search history
(C) by texting online and receiving files on social media platforms
(D) Opening email attachments that carry the virus, clicking malicious links or websites or unintentionally downloading
a dangerous program
(E) None of the above
31. In the following question, find the word that is most similar in the meaning to the word ‘malicious’?
(A) benefactor
(B) benign
(C) benevolent
(D) vitriolic
(E) vituperate
PAGE- 100
32. In the following question, find the word that is most similar in the meaning to the word “illegal”?
(A) symposium
(B) chagrin
(C) chauvinistic
(D) Felonious
(E) charlatan
33. In the following question, find the word that is opposite in the meaning to the word “unauthorized”?
(A) accredit
(B) unaccredited
(C) inadmissible
(D) increment
(E) liaison
34. In the following question, find the word that is opposite in the meaning to the word “appropriate”?
(A) malapropos
(B) opportunely
(C) sequestrate
(D) usurp
(E) commandeer
35. In the following question, find the word that is most similar in the meaning to the word “unintentional”?
(A) despotic
(B) harbinger
(C) fortuitously
(D) immaculate
(E) innocuous
(Direction 36 – 45): The task which Gandhiji undertook was not only the achievement of political freedom but also the
establishment of a social order based on truth and non-violence, unity and peace, equality and universal brotherhood,
and maximum freedom for all. This unfinished part of his experiment was perhaps even more difficult to achieve than
the achievement of political freedom. Political struggle involved fight against a foreign power and all one could do was
either join it or wish it success and give it his moral support. In establishing the social order of this pattern, there was a
lively possibility of a conflict arising between groups and classes of our own people. Experience shows that man values
his possessions even more than his life because in the former he sees the means for perpetuation and survival of his
descendants even after his body is reduced to ashes. A new order cannot be established without radically changing
the mind and attitude of men towards property and, at some stage or the other, the ‘haves’ have to yield place to the
‘have-nots’. We have seen, in our time, attempts to achieve a kind of egalitarian society and the picture of it after it
was achieved. But this was done, by and large, through the use of physical force.
In the ultimate analysis, it is difficult, if not impossible, to say that the instinct to possess has been rooted out or that it
will not reappear in an even worse from under a different guise. It may even be that, like a gas kept confined within
containers under great pressure, or water held by a big dam, once a barrier breaks, the reaction will one day sweep
back with a violence equal in extent and intensity to what was used to establish and maintain the outward egalitarian
form. This enforced egalitarianism contains, in its bosom, the seed of its own destruction.
The root cause of class conflict is possessiveness or the acquisitive instinct. So long as the ideal that is to be achieved
is one of securing the maximum material satisfaction, possessiveness can neither be suppressed nor eliminated but
will grow on what it feeds. Nor will it cease to be such- it is possessiveness, still, whether it is confined to only a few or
is shared by many.
If egalitarianism is to endure, it has to be based not on the possession of the maximum material goods by a few or by
all but on voluntary, enlightened renunciation of those goods which cannot be shared by others or can be enjoyed
only at the expense of others. This calls for substitution of spiritual values for purely material ones. The paradise of
material satisfaction, that is sometimes equated with progress these days neither spells peace nor progress. Mahatma
Gandhi has shown us how the acquisitive instinct inherent in man could be transmuted by the adoption of the ideal of
PAGE- 101
trusteeship by those who ‘have’ for the benefit of all those who ‘have not’ so that, instead of leading to exploitation
and conflict, it would become a means and incentive for the amelioration and progress of society, respectively.
37. According to the passage, why does man value his possessions more than his life?
(A) He has an inherent desire to share his possessions with others
(B) He is endowed with the possessive instinct
(C) Only his possessions help him earn love and respect from his descendants
(D) Through his possessions he can preserve his name even after his death
(E) None of these
38. According to the passage, which was the unfinished part of Gandhiji’s experiment?
(A) Educating people to avoid class conflict
(B) Achieving total political freedom for the country
(C) Establishment of an egalitarian society
(D) Radically changing the mind and attitude of men towards truth and non-violence
(E) None of these
39. Which of the following statements is ‘not true’ in the context of the passage?
(A) True egalitarianism can be achieved by giving up one’s possessions under compulsion
(B) Man values his life more than his possessions
(C) Possessive instinct is a natural part of the human being
(D) In the political struggle, the fight was against the alien rule
(E) The root cause of class conflict is possessiveness
41. According to the passage, people ultimately overturn the form of a social order
(A) Which is based on coercion and oppression
(B) Which does not satisfy their basic needs
(C) Which is based upon conciliation and rapprochement
(D) Which is not congenial to the spiritual values of the people
(E) None of these
43. Which of the following statements is not true in the context of the passage?
PAGE- 102
(A) A new order can be established by radically changing the outlook of people towards it
(B) Adoption of the ideal of trusteeship can minimise possessive instinct
(C) Enforced egalitarianism can be the cause of its own destruction
(D) Idea of new order is to secure maximum material satisfaction
(E) None of these
45. According to the passage, what does adoption of the ideal of trusteeship mean?
(A) Equating peace and progress with material satisfaction
(B) Adoption of the ideal by the ‘haves’ for the benefit of the society
(C) Voluntary, enlightened renunciation of the possessive instinct by the privileged class
(D) Substitution of spiritual values by material ones by those who live in the paradise of material satisfaction
(E) None of these
(Direction 46 – 54): A few weeks ago, I ran into an old friend who is currently one of the mandarins deciding India’s
economic and financial policies. He asked “And so, how is IIT doing?” As one can only indulge in friendly banter at such
gatherings, I responded with ‘Not so well actually. Your market-friendly policies have forced up to raise the fee, so we
have 50% fewer Ph.D. applicants this year’. Not batting an eyelid, he shot back: “Obviously. Your Ph.D. students don’t
have any market value.” Taken aback, I shifted to a more serious tone and tried to start a discussion on the need for
research in these globalised times. But he had already walked away. The last word on the imperatives of the ‘market’
had been spoken.
This view of higher education should not have surprised me. Worthies who look at everything as consumer products
classify higher education as a ‘non-merit’ good. Non-merit goods are those where only the individual benefits from
acquiring them and not the society as a whole. Multilateral agencies like The World Bank have too been pushing
countries like India to stop subsidies to higher education.
When Ron Brown, former US commerce secretary visited India, a public meeting was organized at IIT Delhi. At that
meeting I asked him: “I understand that since the 19th century all the way up to the 1970s, most land grant and State
universities in the US virtually provided free education to State citizens. Was that good for the economy, or should
they have charged high fees in the early 20th century? “He replied,” it was great for the economy. It was one of the
best things that the US government did at that particular time in American history-building institutions of higher
education which were accessible to the masses of the people. I think it is one of the reasons why our economy grew
and prospered, one of the way in which the US was able to close some of its social gaps. So people who lived in rural
areas would have the same kind of access to higher education as people living in other parts of the country. It was one
of the reasons for making America strong.”
Our policy-makers seem unaware that their mentors in the US did not follow policies at home which they now
prescribe for other countries. Ron Brown’s remarks summarise the importance policy-makers in the US place on
higher education as a vehicle for upward mobility for the poorer sectors of their population. Even today, a majority of
Americans study in State-run institutions. Some of these institutions, like Berkeley and the Universities of Michigan,
Illinois, Ohio, Wisconsin and Taxas are among the best in the world. The annual tuition charged from State residents
(about $5000 a year) is about a month’s salary paid to a lecturer. Even this fee is waived for most students. In
addition, students receive stipends for books, food and hostel charges. The basic principle is that no student who gets
admission to a university should have to depend on parental support if it is not available.
Ron Brown’s remarks went unnoticed in India. Every other day some luminary or the other opines that universities
and technical education institutions should increase their charges and that such education should not be subsidized.
Most editorials echo these sentiments. Eminent industrialists pontificate that we should run educational institutions
like business houses. Visiting experts from the Bank and the IMF, in their newly emerging concern for the poor, advise
PAGE- 103
us to divert funds from higher education to primary education.
47. What was the net tangible impact of raising fees on the higher level of technological research?
(A) The number of prospective researchers was reduced to almost a half
(B) The market value of Ph.D. students was almost lost
(C) Research studies attained a higher market value
(D) Research became more and more relevant to market demands
(E) In the current globalised times, the need for research was less than ever
49. Who among the following support the view that higher education should be free to everyone aspiring for it?
(A) Editors and Journalists’
(B) Industrialists
(C) Visiting Experts from Banks and IMF
(a) A only
(b) B only
(c) C only
(d) All the three
(e) None of these
50. Which of the following makes the policy-makers classify education as “non-merit” commodity?
(A) The tendency of people to seek any individual benefits
(B) The attitude of giving unreasonably more weightage to society
(C) The tendency of viewing everything as mere consumer product
(D) Undue pressure from International Agencies like the World Bank, etc.
(E) None of these
51. What was Ron Brown’s reaction to the author’s question on free education provided by US universities to their
citizens? Ron Brown
(A) Criticized the US government for their action
(B) Appreciated the author but remained non-committed
(C) Ignored the fact and gave an ambiguous reaction
(D) Mentioned that the author’s information was not correct
(E) None of these
52. The basic principle adopted by the renowned State-run universities in the US is that the students __________.
(A) Must pay the lecturer’s salary from their own resources
(B) Should earn while they learn and pay higher education fees
(C) Must seek the necessary help from their parents on whom they depend
(D) Need not be required to depend upon their parents for acquiring higher education
PAGE- 104
(E) None of these
53. What was the outcome of US strategy of imparting free university education to US citizens?
(A) Education was easily accessible to the vast majority
54. Multilateral agencies like The World Bank have been __________
(A) Pressurizing India and other countries to stop substantial higher education
(B) Insisting on discontinuance of subsidies to higher education
(C) Analyzing the possibilities of increasing subsidies to higher learning
(d) emphasizing on the need of lowering fees for higher education
(e) Forcing countries like India to strengthen only industrial development
(Direction 55 – 64): Environmental protection and management is deservedly attracting a lot of attention these days.
This is a desirable development in the face of the alarming rate of natural resource degradation which greatly
hampers their optimal utilization. When waste waters emanating from municipal sewage, industrial effluent,
agricultural and land runoffs, find their way either to ground water reservoirs or other surface water sources, the
quality of water deteriorates, rendering it unfit for use. The natural balance is disturbed when concentrated
discharges of waste water is not controlled. This because the cleansing forces of nature cannot do their job in
proportion to the production of filthy matter.
According to the National Environment Engineering and Research Institute (NEERI), a staggering 70 percent of water
available in the country is polluted. According to the Planning Commission, “From the Dal lake in the North to the
Periyar and chaliyar rivers in the South, from Damodar and Hoogly in the East to the Thane creek in the West, the
picture of water pollution is uniformly gloomy. Even our large perennial rivers, like the Ganga, are today heavily
polluted.”
According to one study, all the 14 major rivers of India are highly polluted. Besides the Ganga, these rivers include the
Yamuna, Narmada, Godavari, Krishna and Cauvery. These rivers carry 85 percent of the surface runoff and their
drainage basins cover 73 percent of the country. The pollution of the much revered Ganga is due in particular to
municipal sewage that accounts for 3/4th of its pollution load. Despite India having legislation on water pollution [The
water (Prevention and Control of Pollution) Act, 1974] and various water pollution control boards, rivers have today
become synonymous with drains and sewers.
Untreated community wastes discharged into water courses from human settlements account for four times as much
waste water as industrial effluent. Out of India’s 3,119 towns and cities, only 217 have partial (209) or full (8)
sewerage treatment facilities and cover less than of third of the urban population, Statistics from a report of the
Central Board for Prevention and Control of Water Pollution reveal that 1,700 of 2,700 water using industries in India,
are polluting the water around their factories. Only 160 industries have waste water treatment plants. One estimate
suggests that the volume of waste water of industrial origin will be comparable to that of domestic sewage in India by
2000 A.D. Discharges from agricultural fields, which carry fertilizing ingredients of nitrogen, phosphorus and pesticides
are expected to be three times as much as domestic sewage. By that date, thermal pollution generated by discharges
from thermal power plants will be the largest in volume.
Toxic effluents deplete the level of oxygen in the rivers, endanger all aquatic life and render water absolutely unfit for
human consumption, apart from affecting industrial production. Sometimes, these effects have been disastrous. A
recent study reveals that the water of the Ganga, Yamuna, Kali and Hindon rivers have considerable concentration of
heavy metals due to inflow of industrial wastes, which pose a serious health hazard to the millions living on their
bands. Similarly, the Cauvery and Kapila rivers in Karnataka have been found to contain metal pollution which
hreatens the health of people in riverine towns. The Periyar, the largest river of Kerala, receivers extremely toxic
PAGE- 105
effluent that result in high incidence of skin problems and fish kills. The Godavari of Andhra Pradesh and the Damodar
and Hoogly in West Bengal receive untreated industrial toxic wastes. A high level of pollution has been found in the
Yamuna, while the Chambal of Rajasthan is considered the most polluted river in Rajasthan. Even in industrially
backward Orissa, the Rushikula river is extremely polluted. The fate of the Krishna in Andhra Pradesh, the
Tungabhadra in Karnataka, the Chaliyar in Kerala, the Gomati in U.P., the Narmada in M.P. and the Sone and the
Subarnarekha rivers in Bihar is no different.
According to the W.H.O., eighty percent of diseases prevalent in India are water-borne; many of them assume
epidemic proportions. The prevalence of these diseases heighten under conditions of drought. It is also estimated that
India loses as many as 73 million man days every year due to water borne diseases, costing Rs. 600 crore by way of
treatment expenditure and production losses. Management of water resources with respect to their quality also
assumes greater importance especially when the country can no more afford to waste water.
The recent Clean-the-Ganga Project, with an action plan estimated to cost the exchequer Rs. 250 crore (which has
been accorded top priority) is a trend setter in achieving this goal. The action plan evoked such great interest that
offers of assistance have been received from France, U.K., U.S. and the Netherlands, as also from the World Bank. This
is indeed laudable. Poland too has now joined this list.
The very fact that these countries have volunteered themselves to contribute their mite is a healthy reflection of
global concern over growing environmental degradation and the readiness of the international community to
participate in what is a truly formidable task. It may be recalled that the task of cleansing the Ganga along the
Rishikesh-Hardwar stretch under the first phase of the Ganga Action Plan, has been completed and the results are
reported to be encouraging.
The crisis of drinking water is deepening because water resources are drying up and the lowering of ground water
through over pumping, this is compounded by the pollution of water resources. All these factors increase the
magnitude of the problem. An assessment of the progress achieved by the end of March 1985, on completion of the
first phase of the International Drinking Water Supply and Sanitation Dacade (1981–91), reveals that drinking water
has been made available to 73 percent of the urban population and 56 percent to the rural population only. This
means that nearly half the country’s rural population has to get drinking water facilities. This needs to be urgently
geared up especially when considered against the Government’s professed objective of providing safe drinking water
and sanitation to all by the end of the International Drinking Water Supply and Sanitation Decade, i.e., March 1991.
The foremost action in this would be to clean up our water resources.
As per surveys conducted by the NEERI, per capita drinking water losses in different cities in the country range
between 11,000 to 31,000 liters annually. This indicates a waste level of 20-35 percent of the total flow of water in the
distribution system, primarily due to leaks in mains and household service pipes. Preventive maintenance programme
would substantially reduce losses/wastages and would certainly go a long way in solving the problem.
According to the Union Ministry of Works and Housing, out of 2.31 lakh problem villages identified in 1980, 1.92 lakh
(83 percent) villages have been provided with at least one source of drinking water as of March 1986. The balance
(38,748) villages are expected to be covered during the seventh plan. A time-bound national policy on drinking water
is being formulated by Government, wherein the task is proposed to be completed by the end of the seventh plan. An
outlay of Rs. 6,522.47 crores has been allotted for the water supply and sanitation sector in the seventh plan period,
against an outlay of Rs. 3,922.02 crores in the sixth plan. Of this, outlay for rural water supply sector is Rs. 3,454.47
crores. It is expected that this outlay would help to cover about 86.4 percent of the urban and 82.2 percent of the
rural population with safe drinking water facilities by March 1991.Hygienic sanitation facilities would be provided to
44.7 percent and 1.8 percent of the urban and rural population respectively within, the same period.
59. The cost of the Clean-the-Ganga Pollution Project Action Plan is likely to be sourced from:
(A) the Indian Exchequer.
(B) France, U.K., U.S and the Netherlands.
(C) the World Bank, Poland, U.K.
(D) the U.S., U.K., Netherlands, Poland, France, the World Bank and India.
(E) None of the above.
(Direction 65 – 74): As far as we know, all animals dream; and humans probably started to dream even before they
were sufficiently intelligent to think about the process. It has been suggested that some of the earliest prehistoric cave
paintings are records of dreams. Freud set out the theory that, although they may be
prompted by external stimuli, wish-fulfillment was the basis of most dreams. According to him, our dreams reflected
our deepest desires, rooted in our infancy, and always held a serious meaning. He stressed the erotic content of
dreams. Carl Gustav Jung collaborated with Freud for some years, but disagreed with him on this very point: hidden
sexual problems were not, Jung argued at the root of most dreams. Freud believed that dreams were the result of
concealed desires and, continued, on the whole, to conceal them; Jung, on the other hand, felt that dreams revealed
our deepest wishes and longings enabling us to realize our unconscious ambitions and helping us to fulfil them. Jung
suggested that dreams are, in fact, important messages from ourselves to ourselves, and messages that we ignore to
our loss. Most modern psychologists tend to lean towards Jung rather than Freud. Freud would have denied that
someone could be taught to interpret their own dreams; whereas Jung believed that although it was a difficult task it
could and should be done for, dreams were “meant” to be understood. The idea of losing consciousness, of ceasing to
be ourselves, and of relinquishing all control over our thoughts and movements, is dreadful to us; and yet it happens
every night when we sleep, For centuries, people thought of sleep as a period when humans rested their bodies and
their minds. Even in the early part of the last century, It was believed that during the day, blood rose to the brain and
caused congestion there. During sleep, the blood drained back into the rest of the body (and therefore it was best to
sleep without a pillow so that the blood could flow more easily from the brain). Early this century, scientists suggested
that certain chemicals, such as lactic acid, carbon dioxide and cholesterol, collected in the brain during waking hours
and were then depleted during sleep. The question remains, what is the purpose of sleep? No cases have ever been
recorded in which physical illness has resulted from lack of sleep, although the brain probably does need sleep, since
measurements of brain activity have shown some chemical changes during sleep deprivation.
The modern understating of the nature of sleep began just over 40 years ago. In 1952, a researcher noticed that at
certain times during a period of sleep the eyes of the subjects could be seen stirring beneath their closed lids – as
though they were watching moving figures. These motions were called “rapid eye movements” and the phases of
sleep were called REM periods. Three years later, it was found that during REM sleep, the flow of blood to the brain
increased, as did the brain’s temperature, particular brain wave patterns showed up on an electroencephalograph
(EEG). Irregularities in breathing and heartbeat were noted during REM sleep, and a reduction in electrical activity in
certain muscles. It was also discovered that if a person was woken up during REM sleep, they could usually remember
vivid dreams; while only about six per cent of people woken during NREM (non-rapid eye movement) sleep claimed to
have been dreaming. It seemed to be the case that only during NREM sleep were humans really “unconscious”, and
apparently indulging in complete rest. Although about half of the people awakened during this period believed they
had been dreaming, they thought that their dreams were more like daydreams – seeming less surreal than “real”
dreams. These discoveries were so interesting that they led to an intense period of the study of sleep patterns, and
most of our knowledge about the nature of sleep emerged from studies made over the next 20 years.
When we fall asleep we enter a cycle of sleep – a pattern that is usually repeated several times during the night.
Scientists identify four stages of sleep – the first stage is simply a transition from wakefulness to real sleep; while stage
two may be described as “normal” sleep. During stage three, there is another transition, or sinking into a deeper sleep
– that of stage four. During sleep, what is happening in the brain can be measured by the use an EEG. Electrodes
placed on the scalp pick up “brain waves” of about one – millionth of a volt in strength, which are amplified and traced
on paper or recorded on tape, where changes in frequency (the number of waves taking place within one second) can
be seen. Four types of EEG have been particularly studied: Beta waves are fast waves that show when the brain is
animated or anxious. Alpha waves which show during periods of meditation, when the brain is wakeful but relaxed.
PAGE- 108
Theta waves occurring during drowsiness or light sleep; and Delta waves slow waves that are seen during times of
deep sleep. The whole cycle lasts around ten minutes of REM sleep, when dreams occur, before “climbing” back
through three layers of NREM sleep. The whole cycle lasts around 80 or 90 minutes. During perhaps four cycles
repeated throughout a single night’s sleep, we spend around six hours in NREM sleep, and the remaining two in REM
“dream-time”. Just under half of us wake only from NREM sleep, and these include those people “who claim that they
never dream.” It has been suggested by some psychologists that these people unconsciously wake themselves at a
time when they are not dreaming because they want to repress what their dreams are telling them.
65. If you get up actually remembering a dream, then you would most probably have got up from
(A) REM sleep
(B) A stage where your brain waves are of theta type.
(C) NREM stage.
(D) Surreal dreams.
(E) None of these
69. If you are stuck in a lecture which is uninteresting in your view, your brain, most probably, sends out
(A) Beta waves.
(B) Alpha waves.
(C) Delta waves.
(D) Theta waves.
(E) None of these
71. “……. Yet it happens every night” the world ‘it’ implies
(A) Sleep.
(B) dreaming.
PAGE- 109
(C) Dreading.
(D) Being not in control of one’s movements or thoughts.
(E) None of these
73. As suggested by a few psychologists, people who say that they never dream are
(A) obviously bluffing.
(B) In fact waking themselves up during REM sleep.
(C) Not trying to listen to what their dreams are saying.
(D) Depriving themselves of a chance of attaining their unrealized desires.
(E) None of these
74. What happens to the level of cholesterol in the brain during sleep?
(A) It goes up
(B) It goes down.
(C) It remains constant.
(D) It fluctuates
(E) None of these
Directions (Q. 75–79): Read the passage carefully and answer the questions given below it. Certain words/ phrases
have been given in bold to help you locate them while answering some of the questions.
Sting operations are a decade-and-a-half old in India. In 2000, sting pioneer Tehelka did one on cricketers to throw
light on how matches were fixed. In 2001, their ‘Operation West End’ was on middlemen in defence deals and bribes
taken by political leaders. And in 2007, ‘Operation Duryodhan’ caught 11 MPs taking bribes to put dictated questions
in Parliament. Many more followed. It is said that the expression ‘sting operation’ seemed to have emerged from the
name of a popular 1973 American movie ‘The Sting’, which was based on a complicated plot hatched by two persons
to trick a third person into committing a crime. A sting operation raises certain moral and ethical questions. The
victim, who is otherwise innocent, is lured into committing a crime on the assurance of absolute secrecy and
confidentiality of the circumstances, raising the potential question as to how such a victim can be held responsible for
the crime which he would not have committed but for the enticement.
Though sting operations have been around, it was Aam Aadmi Party and its chief Arvind Kejriwal who sharpened it as
a weapon in the mobile phone-wielding hands of the public to catch the corrupt. AAP and Kejriwal relentlessly
exhorted people to sting officials who demanded illegal gratification. The stings streamed in, catching both the big and
the small. The latest one doing the rounds is the one on Kejriwal himself. A private conversation secretly recorded has
become the hot topic of public debate.
75. According to the author, what is the main cause of sting operations?
(A). Sting operations are for the betterment of society and public good.
(B) Sting operations are about organised people in law enforcement.
(C) Because all such operations are paid.
(D)It is a source of employment.
(E)Other than given options.
78. Choose the word which is most similar in meaning to the word 'Pioneer' as used in the passage?
(A) Speculator
(B)Trailblazer
(C)Settler
(D)Follower
(E)Immigrant
79. Choose the word which is most opposite in meaning to the word 'Hatched' as used in the passage?
(A)Conspired
(B)Produced
(C)Devised
(D)Managed
(E)Unplanned
Directions (Q. 80 - 85): Read the following passage carefully and answer the questions given below it.
The Central government’s framework for 20 cities to become ‘smart’ over a five-year period can cover new ground if it
makes intelligent use of information technology to deliver better civic services. Rapid and poorly regulated
urbanisation has overwhelmed urban governments, rendering them incapable of providing even basic services such
as clean water, sewerage, pedestrian-friendly roads, public transport, uninterrupted power, street lighting, parks and
recreational spaces. So weak and uncoordinated is governance that commercial entities have wilfully violated building
regulations and put up unauthorised structures — with severe impact on congestion, air quality and flood
management — and governments have gladly regularised the violations later. The smart city plan now proposes to
intervene and bring some order by upgrading the physical infrastructure in select enclaves, and incentivising the use
of information and communication technologies. Urban Development Minister M. Venkaiah Naidu has come up with a
generalised definition of a smart Indian city as one that “enables a decent life to the citizens, and green and
sustainable environment, besides enabling adoption of smart solutions”, but the exercise should lead to measurable
outcomes.
The first batch of smart cities would create virtually new business districts in several cities, marking a departure from
the disaggregated urban development witnessed over the past few decades. This area-based development approach
makes it imperative that the resulting demand for mobility to and from the ‘smart’ area be made an integral part of
the plan, with an emphasis on walk ability, use of non-motorised transport and access to public transport. Ahmedabad
and Bhubaneswar have shown high ambition by opting for a common travel card. Others such as Indore, Davangere
and Belagavi plan Intelligent Transport Solutions, something that has been unattainable for even a big metro such as
Chennai. Although it enjoys high visibility, the smart city programme is merely a framework for urban development
aided by the Centre with a small initial seed fund of Rs.500 crore, while additional finances have to come from public-
private partnerships and local revenue. State governments, including those left out of the first list, could unlock the
potential of all cities with development policies that aim at structural change. Improved public transport, for instance,
has an immediate positive impact on the local economy. Technologies such as GPS to inform passengers in real time
on their mobile phones, and common ticketing, increase the efficiency of transport use. Universal design in public
buildings and streets would help all people, including those with disabilities. The challenge for Smart Cities 1.0 is to
PAGE- 111
provide proof of concept quickly and make outcomes sustainable. Care also needs to be taken that the effect is not to
create gated communities of best practices and civic upgrade in a wider landscape of urban distress. It is crucial that
these urban enclaves cater to the housing, health, education and recreation needs of a wide cross section of society,
and that the convergence of the Smart Cities programme with existing urban renewal projects countrywide be
smooth.
Directions (Q. 83-85): Choose the word which is opposite in meaning to the word printed in bold as used in the
passage.
83. IMPERATIVE
(A) secondary
(B) inescapable
(C) crucial
(D) hanging by thread
(E) None of the above
84. RENDERING
(A) misunderstanding
(B) explanation
(C) realization
(D) comprehension
(E) None of the above
85. CONVERGENCE
(A) concurrence
(B) synchronism
(C) confluence
(D) divide
(E) None of the above
Directions (Q. 86-89): Choose the word which is most similar in meaning to the word printed in bold as used in the
passage
PAGE- 112
86. SUSTAINABLE
(A) imperishable
(B) perishable
(C) unendurable
(D) temporary
(E) None of the above
87. URBANISATION
(A) immunization
(B) sophisticate
(C) metropolitanize
(D) down town
(E) All of the above
88. RENEWAL
(A) reclamation
(B) delayed
(C) damage
(D) break
(E) destroy
PAGE- 113
ANSWER KEY
2. MENSURATION
1. B 6. B 11. C 16. A 21. C 26. C
2. A 7. A 12. C 17. D 22. B 27. A
3. B 8. B 13. B 18. A 23. B 28. B
4. A 9. B 14. A 19. D 24. B 29. A
5. C 10.D 15. B 20. A 25. C 30.
RACE
1. B 6. A 11. B 16. B
2. B 7. C 12. A 17. A
3. C 8. A 13. C 18. D
4. D 9. C 14. D 19. C
5. B 10. C 15. C 20. D
PAGE- 114
6. DATA SUFFICIENCY
1. E 6.B 11.D 16.A 21.B 26.C 31.D
2.D 7.C 12.E 17.B 22.C 27.D 32.E
3.A 8.A 13.D 18.C 23.B 28.C 33.E
4.C 9.D 14.D 19.B 24.C 29.A 34.D
5.E 10.D 15.B 20.D 25.A 30.E 35.B
7. SEATING ARRANGEMENT
1.C 6.C 11.C 16.C 21.B 26.B 31.B 36.D 41.B 46.B
2.D 7.D 12.E 17.B 22.D 27.D 32.D 37.A 42.C 47.E
3.C 8.C 13.A 18.A 23.B 28.C 33.B 38.A 43.D 48.A
4.E 9.C 14.D 19.D 24.D 29.C 34.A 39.C 44.C 49.B
5.C 10.D 15.B 20.A 25.B 30.D 35.B 40.D 45.B 50.D
8. SYLLOGISM
1. E 11. D 21. B 31. E 41. B 51. A 61. E 71. E
2. E 12. A 22. A 32. A 42. D 52. D 62. D 72. B
3. A 13. C 23. A 33. D 43. D 53. D 63. A 73. A
4. D 14. E 24. A 34. A 44. E 54. C 64. A 74. C
5. B 15. E 25. A 35. E 45. D 55. A 65. D 75. C
6. E 16. E 26. E 36. C 46. B 56. B 66. C 76. B
7. B 17. C 27. A 37. B 47. A 57. E 67. B 77.A
8. D 18. D 28. A 38. C 48. E 58. E 68. C
9. D 19. B 29. A 39. A 49. A 59. D 69. D
10.III 20. B 30. A 40. D 50. B 60. B 70. C
PAGE- 115
12. STATEMENT CONCLUSION
1. D 6. D 11.D 16.E 21.E 26.D 31.D 36.E
2. A 7. D 12.D 17.B 22.E 27.E 32.D 37.D
3. E 8. E 13.D 18.D 23.B 28.A 33.C 38.A
4. B 9. B 14.A 19.A 24.B 29.E 34.E 39.B
5. D 10.B 15.E 20.A 25.D 30.E 35.B 40.B
15. ADJECTIVES
1.C 6. C 11.B 16.C 21.C 26.A 31.C 36.C
2.A 7. A 12.C 17.C 22.A 27.B 32.B 37.B
3.B 8. B 13.B 18.C 23.A 28.B 33.C 38.A
4.A 9. B 14.C 19.C 24.C 29.B 34.B 39.A
5.B 10.C 15.B 20.C 25.A 30.B 35.C 40.B
16. ADVERB
1.C 6.C 11.D 16.D 21.A 26.C 31.B 36.B
2.C 7.A 12.A 17.B 22.C 27.B 32.B 37.B
3.B 8.B 13.B 18.B 23.B 28.A 33.B 38.A
4.A 9.A 14.B 19.B 24.C 29.C 34.B 39.B
5.A 10.C 15.B 20.C 25.C 30.A 35.B 40.A
PAGE- 116
19. SUBJECT VERB AGREEMENT
1. B 6.C 11.C 16.C 21.C 26.B 31.B 36.B
2.A 7.B 12.C 17.C 22.A 27.B 32.B 37.C
3.B 8.C 13.B 18.B 23.C 28.B 33.A 38.C
4.B 9.C 14.B 19.B 24.C 29.B 34.B 39.C
5.B 10.B 15.B 20.B 25.B 30.A 35.B 40.B
PAGE- 117
CAMPUS RECRUITMENT TRAINING
(B.TECH – 6TH SEMESTER)
INDEX
1. Data Interpretation…..……….…..……………………………………………………………………………………………………..119 - 124
5. Probability………………….………………………..………………………………………………………………………………………134 - 136
6. Statistics…………..…………………………..………….…………………………………………………………………………………..137 – 140
8. Syllogism……………………………….………………………………………………………………………………………………………146 – 153
PAGE- 118
DATA INTERPRETATION
Direction for the question: Refer the given diagram to answer the questions:
1. If in 2005-06, the total project outlay increased by 20%, find the percentage increase in MP’s outlay.
(a) 50% (b) 25% (c) 20% (d) 80% (e) None of these
2. If in 2005-06, the total project outlay increased by Rs.200 crore and that of UP remained unchanged, find the total
project outlay in 2005-2006, in Rs crore.
(a) 350 (b) 800 (c) 600 (d) 420 (d) None of these
3. If in 2005-06, the total project outlay of Gujarat increased by ‘x’ per cent, find the per cent increase in the total outlay.
(a) (100-4x)/5 (b) (5x-100)/4 (c) (100 – 5x)/4 (d) (4x -100)/5 (d) None of these
Directions (4 - 6): The two pie charts below show the percentage market share on value basis of the companies A to D and
others in a sectorial market for 1999 and 2000
1999 2000
5% 10%
15% A A
B B
40% 30%
20% C
C
20%
D 15% 25% D
20%
OTHERS OTHERS
5. If each company increases its sales value by 10%, then what is the percentage growth of the detergent market?
A. 10% B. 20% C. 30% D. 8%
6. If the total sales of the market is doubled for 1999 and 2000, what would be the ratio of sales of D for 2000 to 1999? A.
2:1 B. 3:4 C. 15:8 D. 10:3
Direction for the Question 7 – 11: Study the information given below and answer the following questions:
Mr X has built a mansion with 10 rooms. He was confused about the colours he should use while painting each room. He
had the following choice of colours: blue, hazy grey, jumping yellow, teal, violet latte, Terry Cherry and happy pink. It was
also known that he could paint more than 1 room with a single colour. Finally, he set up an algorithm to decide the colours
that he would be using.
● If he painted any room teal, then he did not paint any other room happy pink.
● If he painted any room blue, then he did not paint any other room jumping yellow.
● If he painted any room blue, then he painted at least one room happy pink.
● If he painted any room jumping yellow, then he painted at least one room violet latte.
● If he painted any room violet latte, then he painted at least one room happy pink.
● If he painted any room happy pink, then he painted at least rooms happy pink.
7. Which one of the following could be a complete list of the number of rooms and colours that Mr X used to paint some of
the rooms of his house?
A. one blue, one Terry cherry, one violet latte, two happy pink
B. one blue, one teal, one Terry cherry, three happy pink
C. two blue, one teal, three Terry Cherry
D. one jacket, one Terry cherry, two violet latte and one happy pink
8. If Mr. X did not paint any room happy pink, what was the maximum number of the different types of colours that he could
paint ?
A. two B. three C. four D. five
10. If Mr. X painted as many different types of colours as possible, then it must be true that he did not paint one of the
following types of colours.
A. blue B. hazy grey C. teal D. jumping yellow
11. If Mr. X painted at least one room, find out which one of the following are the minimum and the maximum numbers of
the types of colours that he could paint ?
A. 1, 4 B. 1, 5 C. 1, 6 D. 2, 5
Direction (12-16): Study the following table and answer the questions that follow:
PAGE- 120
12. What is the number of employees in Finance Department of offices A and D?
(a) 63 (b) 61 (c) 52 (d)59 (e) 55
14. In Office C, 25%, 20% and 50% people in HR, Finance and Others Department respectively are females. What is the
number of males in these departments in office C?
(a) 147 (b) 202 (c) 296 (d)213 (e) 234
15. What is the ratio of number of employees in Other Departments in offices A and D?
(a) 36 : 25 (b) 38 : 25 (c) 45 : 38 (d) 34 : 29 (e) 43 : 34
16. What is the total number of employees in Finance Department in all offices?
(a) 185 (b) 174 (c) 198 (d) 227 (e) 216
Direction (17- 21) Study the following table to answer the questions that follow. Some values are missing. Study
thetable carefully and answer the questions based on data in table and particular questions.
PAGE- 121
17. On Friday, if ratio of speed of boat to speed of stream is 2 : 1, what is the difference between the time taken by the
boat to go upstream and the time taken by the boat to go downstream?
(a)36 (b) 54 (c) 45 (d)72 (e) 65
18. On Monday, if difference between the time taken by the boat to go upstream and the time taken by the boat to go
downstream is 20 hours, find the total time taken by the boat to go upstream and downstream.
(a) 20 hours (b) 40 hours (c) 30 hours (d) 80 hours (e) 60 hours
19. On Tuesday, if difference between the time taken by the boat to go upstream and the time taken by the boat to go
downstream is 45 hours, find the total distance covered by the boat to go upstream and downstream. Downstream
speed is 24 km/hr.
(a) 720 km (b) 540 km (c) 360 km (d) 680 km (e) 450 km
20. On Wednesday, if the boat covered half distance upstream with usual speed and other half with double its speed,
then it takes 33 3/4 hours less time than usual time to go upstream. Find the total time taken by the boat to go
upstream and downstream.
(a) 120 hours (b) 108 hours (c) 108 hours (d) 84 hours (e) 128 hours
21. On Thursday, difference between time taken by the boat to cover ‘X’ km upstream and (X+120) km downstream is
26 hours. Had the boat covered same distance upstream as downstream, what would be the difference in time taken
then?
(a) 120 hours (b) 108 hours (c) 108 hours (d) 84 hours (e) 128 hours
Directions (22-26): Study the information carefully to answer the following questions
Data regarding number of employees working in various departments in Company A and B in the year 2018. Both
Companies have six departments namely Production, HR, Finance, R&D, Marketing and Accounts. The total number of
employees in company A is 9000. In Company A, number of employees in production, HR and finance together is 60 % of
the total number of employees. The number of employees in R&D, Marketing and Accounts were 1300, 1440 and 860
respectively. The number of employees in Production department was 25 % more than that of finance department. In
company B, the number of employees in Marketing was 900 and they constituted 12 % of the total number of
employees. Also the number of employees in Marketing was 40 % less than that of HR department. The number of
employees in production from company B was 10 % less than the same department from Company A. The Number of
employees in accounts is 500. Number of employees in finance and R&D department is same. Total Number of
employees in finance and R&D together were double the total Number of employees in Marketing and accounts
together.
22. What is the difference between the total Number of employees in Marketing and accounts together in Company A
and that in the same courses together in Company B?
(a) 700 (b) 200 (c) 400 (d) 600 (e) 900
23. 3/4th of the number of R&D employees in Company A was female. If the number of female R&D employees in
Company A is less than that of Company B by 175, what is the number of male R&D employee in Company B?
(a) 600 (b) 400 (c) 500 (d) 100 (e) 800
24. What is the respective ratio between the total number of employees in finance and production together in Company
A and that in the same courses together in company B?
PAGE- 122
(a) 1:9 (b) 7:3 (c) 4:9 (d) 9:8 (e) 3:2
25. Number of HR employees in Company B is what percent less than that in Company A?
(a) 10/4% (b) 50/3% (c) 26/7% (d) 12/6% (e) 43/6%
Directions (27-31): The bar graph shows the production (in thousand tonnes) of Wheat, Rice and Maize in different
states.
50
40
30 WHEAT
20 RICE
MAZE
10
0
UP MP BIHAR ODISHA HARYANA PUNJAB
The pie-chart shows the percentage of agricultural land in the given six states.
𝑻𝒐𝒕𝒂𝒍 𝑷𝒓𝒐𝒅𝒖𝒄𝒕𝒊𝒐𝒏
Productivity =
𝑨𝒓𝒆𝒂 𝒐𝒇 𝑨𝒈𝒓𝒊𝒄𝒖𝒍𝒕𝒖𝒓𝒂𝒍 𝑳𝒂𝒏𝒅
HARYANA
,8 UP
PUNJAB
UP, 30 ODISHA
MP, 25
BIHAR
BIHAR, MP
20 PUNJAB,
ODISHA, 12 HARYANA
5
29. The prodution of wheat in punjab is what per cent more than the production of Maize in odisha ?
A. 350% B. 250% C. 300% D. 200% E. 400%
30. What is the ratio of the production of Rice in Bihar to the production of Wheat in Haryana ?
A. 2:3 B. 3:2 C. 2:1 D. 1:1 E. 1:2
31. If MP exports 40% of Rice at the rate of Rs.30 per kg and UP exports 30% of Rice at the rate of Rs.32 per kg, then what is
the ratio of the income from the exports ?
PAGE- 123
A. 65:48 B. 31:42 C. 43:54 D. 57:62 E. 1:2
Directions (32- 39): Study the following graph and answer questions
32. Which year shows the maximum percentage of export with respect to production?
(a)1992 (b)1993 (c)1996 (d)1995 (e) None of these
34. If the area under tea production was less by 10% in 1994 than in 1993, then the approximate rate of increase in
productivity of tea in 1994 was
(a)97.22 (b)3 (c)35 (d)49 (e)None of the above
35. The average proportion of tea exported to the tea produced over the period is
(a)0.87 (b)0.47 (c)0.48 (d)0.66 (e) None of these
36. What is the first half-decade average per capita availability of tea?
(a)457 gm (b)535 gm (c)446 gm (d)430 gm (e) None of these
37. In which year was the per capita availability of tea minimum?
(a)1996 (b)1994 (c)1991 (d)1991 (e)None of these
38. In which year was there minimum percentage of export with respect to production?
((a)1991 (b)1992 (c)1993 (d)1994 (e)None of these
39. In which year we had maximum quantity of tea for domestic consumption?
(a)1994 (b)1991 (c)1993 (d)1996 (e)None of these
PAGE- 124
DATA SUFFICIENCY
Directions (1-10): In each of the following questions, a question is followed by two statements numbered I and II.
Read both the statements and answer accordingly.
A) If the data in statement I alone is sufficient to answer the question.
B) If the data in statement II alone is sufficient to answer the question.
C) If the data either in statement I alone or statement II alone are sufficient to answer question.
D) If the data given in both I and II together are not sufficient to answer the question.
E) If the data in both the statements I and II together are necessary to answer the question
1. What is the probability of getting two black socks from a drawer containing only black and red socks?
Statement I: There are total 36 socks in the drawer.
Statement II: The probability of the first sock being black is 1/6.
2. In a concert in school, amount gathered by selling the tickets was Rs 60000 when all were of equal value. Each
student got 1 ticket. What was the cost of 1 such ticket?
Statement I: If the price of each ticket had been Rs 50 less, the total amount gathered would have been 15000 less.
Statement II: If the price of each ticket had been Rs 50 more, the total amount gathered would have increased by
25%.
9. A man invested a sum of Rs 50,000. He invested some part at 6 % p.a. and remaining at 6.5% p.a. How much money
did he invest at 6 % p.a.?
125 | P a g
e
Statement I: The total interest amount paid after 1 year was Rs 3150
Statement II: The interest on one sum was twice that on the other.
10. What is the total cost of painting a cone at the rate of 20 Rs per square metre?
Statement I: The radius and the slant height of the flask are 14 m and 12 m respectively
Statement II: The area of the base of cone is 154 sq. m and its height is 7 m.
13. A, B and C invested Rs 52000 in a business in ratio 6 : 3 : 4 respectively. What is the profit % earned by them after a
year?
Statement I: C got Rs 8000 as his share of profit.
Statement II: The difference in profits earned by A and B is Rs 6000.
16. If the selling price of an article is Rs 12000, then what is the profit/loss% on it?
Statement I: The ratio of the selling and price and cost price of the article is 4 : 3.
Statement II: The difference between the cost price and selling price of the article is Rs 200.
18. There are 5 people in a group. Average age of the group decreases by 4 when two more people join the group. Find
the average age of the new group.
Statement I: Average age of the two people who joined later is 36 yrs.
Statement II: Total of the ages of people originally was 160 yrs.
19. What is the rate of interest per annum on a sum of money invested?
Statement I: The difference between the compound interest and simple interest at the end of 2 years at same rate
of interest is Rs 80.
126 | P a g
e
Statement II: A compound interest of Rs 4912 is obtained after 3 years of investment.
What is the two digit number?
20. Statement I: The difference between the digits of the two digit number is 6.
Statement II: The number obtained by interchanging the digits of original number is 26 more than twice the
original number.
22. Average age of employees working in Bank is 30 years. Next year, 10 workers will retire. What will be the average
age next year?
Statement I: There are 90 employees in bank
Statement II: Retirement age is 60 year
24. What is the difference between present ages of Amit and Sumit ?
Statement I: The ratio of the present age of Amit to that of Sumit six years ago is 2 : 1.
Statement II: The ratio of the present age of Amit and Sumit is 4 : 3.
25. What is the profit earned by selling an article for Rs. 900?
Statement I: 25% of profit would have been earned if it had been sold for Rs. 750.
Statement II: Selling price is 300% of the profit earned
Directions (26-30): In each of the following questions, a question is followed by three statements numbered I, II and
III. Read all the statements and answer accordingly.
27. A shopkeeper sells a homogeneous mixture of X and Y at a rate of Rs 400 per kg. Find the profit percentage of
the shopkeeper
Statement I: He brought X at the rate of Rs 350 per kg
Statement II: He bought Y at Rs 10 higher than the rate of X per kg.
Statement III: He brought Y at the rate of Rs 420 per kg
A) Only I and II
B) Only I and III
C) Any two of the three
D) All three together are sufficient
E) I , II and III together are not sufficient
28. How many students are there in all in the institute of Computer, Electrical and Mechanical ?
Statement I: 20% of the students study Mechanical.
127 | P a g
e
Statement II: The number of students studying Computer and Electrical are in the ratio of 5:3.
Statement III: The number of students studying Electrical is more than that of studying Mechanical by 100.
A) Only II and III
B) III and either I or II only
C) Any two of the three
D) All I, II and III
E) Question cannot be answered even with the information in all the three statements
128 | P a g
e
CRYPT ARITHEMATIC
9) GO + TO = OUT Then O + U + T ?
A) 4 B) 3 C) 5 D) 6 E) none of these
17) TIN + TIN + TIN = PIPE + PIPE where I =4 ,find the value of TIN ?
A) 840 B) 940 C) 960 D) 980 E) none of these
129 | P a g
e
19) Find the value of ABC if ABC=A!+B!+C! where ABC is a three digit number
A) 145 B) 155 C) 175 D) 185 E) none of these
20) If WAIT+ALL = GIFTS then what is the value of G+I+F+T .IF A=6 and S= 5
A) 15 B) 16 C) 17 D) 18 E) none of these
130 | P a g
e
PERMUTATION AND COMBINATION
1. How many 7-digits numbers can be formed by using the digits 1, 2, 0, 2, 4, 2 and 4?
(a) 720 (b) 360 (c) 420 (d) 120 (e) 620
2. Find the number of ways in which the letters of the word ARRANGE can be arranged such that both R do not come
together.
(a) 950 (b) 800 (c) 900 (d) 750 (e) 920
3. A teacher wants to select a girl out of 6 and a boy out of 8 for the drawing competition. In how many ways can be select?
(a) 48 (b) 52 (c) 36 (d) 24 (e) 60
4. There are 6 students to be seated around a circular table. In how many ways they can be seated if two particular persons
are next to each other.
(a) 60 (b) 52 (c) 72 (d) 48 (e) 42
5. 3 men and 3 women are to sit at a round table. In how many different ways can they sit so that no 2 women sit together?
(a) 16 (b) 12 (c) 18 (d) 21 (e) 10
6. The Chief Minister of 11 states of India meet to discuss the water problem. In how many ways can they sit at a round table
so that the Punjab and Uttar Pradesh Chief Ministers sit together.
(a) 725860 (b) 755760 (c) 725760 (d) 625760 (e) 765260
7. In a cricket championship, there are 21 matches. If each team plays one match with every other team, the number of
teams is
(a) 7 (b) 9 (c) 10 (d) 12 (e) none of these
8. How many three digits number can be formed by using the digits 0, 2, 4, 6, 7 if repetition of digits is not allowed
(a) 52 (b) 60 (c) 48 (d) 36 (e) 72
9. Find the numbers between 100 and 1000 in which all digits are distinct.
(a) 548 (b) 648 (c) 748 (d) 448 (e) 684
10. The letters of the word PROMISE are to be arranged so that three vowels should not come together. Find the number of
arrangements.
(a ) 3420 (b) 4320 (c) 5320 (d) 6420 (e) 4420
11. Find the sum of all the 4 digit numbers that can be formed with the digits 3, 4, 5 and 6
(a) 119988 (b) 11988 (c) 191988 (d) 121988 (e) None of these
12. In a group of 6 boys and 4 girls, four children are to be selected. In how many different ways can they be selected such
that at least one boy should be there?
(a) 159 (b) 194 (c) 205 (d) 209 (e) 224
13. The number of straight lines that can be drawn out of 12 points of which 8 are collinear is
(a) 39 (b) 29 (c) 49 (d) 59 (e) none
14. There are 5 blue flags, 4 red flags and 3 green flags, in Debu’s wardrobe. He has to select 4 flags from this set. In how
many ways can he select these four flags such that there is at least one blue flag and exactly one green flag in them (Do not
consider that the flags are in pairs)?
(a) 245 (b) 240 (c) 495 (d) 60 (e) 310
15. Find total number of the 3 digits odd numbers by using the digits 2, 3, 4, 5 when repetitions of digits are not allowed
(a) 12 (b) 22 (c) 15 (d) 18 (e) 24
131 | P a g
e
16. In how many ways can you select a diamond or a king from a pack of cards?
(a) 16 (b) 20 (c) 24 (d) 8 (e) 25
17. A circular table has 6 chairs, out of this 6, five are identical. In how many ways can the six people be arranged on these
chairs?
(a) 120 (b) 720 (c) 360 (d) 60 (e) 72
18. Jay invited 10 of his friends on his birthday. If all of them greeted each other with a handshake then how many
handshakes will take place?
(a) 90 (b) 110 (c) 45 (d) 55 (e) 24
19. In how many can the 4 couples sit around a circular table so that no two men are sitting together?
(a) 7! (b) 6! (c) 3! * 4! (d) 3! * 3! (e) 24
20. There are three dice each of them having faces with a number from 1 to 6. These dices are rolled Find the number of
possible outcomes such that at least one of the dice shows the number 2.
(a) 36 (b) 91 (c) 81 (d) 116 (e) 124
21. If repetition is not allowed then how many distinct three-digit numbers can be formed using the digits (1, 2, 3, 4, 5)?
(a) 60 ways (b) 50 ways (c) 40 ways (d) 30 ways (e) none
22. Find out the distinct four-letter words that can be formed using the word SINGAPORE.
(a) 256 (b) 1024 (c) 3024 (d) 2048 (e) 124
23. Find out how many distinct three-digit numbers can be formed using the digits 1, 2, 3, 4, 5, 6, 7, 8, 9 such that the digits
are in ascending order.
(a) 80 (b) 81 (c) 83 (d) 84 ` (e) none
24. How many can 3 digits be formed using the digits from 1 to 5 if the digit 2 is never there in the number?
(a) 24 (b) 36 (c) 40 (d) 52 (e) 72
25. If no repetition is not allowed then how many numbers between 2000 and 3000 can be formed using the digits from 0 to
7?
(a) 42 (b) 336 (c) 210 (d) 440 (e) none
26. In how many ways can Kamal choose a consonant and a vowel from the letters of the word ALLAHABAD?
(a) 4 (b) 5 (c) 6 (d) 9 (e) none
27. Find out the number of distinctive words that can be formed using the word GOOD.
(a) 16 (b) 24 (c) 28 (d) 48 (e) 12
28. How many different words can be formed from the alphabets of the word SCISSORS?
(a) 1440 (b) 1680 (c) 1800 (d) 2100 (e) None
29. How many distinct words can be formed using the word MINIMUM?
(a) 420 (b) 450 (c) 1024 (d) 1048 (e) 1028
30. One ticket is selected at random from 50 tickets numbered 0, 01, 02, ……, 49. Then, the probability that the sum of the
digits on the selected ticket is 8, given that the product of these digits is zero equals
(a) 1/14 (b) 1/7 (c) 5/14 (d) 1/50 (e) None
31. It is given that the events A and B are such that P(A) = ¼ , P(A|B) = ½ and P(B|A) = 2/3. Then P(B|A) = 2/3. Then P(B) is
(a) ½ (b) 1/6 (c) 1/3 (d) 2/3 (e) None
132 | P a g
e
32. A die is thrown. Let A be the event that the number obtained is greater than 3. Let B be the event that the number
obtained is less than 5. Then P (AUB) is
(a) 2/5 (b) 3/5 (c) 0 (d) 1 (e) None
33. A pair of fair dice is thrown independently three times. The probability of getting a total of exactly 9 twice is
(a) 1/729 (b) 8/9 (c) 8/729 (d) 8/243 (e) None
34. Two Aero planes I and II bomb a target in succession. The probabilities of I and II scoring a hit correctly are 0.3 and 0.2,
respectively. The second plane will bomb only if the first misses the target. The probability that the target is hit by the second
plane is
(a) 0.06 (b) 0.14 (c) 0.32 (d) 0.7 (e) None
35. A five-digit number is formed using digits 1, 3, 5, 7 and 9 without repeating any one of them. What is the sum of all such
possible numbers?
(a) 6666600 (b) 6666660 (c) 6666666 (d) 777666 (e) None of these
36. A man has 9 friends, 4 boys and 5 girls. In how many ways can he invite them, if there have to be exactly 3 girls in the
invitees?
(a) 320 (b) 160 (c) 80 (d) 200 (e) None of these
37. How many numbers can be formed from 1, 2, 3, 4, 5 (without repetition), when the digit at the unit’s place must be
greater than that in the ten’s place?
(a) 54 (b) 60 (c) 17 (d) 2 × 4! (e) None of these
38. How many numbers can be made with digits 0, 7, 8 which are greater than 0 and less than a million?
(a) 496 (b) 486 (c) 1084 (d) 728 (e) None of these
39. An intelligence agency forms a code of two distinct digits selected from 0, 1, 2, ., 9 such that the first digit of the code is
nonzero. The code, handwritten on a slip, can however potentially create confusion, when read up side down-for example,
the code 91 may appear as 16. How many codes are there for which no such confusion can arise?
(a) 80 (b) 78 (c) 71 (d) 69 (e) None of these
40. 36 identical chairs must be arranged in rows with the same number of chairs in each row. Each row must contain at least
three chairs and there must be at least three rows. A row is parallel to the front of the room. How many different
arrangements are possible?
(a) 2 (b) 4 (c) 5 (d) 6 (e) None of these
133 | P a g
e
PROBABILITY
1. A bag contains 2 yellow, 3 green and 2 blue balls. Two balls are drawn at random. What is the Probability that none of the
balls drawn is blue?
(a) 1/2 (b) 10/21 (c) 9/11 (d) 7/11 (e) 8/11
2. Three coins are tossed. What is the probability of getting at most two tails?
(a) 7/8 (b) 1/8 (c) ½ (d) 1/7 (e) 2/7
3. A bag contains 4 black, 5 yellow and 6 green balls. Three balls are drawn at random from the bag. What is the probability
that all of them are yellow?
(a) 2/91 (b) 1/81 (c) 1/8 (d) 2/81 (e) 1/91
4. In a box, there are 8 red, 7 blue and 6 green balls. One ball is picked up randomly. What is the Probability that it is neither
red nor green?
(a) 1/3 (b) ¾ (c) 7/19 (d) 8/21 (e) 9/21
5. In a class, there are 15 boys and 10 girls. Three students are selected at random. The probability that 1 girl and 2 boys are
selected, is:
(a) 21/46 (b) 25/117 (c) 1/50 (d) 3/25 (e) None of these
6. In a lottery, there are 10 prizes and 25 blanks. A lottery is drawn at random. What is the probability of getting a prize?
(a) 1/10 (b) 2/5 (c) 2/7 (d) 5/7 (e) 3/7
7. From a pack of 52 cards, two cards are drawn together at random. What is the probability of both the cards being kings?
(a) 1/15 (b) 25/57 (c) 35/256 (d) 1/221 (e) 17/225
8. A bag contains 4 white, 5 red and 6 blue balls. Three balls are drawn at random from the bag. The probability that all of
them are red, is:
(a) 1/22 (b) 3/22 (c) 2/91 (d) 2/77 (e) 3/77
9. One card is drawn at random from a pack of 52 cards. What is the probability that the card drawn is a face card (Jack,
Queen and King only)?
(a) 1/13 (b) 3/13 (c) ¼ (d) 9/52 (e) None of these
10. A bag contains 6 black and 8 white balls. One ball is drawn at random. What is the probability that the ball drawn is
white?
(a) ¾ (b) 4/7 (c) 1/8 (d) 3/7 (e) None of these
11. In a box carrying one dozen of oranges, one third have become bad. If 3 oranges are taken out from the box at random,
what is the probability that at least one oranges out of the three oranges picked up is good?
(a) 1/55 (b) 54/55 (c) 45/55 (d) 3/55 (e) None of these
12. A basket contains 5 white and 9 black balls. There is another basket which contains 7 white and 7 black balls. One ball is
to drawn from either of the two baskets. What is the probability of drawing a black ball?
(a) 3/7 (b) 5/7 (c) 4/7 (d) 8/15 (e) None of these
13. A bag contains 5 blue and 4 black balls. Three balls are drawn at random. What is the probability that 2 are blue and 1 is
black?
(a) 1/3 (b) 2/5 (c) 1/6 (d) 1/5 (e) None of these
14. An urn contains 9 red, 7 white and 4 black balls. A ball is drawn at random. What is the probability that the ball drawn is
not red?
(a) 1/11 (b) 9/20 (c) 2/11 (d) 11/20 (e) None of these
134 | P a g
e
15. In a simultaneous throw of two dice, what is the probability of getting a total of 10 or 11?
(a) 7/12 (b) 5/36 (c) 1/6 (d) 1/4 (e) None of these
16. In a single throw of two dice what is the probability of not getting the same number on both the dice?
(a) 1/6 (b) 2/3 (c) 5/6 (d) 1/3 (e) None of these
17. One card is drawn at random from a pack of 52 cards. What is the probability that the card drawn is either a red or a
king?
(a) 6/13 (b) 1/2 (c) 7/13 (d) 27/52 (e) None of these
18. A bag contains 3 red, 5 yellow and 4 green balls. 3 balls are drawn randomly. What is the probability that the balls drawn
contain balls of different colours?
(a) 4/15 (b) 3/11 (c) 1/12 (d) 5/14 (e) None of these
19. A bag contains 5 red, 7 yellow and 6 green balls. 3 balls are drawn randomly. What is the probability that balls drawn
contain exactly 2 green balls?
(a) 14/68 (b) 13/68 (c) 15/91 (d) 15/68 (e) None of these
20. A bag contains 5 red, 6 yellow and 7 green balls. 3 balls are drawn randomly. What is the probability that the balls drawn
contain no red ball?
(a) 55/282 (b) 55/272 (c) 143/408 (d) 143/406 (e) None of these
21. A box contains 4 green, 5 yellow and 4 white marbles. 3 marbles are drawn at random. What is the probability that all
the three marbles are not of same colour?
(a) 9/143 (b) 134/143 (c) 8/143 (d) 135/143 (e) None of these
22. A bag contains 4 red and 7 black balls. Two draws of three balls each are made, the ball being replaced after the first
draw. What is the chance that the balls were red in the first draw and black in the second?
(a) 28/5445 (b) 25/5448 (c) 28/4554 (d) 25/4554 (e) None of these
23. A bag contains 9 red and 7 white balls. Four balls are drawn out one by one and not replaced. What is the probability
that they are alternatively of different colours?
(a) 9/65 (b) 6/65 (c) 9/130 (d) 8/130 (e) None of these
24. A basket contains 5 white and 9 black balls. There is another basket which contains 7 white and 7 black balls. One ball is
to drawn from either of the two baskets. What is the probability of drawing a white ball?
(a) 4/7 (b) 6/7 (c) 3/7 (d) 2/7 (e) None of these
25. 8 persons are seated at a round table. What is the probability that 3 particular persons sit together?
(a) 2/7 (b) 1/7 (c) 3/14 (d) 3/14 (e) None of these
26. Two students Anwar and Aziz are required to solve a problem independently. The respective probabilities of solving the
problem are ¾ and 4/5. What is the probability that
(I) Both will solve the problem.
(II) At most one of them will solve the problem.
(a) 3/5, 2/5 (b) 4/5, 3/5 (c) 2/5, 3/5 (d) None of these
27. The probability that footballs team A wins the match is 4/5 and the probability that it rains on a particular day is 2/3.
What is the probability that it won’t rain and the team A loses the match?
(a) 1/30 (b) 1/15 (c) 2/15 (d) 1/5 (e) None of these
28. A tells the truth in 80% cases and B tells truth in 75% cases. In an argument, what is the probability that they contradict
each other?
(a) 13/20 (b) 7/20 (c) 9/20 (d) 11/20 (e) None of these
135 | P a g
e
29. There are 15 boys and 10 girls in a class. If three students are selected at random, what is the probability that 1 girl and 2
boys are selected?
(a) 1/40 (b) 1/2 (c) 21/46 (d) 7/42 (e) None of these
30. 3 balls are drawn randomly from a bag contains 3 black, 5 red and 4 blue balls. What is the probability that the balls
drawn contain balls of different colors?
(a) 3/11 (b) 1/3 (c) 1/2 (d) 2/11 (e) None of these
31. A card is randomly drawn from a deck of 52 cards. What is the probability getting an Ace or King or Queen?
(a) 3/13 (b) 2/13 (c) 1/13 (d) 1/2 (e) None of these
32. There is a point in a circle. What is the probability that this point is closer to its circumference than to the centre?
(a) 1/2 (b) 1/3 (c) 1/4 (d) 3/4 (e) None of these
33. Raja has 4 tickets of a lottery for which 12 tickets were sold and 3 prices are to be given. What is the probability that Raja
will win at least one price?
(a) 41/55 (b) 43/55 (c) 12/55 (d) 14/55 (e) None of these
34. A basket contains 15 apples and 10 oranges out of which 4 apples and 2 oranges are defective. If a person takes two
fruits at random, what is the probability that either both are apples or both are good?
(a) 221/300 (b) 312/401 (c) 1/2 (d) 1/3 (e) None of these
35. Three boys P, Q and R are to speak at a function along with 5 others. If all of them speak in random order, what is the
probability that P speaks before Q and Q speaks before R?
(a) 2/9 (b) 1/3 (c) 1/6 (d) 1/9 (e) None of these
136 | P a g
e
MEASURES OF CENTRAL TENDENCY
(a) Mean: The mean of the data is equal to the AVERAGE of the data and that can be found by the help of formula
Sum of All the Observations
Average (Mean) =
Number of Observation
(b) Mode: Mode is the value of the observation which occurs most frequently, called mode.
(c) Median: The middle most observation of the data is called median of the data. We can find the median by following
steps:
Step – 1: Arrange the given data in ascending or descending order first.
Step – 2: Count the number of observations.
Step – 3: (Case – 1) If there is an odd amount of numbers, the median value is the number that is in the middle, with
the same amount of numbers below and above.
(Case – 2) If there is an even amount of numbers in the list, the middle pair must be determined, added together,
and divided by two to find the median value.
(d) Variate: The quantity that we measure from observation to observation is called a variate.
(e) Frequency: The number of observations of a particular variation of a class interval is called the frequency of that
variate or interval.
(f) Cumulative Frequency: Cumulative frequency is used to determine the number of observations that lie above (or
below) a particular value in a data set. The cumulative frequency is calculated by adding each frequency from a
frequency distribution table to the sum of its predecessors.
(g) Class Interval: The class interval is a term used in statistics when we are given a continuous series. Class means a
group of numbers in which items are placed such as 0-10, 10-20, 20-30, etc. Class interval refers to the numerical
width of any class in a particular distribution.
Mathematically it is defined as the difference between the upper-class limit and the lower class limit.
Class interval = upper-class limit - lower class limit.
(h) Upper Limit: The right end value of a class is called upper limit of the class. Example: (20 – 30), here 30 is the
upperlimit
(i) Lower Limit: The left end value of a class is called lower limit of the class. Example: (20 – 30), here 20 is the lower.
(j) Class Size: The difference between upper limit and lower limit of a particular class is called class size.
(k) Class Mark: The mid value (average value) of a class interval is class mark of that class.
(m) Range: The range in statistics for a given data set is the difference between the highest and lowest values.
(n) Variance: The Variance is defined as the average of the squared differences from the Mean.
137 | P a g
e
To calculate the variance, follow these steps:
Work out the Mean (the simple average of the numbers)
Then for each number: subtract the Mean and square the result (the squared difference).
Then work out the average of those squared differences.
Example: Mean, Variance and Standard Deviation with the help of an example.
The height of five persons is: 600, 470, 170, 430 and 300 respectively. Find mean, variance and standard deviation.
Solution:
600 + 4 70+ 170 + 4 30+ 300 1970
(1) Mean =
5
= 5
= 394
(p) Quartile Deviation: Quartile deviation is a statistic that measures the deviation in the middle of the data. Quartile
deviation is also referred to as the semi inter quartile range and is half of the difference between the third quartile and
the first quartile value. The formula for quartile deviation of the data is
Q3 – Q1
Q.D =
2
Inter Quartile Range = Q3 – Q1
Q3 – Q1
Quartile Coefficient =
Q3 + Q1
2. Here we have three quartiles Q1, Q2, Q3 which divide the data into three quarters (to form quartiles, divide the whole
series into 4 equal parts). The median of the data has been referred as the second quartile Q2. Also, the first quartile
Q1 is the median of the first half of the data, and the third quartile Q3 is the median of the second half of the data.
Q1 Q2 Q3
Example: Find the quartile deviation and the coefficient of quartile deviation for the following given data.
138 | P a g
e
23, 8, 5, 16, 33, 7, 24, 5, 30, 33, 37, 30, 9, 11, 26, 32
5, 5, 7, 8, 9, 11, 16, 23, 24, 26, 30, 30, 32, 33, 33, 37
From the above data we have Q1 = ( 8 + 9)/2 = 17/2 = 8.5, and Q3 = (30 + 32)/2 = 62/2 = 31
Q3 – Q1 31 – 8.5
Quartile Deviation = = = 11.25
2 2
Q3 – Q1 31 – 8.5
Coefficient of Quartile Deviation = = = 0.57
Q3 + Q1 31 + 8.5
PRACTICE QUESTIONS
1. If the variance of 5 values is 15.6. What is the standard deviation of those values?
(a) 4.35 (b) 3.95 (c) 2.85 (d) 25.65
4. In a moderately asymmetrical series, the values of arithmetic mean and mode are 20.6 and 26.1 respectively. The
value of the median is
(a) 35.3 (b) 28.0 (c) 22.4 (d) 25.1
6. The standard deviation of 10 values is 4. If each value is increased by 3, then find the variance of the new set of values
(a) 49 (b) 16 (c) 19 (d) 7
7. The mean of the runs scored by three batsman A,B and C in the same series of innings are 58, 48 and 147 respectively.
The S.P. of their runs are 15, 12 and 2 respectively. Who is the most consistent batsman?
(a) C (b) B (c) A (d) CBD
8. The median of the following data: 31, 35, 27, 29, 43, 37, 41, 35, 30
(a) 31 (b) 30 (c) 37 (d) 35
139 | P a g
e
9. The annual production (in tones) of two farms during a period of 5 years is given below:
A = 32, 48, 50, 36, 34 B = 25, 40, 35, 75, 25
Which statement is correct?
(a) Both are equal
(b) A is more consistent than B
(c) B is more consistent than A
(d) CBD
11. Find the quartile deviation of 4, 6, 9, 12, 18, 20, 23, 27, 34, 48 and 53?
(a) 12.5 (b) 13 (c) 12 (d) 11.5
12. Consider a data set of following numbers: 22, 12, 14, 7, 18, 16, 11, 15, 12. You are required to calculate the Quartile
Deviation.
(a) 2.5 (b) 2.75 (c) 3 (d) 3.5
13. Harry ltd. is a textile manufacturer and is working upon a reward structure. The management is in discussion to start
a new initiative, but they first want to know how much their production spread is.
The management has collected its average daily production data for the last 10 days per (average) employee.
155, 169, 188, 150, 177, 145, 140, 190, 175, 156.
140 | P a g
e
DATA SUFFICIENCY
2. How many people are standing in a straight line. All are facing North
Statement I. M stands third from the left end of the line. Only one person stands between M and O. P stands second to
the right of O. P stands at one of the extreme lines
Statement II. K stands exactly at the centre of the line. Only two people stand between K and L. Only three people stands
between L and O. Only one persons stands between O and P
A)Only I B)Only II C)Both I and II D)Either I or II E)Neither I or II
4. Among K, L, M, N, O and P each has different age, who is the youngest among them ?
Statement I. L is younger than only K, and P. N is neither the oldest nor the youngest
Statement II. M is older than N but not the oldest
A) Only I B)Only II C)Both I and II D)Either I or II E)Neither I or II
5. How is H related to C?
Statement I. M has two daughters. One of them is Z, who is married to H.
Statement II. C is the mother of Y, the younger sister of Z.
Statement III. M is C’s husband.
A)Only I and II B)Only I and III C)Only I and either II or III
D)Any two of the three E)All are necessary
141 | P a g
e
D)Only I and III E)None of these
9. Arun, Mani, Ravi, Sahil and Mohan are sitting around a circular table facing the centre. Who is on the immediate left
of Ravi?
Statement I. Only Arun is sitting between Mani and Mohan.
Statement II. Arun is on the immediate left of Mani.
Statement III. Sahil is on the immediate left of Mani.
A)All I, II and III together B)Only II C)Only II and III
D)Only I and III E)None of these
Directions (11-20):
(A) If the data in statement I alone is sufficient to answer the question.
(B) If the data in statement II alone is sufficient to answer the question.
(C) If the data either in statement I alone or statement II alone are sufficient to answer the question.
(D) If the data given in both I and II together are not sufficient to answer the question.
(E) If the data in both the statements I and II together are necessary to answer the question.
16. How many persons are sitting between Renu and Ruchi?
Statement I.Renu is 5th from right end of row and also 4th to the right of Anu.
Statement II.Anu is 6th from the left end of row and also 6th to the left of Ruchi.
142 | P a g
e
Statement I.Bhavna is 12th from right to Shruti.
Statement II.Yogesh is 6th to left of Bhavna and also 10th from left end.
20. In how many days Karuna completed her assignment when announced to complete so?
Statement I. Her teacher wanted the assignment to be checked within 10 days when announced.
Statement II. Karuna went to outstation with family the same day and came back before 1 day the day,
assignmentneeded to get checked.
Directions (21-25): Each of the following questions below consists of a question and two statements numbered I and II
given below it. You have to decide whether the data provided in the statements are sufficient to answer the question.
Read both the statements and give answer.
(a) if the data in statement I alone are sufficient to answer the question, while the data in statement II alone are not
sufficient in answer the question.
(b) if the data in statement II alone are sufficient to answer the question, while the data in statement I alone are not
sufficient to answer the question.
(c) if the data in either in statement I alone or in statement II alone are sufficient to answer the question.
(d) if the data in both the statements I and II together are not sufficient to answer the question.
(e) if the data in both the statements I and II are together necessary to answer the question.
Directions (26-27): Each of the questions below consists of a question and three statements numbered I, II and III
given below it. You have to decide whether the data provided in the statements are sufficient to answer the question.
Read both the statements and given answer.
(a) If the data in statement I and II are sufficient to answer the question.
143 | P a g
e
(b) If the data in statement II and III are sufficient to answer the question.
(c) If the data in statement I and III are sufficient to answer the question.
(d) If the data in all the statement I, II and III together are not sufficient to answer the question.
(e) If all the statements I, II and III are necessary to answer the question.
26. Six kids viz. U, V, W, X, Y and Z were born in the same week starting from Monday to Saturday, one on each day. How
many of them are older than W?
Statement I. X is older than at least three kids. U was born on Tuesday.
Statement II. Y is older than at least one of them. W was born immediately after X.
Statement III. At least four persons were born after V.
27. In a certain code language ‘fa ka la ju’ means ‘will black high fly’. Then what is the code of ‘feed black’? If,
Statement I. ‘lu ja ka hu’ means ‘will crow feed us’, ‘
Statement II. ‘ju lu na fu’ means ‘fly of the us’
Statement III. la fu ja ju’ means ‘feed black the fly’
Directions (28-32): Each of the questions below consists of a question and some statements numbered given below it.
You have to decide whether the data provided in the statements are sufficient to answer the question. Read all the
statements and answer the following questions.
(a) If the data in statement I alone or in the statement II alone or in the statement III alone is sufficient to answer the
question.
(b) If the data in statement I and II are sufficient to answer the question, while the data in statement III are not sufficient
to answer the question.
(c) If the data in statement I and III are sufficient to answer the question, while the data in statement II is not sufficient
to answer the question.
(d) If the data in statement II and III are sufficient to answer the question, while the data in statement I is not sufficient
to answer the question.
(e) If the data in all the statement I, II and III are necessary to answer the question.
28. There are 5 boxes V, W, X, Y, and Z of chocolate stored in 5 floor shelves one above the others. In which floor does
box X store?
Statement (I) Box Y is at topmost floor. Box W is just below V but store in even numbered floor.
Statement (II) Box Y does not store in an even number floor.
Statement (III) Box Z does not store in even number floor and Y is above Z.
31. In a certain code language “zebpaybit coin” is written as “AZ12 YO14” and “Ethereum coin” is written as “VN16
XO8”.Than following the same pattern what is the code for “Ripple will increase”?
Statement (I) “Ripple is very popular” is coded as “IF12 RT4 EZ8 KS14”
Statement (II) “Zebpay will increase” is coded as “AZ12 DM8 RF16”
Statement (III) “Rollex watch costly” is coded as “IY12 DI10 XZ12”
32. Six persons A, B, C, D, E and F are sitting in a parallel row facing in the north direction. Who sits between C and E?
Statement (I) A Sits second from the left end. C sits third to the left of B who is near to E.
144 | P a g
e
Statement (II) B is not an immediate neighbor of F who does not sits any extreme end of the row.
Statement (III) E is not the neighbor of A.
Directions (33-35): Each of the questions below consists of a question and two statements numbered I and II given
below it. You have to decide whether the data provided in the statement are sufficient to answer the question. Read
both the statements and
Give answer:
(a) If the data in statement I alone are sufficient to answer the question, while the data in statement II alone are not
sufficient to answer the question.
(b) If the data in statement II alone are sufficient to answer the question, while the data in statement I alone are not
sufficient to answer the question.
(c) If the data either in statement I alone or in statement II alone are sufficient to answer the question.
(d) If the data even in both statements I and II together are not sufficient to answer the question.
(e) If the data in both statement I and II together are necessary to answer the question.
35. M, N, O, P, R, and Q are sitting in a circular table. How many persons are facing opposite to the centre?
Statement I. There is only one person sit between P and Q. R is not an immediate neighbour of Q and sits immediate
left of P.
Statement II. O sits second to the left of R. N is not an immediate neighbour of R, but faces outside the centre. O faces
inside the centre.
145 | P a g
e
SYLLOGISM
DIRECTIONS (1 to 10): In each of the questions below are given few statements are given followed by few
Conclusions. You have to take the given statements to be true even if they seem to be at variance from commonly
known facts. Read all the Conclusions and then decide which of the given Conclusions logically follows from the
given statements disregarding commonly known facts.
1. Statements:
All booklets are packets.
All packets are bottles.
Some bottles are cans.
Some cans are pitchers.
Conclusions:
I. Some pitchers are bottles.
II. Some cans are packets.
III. Some bottles are booklets.
A. None follows B. Only I follows C. Only II follows D. Only III follows E. Only II and III follow
2. Statements:
Some ropes are walls.
Some walls are sticks.
All sticks are chairs.
All chairs are tables.
Conclusions:
I. Some tables are walls.
II. Some chairs are ropes.
III. Some sticks are ropes.
A. None follows B. Only I follows C. Only II follows D. Only III follows E. Only II and III follow
3. Statements:
Some rivers are jungles.
Some jungles are horses.
Some horses are tents.
Some tents are buildings.
Conclusions:
I. Some buildings are horses.
II. Some tents are jungles.
III. Some horses are rivers.
A. None follows B. Only I follows C. Only II follows D. Only III follows E. Only I and II follow
4. Statements:
Some pens are knives.
All knives are pins.
Some pins are needles.
All needles are chains.
Conclusions:
I. Some chains are pins.
II. Some needles are knives.
III. Some pins are pens.
A. Only I follows B. Only II follows C. Only III follows D. Only II and III follow E. None of these
5. Statements:
All fields are ponds.
146 | P a g
e
No ponds is tree.
Some trees are huts.
All huts are goats.
Conclusions:
I. Some goats are fields.
II. No goat is field.
III. Some goats are trees.
A. Only I follows B. Only II follows C. Only III follows D. Only either I or II follows
E. Only either I or II and III follow
6. Statements:
Some doctors are treatments.
All treatments are Indians.
Some Indians are stars.
Conclusions:
I. Some treatments are Indians.
II. Some doctors are Indians.
III. Some doctors are stars.
IV. Some doctors are not stars.
A. I, II and III follow B. I, II and IV follow C. II and III follow
D. I, II and either III or IV follow E. None of these
7. Statements:
Some docks are treats.
All treats are Indians.
Some Indians are stars.
Conclusions:
I. Some treats are Indians.
II. Some docks are Indians.
III. Some docks are stars.
IV. Some docks are not stars.
A. I, II and III follow B. I, II and IV follow C. II and III follow
D. I, II and either III or IV follow E. None of these
8. Statements:
Some bunkers are arms.
No gun is a dagger.
All arms are daggers.
Conclusions:
I. No arm is a gun.
II. Some bunkers are not guns.
III. Some daggers are bunkers.
IV. Some daggers ate not bunkers.
A. I and II follow B. I, II and III follow C. Either III or IV follows
D. I, II and either III or IV follow E. None of these
9. Statements:
All bulbs are radios.
All radios are fans.
No fans are tables.
Conclusions:
I. Some fans are bulbs.
II. No table is a bulb.
147 | P a g
e
III. Some radios are bulbs.
IV. Some tables are radios.
A. I, II and III follow B. II, III and IV follow C. Only I and II follow
D. Only I and III follow E. None of these
10. Statements:
Some charts are dart.
All dart are carts.
Some carts are rigid.
Conclusions:
I. Some charts are carts.
II. Some carts are dart.
III. Some dart are rigid.
IV. Some rigid are charts.
A. Only I and III follow B. Only II and III follow C. I and II follow
D. I, III and IV follow E. None of these
(DIRECTIONS 11 to 15): In each question, a set of six statements is given, followed by five answer choices. Each of
the answer choices has a combination of three statements from the given set of six statements. You are required to
identify the answer choice in which the third statement is logically follows the first two in the same order.
11. Statements:
A. All red is green.
B. All red is white.
C. All red is black.
D. All black is white.
E. All green is yellow.
F. All green is white.
A. ABF B. AEF C. CDB D. CBE E. None of these
12. Statements:
A. All cows are goats.
B. All goats are dogs.
C. No goats are cows.
D. No goats are dogs.
E. All cows are dogs.
F. All dogs are cows.
A. FAB B. ABE C. AFB D. ABF E. None of these
13. Statements:
A. Singers know English.
B. He does not know English.
C. He is a Singer.
D. He is not a Singer.
E. He knows Hindi.
F. He should know English.
A. ABD B. AEF C. DEA D. ACF E. None of these
14. Statements:
A. P is taller than Q.
B. P and Q play golf.
C. R is shorter than Q but taller than S.
D. Golf and cricket are outdoor games.
148 | P a g
e
E. R is the second shortest.
F. All outdoor games require energy.
A. ABC B. ACE C. ADF D. FBD E. None of these
15. Statements:
A. Some sharpeners are pencils.
B. Rubbers are sticky.
C. All sharpeners are rubbers.
D. Sharpeners must be sticky.
E. No pen is a rubber.
F. Some sharpeners are rubbers.
A. ACE B. FCA C. CBD D. ABF E. None of these
(DIRECTIONS 16 to 19): In each of the questions below are given few statements are given followed by few
Conclusions. You have to take the given statements to be true even if they seem to be at variance from commonly
known facts. Read all the Conclusions and then decide which of the given Conclusions logically follows from the
given statements disregarding commonly known facts.
16. Statements:
Some wins are trophies.
Some trophies are cups.
No cup is a prize.
Conclusions:
I. At least some cups are wins.
II. All prizes being trophies is a possibility.
A. If only conclusion I is true B. If only conclusion II is true C. If either conclusion I or II is true
D. If both conclusions are true E. If neither conclusion I nor II is true
17. Statements:
Some baskets are caskets.
Some caskets are trunks.
All trunks are fans.
All sweets are fans.
Conclusions:
I. At least some baskets are trunks is a possibility.
II. At least some fans are caskets is a possibility
III. All fans are baskets is a possibility.
IV. At least some sweets are not caskets is a possibility.
A. Only I and III follow B. Only II and IV follow C. Only I, II and III follow
D. All follow E. None of these
18. Statements:
Some forks are spades.
Some spades are not shovels.
All chisels are shovels.
No potato is a chisel.
Conclusions:
I. Some shovels are not a potatoes is a possibility.
II. At least one chisel is a spade is a possibility.
III. All potatoes are shovels is a possibility.
IV. Some forks are chisels is a possibility.
A. Only III and IV follows B. Only II and IV follows C. Only I, II and IV follows
D. All follow E. None of these
149 | P a g
e
19. Statements:
No bank is a school.
Some schools are colleges.
Conclusions:
I. Some colleges are definitely not schools.
II. All banks being colleges is a possibility.
A. If only conclusion I follows
B. If only conclusion II follows
C. If either conclusion I or conclusion II follows
D. If neither conclusion I nor conclusion II follows
E. If both conclusion I and conclusion II follow
(DIRECTIONS 20 to 25): In each group of questions below are few conclusions followed by set of statements. You
have to choose the correct set of statements that logically satisfies given conclusions.
20. Conclusion:
Some Kids are Royal.
No eraser is a kid.
Statement:
I. Some Kids are erasers. All erasers are Royals.
II. No kid is an eraser. No erasers is Royal.
III. Some Royals are erasers. All Kids are erasers.
IV. All Royals are kid. No kid is an eraser.
V. Some Royals are definitely erasers. No Kid is a Royal.
A. Only Statement I follows
B. Only Statement II follows
C. Only Statement III follows
D. Only Statement IV follows
E. Only Statement V follows
21. Conclusions:
Some 50s are 40s
Some 90s are 40s
Statements:
I. Some 70s are 50s. All 50s are 90s. No 90s is 80s. No 40s is 90s
II. All 70 s are 50s. All 50s are 90s. No 90 is 80s. All 40s are 50s
III. Some 70s are 50s. All 50s are 90s. No 90s is 80s. Some 40s are 80s
IV. All 40s are 50s. All 50s are 90s. No 90s is 80s. Some 80s are 70s.
A. Only statement I follows
B. Only statement II follows
C. Both statement II and statement IV follows
D. Only statement IV follows
E. None of them follow
22. Conclusions:
All Donald being Micky is a possibility.
All Tom being Jerry is a possibility.
Statements:
I. All Donald is Jerry. All Jerry is Micky. No Micky is Tom.
II. Some Donald is Jerry. No Jerry is Micky. Some Micky is Tom.
III. Some Donald is Jerry. Some Jerry is Micky. No Micky is Tom.
IV. All Donald is Jerry. No Jerry is Micky. All Micky is Tom.
150 | P a g
e
V. No Jerry is Tom. Some Donald is Tom. No Micky is Donald.
23. Conclusions:
Some Table is plastic.
Some plastic are bench
Statements:
I. All Table are Chair. All Chairs is bench. Some benches are plastic.
II. All Table is Chair. Some Chair is bench. All benches are plastic.
III. No Table is Chair. All Chairs are bench. Some benches are plastic.
IV. Some Table is Chair. All Chairs are bench. No bench is plastic
V. All Table is Chair. All Chairs are bench. All benches are plastic.
A. Only Statement I follows
B. Only Statement II follows
C. Only Statement III follows
D. Only Statement IV follows
E. Only Statement V follows
24. Conclusions:
Some dates are day.
Some years are day.
Statements:
I. All day are month. All month are year. Some years are date.
II. Some days are month. All month are year. Some days are date.
III. All day are month. Some month is year. All year are date.
IV. All day are month. All month are date. Some years are date.
V. No year is day. Some day is date. Some date is month.
A. Only Statement I follows
B. Only Statement II follows
C. Only Statement III follows
D. Only Statement IV follows
E. Only Statement V follows
25. Conclusions:
Some teachers are not student.
Some lessons are classroom.
Statements:
I. Some classroom is not student. All classrooms are teacher. All teachers are lesson.
II. All classrooms are teachers. Some student is not classroom. Some teacher is lesson.
III. All classrooms are student. No student is teacher. No teachers are lesson.
IV. Some classrooms are student. No student is teacher. Some teachers are lesson.
V. All students are classroom. All classrooms are lessons. All lessons are teachers.
A. Only Statement I follows
B. Only Statement II follows
C. Only Statement III follows
D. Only Statement IV follows
E. Only Statement V follows
151 | P a g
e
Directions (26-30): In each of the following questions two statements are given and these statements are followed
by two conclusions numbered (1) and (2). You have to take the given two statements to be true even if they seem
to be at variance from commonly known facts. Read the conclusions and then decide which of the given conclusions
logically follows from the two given statements, disregarding commonly known facts. Give answer:
28. Statements: Some red are blue. Some green are red.
Conclusions: I. Some green are blue.
II. All green being blue is a possibility
29. Statements: All bronze are gold. Some silver are gold.
Conclusions: I. Some bronze are silver. II. No bronze is a silver
31. Conclusions:
I. No helicopter is a glider.
II. All parachutes being helicopters is a possibility
32. Conclusions:
I. No glider is an airplane.
II. All gliders being helicopters is a possibility.
152 | P a g
e
I. All mails being updates is a possibility.
II. No update is a mail.
(Q. 34-35):
Statements: No stone is metal.
Some metals are papers.
All papers are glass.
34. Conclusions:
I. No glass is metal.
II. At least some glasses are metals.
35. Conclusions:
I. All stones being glass is a possibility.
II. No stone is paper
Directions (36-40): In each question below are two/three Statements followed by two conclusions numbered I and
II. You have to take the two/three given statements to be true even if they seem to be at variance from commonly
known facts and then decide which of the given conclusions logically follows from the given statements
disregarding commonly known facts. Give answers:
For (37-38): Statements: No star is an actor. Some actors are models. All models are heroes.
For (39-40): Statements: All clerks are assistants. No assistant is an officer. All officers are managers.
153 | P a g
e
SEATING ARRANGEMENT
Directions (1-5): Study the information carefully and answer the questions given below.
Eight persons are sitting around a rectangular table such that two persons sit on each side of the table. The ones sitting on the
shorter side of the table face away from the center and the one sitting on the longer side of the table face toward the center. No
two persons sitting opposite to each other are next to each other according to alphabetical series. E is one of the persons. A is
immediate left to G, who sits at the shorter side of the table. H is 2nd right to F. C is immediate left to B, who sits opposite to D.
A sits opposite to one of the immediate neighbor of C.
2. Who among the following 4th to the left to the one who is 2nd left to A?
(a) E (b) F (c) C (d) B (e) none of these
5. How many persons sit between D and H, when counted from the left to H?
(a) Two (b) One (c) Three (d) Four (e) None of these
(Direction 6-10): Read the following information carefully and answer the questions that follow:
Eight persons P, Q, R, S, T, U, V and W are sitting in a straight line facing North direction. They have different number of
chocolates among 12, 17, 18, 15, 26, 34, 30 and 40 but not necessarily in the same order.
R’s chocolate is a multiple of 6 but not a multiple of 5 and sits to the immediate right of Q. One person sits between R and
W who has 26 chocolates. W is not a neighbor of Q. The sum of chocolates of R and P is equal to U. T has more chocolates
than U and less than Q. Two persons sit between W and V. R has more chocolates than S who does not has 12 chocolates.
The number of persons sitting between R and S is same as the number of persons sitting between S and U. The sum of
chocolates of S and the person who sits to the immediate right of S has 49 chocolates.
9. If all the persons are made to sit in the alphabetical order from right end, position of how many persons does not
change?
154 | P a g
e
A) None B) One C) Two D) Three E) Four
(Directions 11 – 15): Read the following information carefully and answer the questions given below it:
I, J, K, L, M, N and O are sitting in a straight line at equidistant from each other but not necessarily in the same order and
some of them are facing south and some are facing north. Each of them likes a different colour viz. Pink, Red, Yellow, Blue,
Black, Brown and Green but not necessarily in the same order.
M likes Yellow colour and sits third from one of the extreme ends of the line. Only two persons sit between M and J who
likes red colour. J does not sit at either of the extreme ends of the line. The one who likes Blue and Pink colour is not an
immediate neighbour of the one who likes Red colour. L likes blue colour and sits third to the right of O who likes Pink
colour. I does not black colour. K does not like brown colour. L does not sit at one of the extreme ends of the line. O faces
south. N neither likes brown nor black. The immediate neighbours of L face the same direction as L. I sits at one of the
extreme ends of the line L sits second to the right of N and both face opposite direction.
Directions (16-20): Study the following information carefully and answer the questions.
10 friends sitting in a restaurant , five of them i.e. Amar, Vimal, Diwan, Raj and Renu are sitting in a row facing
North and the other five friends Seetha, Padma, Manu, Ramu and Tilak are sitting in row facing south. Each friend
in row is facing exactly one from the other row. One of them likes grapes. Seetha sits opposite the friend who likes
a banana and sits at one of the extremes. The friend who likes kiwi who is not Manu sits second to the right of
Seetha.Tilak sits exactly in the middle of Manu and the friend who likes a cherry who is not Seetha.Vimal who
does not sit at extreme end and likes an orange and sits opposite the friend who likes an apple. The one who likes
pineapple sits opposite the friend who is to the immediate left of Vimal. The friend who likes a strawberry who is
notDiwan sits opposite Ramu. Padma does not sit at any of the extremes but sits opposite the friend who likes a
guava, who is adjacent to Amar and the friend who likes a mango. Raj neither likes a guava nor likes a
strawberry.
17. Vimal is how many places away from the one who likes Banana?
A) One B) Three C) Four D) Two E) None of these
18. If Tilak likes Guava, Padma likes Banana then, who will like orange?
A) Manu B) Renu C) Ramu D) Seetha E) Raj
19. Who sits third to the left of the one who likes Guava?
155 | P a g
e
A)Diwan B)Renu C) Amar D) Raj E) Padma
Directions (21-27): Study the following information carefully and answer the questions given below:
There are nine people A, B, C, D, E, F, G, H and I who sit around a circular table. Some are facing towards and some are
facing away from the center. They like different seasons among summer, winter, Rainy, spring and autumn. Not more than
2 people like the same season. The following information is known about them.
C and G form the only pair that is sitting together and like the same season. They are facing in opposite directions to each
other. The one who likes Rainy season sits third to the left of B but is neither C nor F. A sits second to the left of F, who likes
winter. The one who likes summer is an immediate neighbour of H and E but none of them sits adjacent to C or G. A and H
face in the same direction as that of E. I sits third to the right of D, who is facing away from the center and likes spring. E sits
second to the left of C. The persons who like winter face towards the center. A either likes Summer or Spring season. One
who sits second to the right of I likes spring and faces opposite to the center.
Directions (28-34): Study the following information and answer the questions given below:
Eight students – A, B, C, D, E, F, G and H are sitting around a circular table but not necessarily in the same
order. Three of them facing outward while five are facing towards the centre. There are equal number of males and
females in the group. C is facing the centre. E is sitting third to the right of C. F is sitting third to the left of E. Three persons
are sitting between F and B. The immediate neighbours of B are females. G is sitting third to the right of F. D is sitting third
to the right of A. A is not an immediate neighbour of E. The immediate neighbours of E are males and are facing the
centreThe immediate neighbours of D are females and face outside. The one sitting third to the left of B is a male. No
female is an immediate neighbour of G.
156 | P a g
e
29. How many students are sitting between H and C when counted from the left side of H?
A) One B) Two C) Three D) Four E) More than four
For Qs (32-33): Four of the following five are alike in a certain way based on their seating positions in the above
arrangement and hence form a group. Which of the following does not belong to the group?
32.A) BE B) CG C) GA D) DH E) AF
33.A) B B) F C) G D) A E) D
34. If all the friends are asked of sit in an alphabetical order starting from A in an anticlockwise direction, the positions of
how many will remain unchanged (excluding A)?
A) Four B) Three C) Two D) One E) None
(Directions 35 – 39): Study the following information and answer the given questions.
Eight friends A, B, C, D, E, F, G and H are sitting around a circular table facing the centreAll persons are sitting around the
circumference of the circle and has equal distance between them. The radius (r) of the circle is 140m and circumference of
the circle is 2πr (π=22/7). Each of them likes different colour viz. Red, Blue, Yellow, Green, Pink, Black, White and Brown but
not necessarily in the same order.
The one who likes Pink sits 220m away from the right of A There are only two people sit between A and the one who likes
Blue G sits immediate left of H. The one who likes Green sits on the immediate right of C. F sits 220m away from the one
who likes Pink. The one who likes Black sits second to the right of F. C does not like Yellow. The one who likes White sits
220m way from the right of B. A does not like Red The one who likes red is neither an immediate neighbour of the one who
likes black nor the one who likes pink. E sits 440m away from B.E sits second to the left of the one who likes yellow. The one
who likes yellow sits 110m away from the one who likes red C is neither an immediate neighbour of the one who likes Blue
nor the one who likes Yellow.
36.How many persons sit between A and the one who likes red colour?
A) None B) One C) Two D) Three E) Four
37.What is the distance between C and the one who likes yellow colour?
A) 110m B) 330m C) 550m D) 440m E) 660m
157 | P a g
e
A) E
B) The one who likes yellow colour
C) A
D) The one who likes red colour
E) The one who likes black colour
(Directions 40 – 44): Study the following information and answer the given questions.
Six persons – M, N, O, P, Q and R are sitting in a equilateral triangular table Three of them sit at the corner of the triangular
table and three of them sits at the middle of the side Some of them facing the centre and some of them facing outward of
the table Each of them has different number of chocolates viz. 2, 4, 15, 20, 300 and 400 but not necessarily in the same
order.
M is an immediate neighbour of Q. The one who has 4 chocolates sits at the corner of the tableOnly one person sit
between O and R and both are facing same directions. Q and N are facing opposite directions. Only one person sit between
the one who has 2 chocolates and the one who has 15 chocolates and both are facing same direction. Product of chocolates
of M and R is equal to Q’s chocolates. Q sits third to the right of R. R’s chocolates is five times of P’s chocolates. Q’s
chocolates is twenty times of M’s chocolates. The one who has 2 chocolates is facing opposite direction of the one who has
400 chocolates. The one who has 15 chocolates sits immediate left of P. O sits third to the left of the one who has 4
chocolates and both are facing opposite direction. Q faces centre
Directions (45-49): Study the information carefully and answer the questions given below.
Eleven persons are sitting in a row. They all are facing either north or south. B and D are left to I but to the right of F. J sits
at the leftmost end G sits 4th to the right of J. Two persons sit between G and I. The number of persons sitting between B
and I is same as between B and D. K is immediate right to A, who does not face north. At least two persons sit between C
and B.The number of persons sitting between E and H is same between H and G. F is 3rd from one of the ends. E is
immediate right to I.
45. How many persons are sitting to the right of A?
A) 2 B) 3 C) 4 D) 1 E) None of these
158 | P a g
e
47. What is the position of C with respect to D?
A) 3rd to the left B) 3rd to the right C) 4th to the left D) 5th to the right E) None of these
50. On a particular Saturday, a student will perform six activities—grocery shopping, hedge trimming, jogging, kitchen
cleaning, laundry, and motorbike servicing. Each activity will be performed once, one at a time The order in which the
activities are performed is subject to the following conditions:
Grocery shopping has to be immediately after hedge trimming.
Kitchen cleaning has to be earlier than grocery shopping.
Motorbike servicing has to be earlier than laundry.
Motorbike servicing has to be either immediately before or immediately after jogging.
159 | P a g
e
PUZZLE
Directions (1-5): Study the following information carefully and answer the questions given below :
Seven persons A, B, C, D, E, F and G are going to attend a seminar, but not necessarily in the same order, in seven different
months, viz January, February, March, June, August, October and December. Each of them also likes a different colour, viz
Red, Green, Blue, Black, Yellow, white, and Pink but not necessarily in the same order.
C. attends the seminar in the month which has less than 31 days. Only two persons attend the seminar between the
months in which C and D attend the seminar. The one who likes Red attends the seminar immediately before E. Only one
person attends the seminar before the one who likes Blue. B attends the seminar immediately after the one who like Blue.
Only three persons attend the seminar between B and the one who likes Yellow. E likes neither Yellow nor Blue. A attends
the seminar immediately before E. G likes Pink. The one who likes Green attends the seminar in a month which has less
than 31 days. The one who attends the seminar in March does not like a Black colour.
1) How many persons attend seminar between the months in which G and A attend a seminar?
A) None B) Two C) Three
D) One E) More than three
2) who among the following attend the seminar in January and June respectively?
A) F, C B) E, D C) G, C
D) B, F E) A, C
5) As per the given arrangement if “C” is related to ‘Red” and “A” is related to “Black” then, following the same pattern,
which of the following is “G” related to?
A) Blue B) white C) Yellow
D) Green E) Pink
Directions (6-10): Study the following information carefully and answer the questions given below:
P, Q, R, S, T, U and V are seven teachers in a school. All of them belong to different cities, viz, Delhi, Mumbai, Chandigarh,
Pune, Bengaluru, Kolkata and Ahmedabad but not necessarily in the same order. Each of them teaches a different subject,
viz Physics, Chemistry, Maths, English, Commerce, Sanskrit, and Hindi but not necessarily in the same order.Q. belongs to
Pune and does not teach either Commerce or English. R teaches Physics and does not belong to Chandigarh or Kolkata. T
belongs to Bengaluru and teaches Sanskrit. The one who teaches Chemistry belongs to Ahmedabad. S belongs to Delhi. V
teaches Hindi. P does not belong to Ahmedabad. The one who teaches English belongs to Chandigarh.
160 | P a g
e
A) Maths B) Commerce C) English
D) Chemistry E) None
Directions (11-15): Study the following information carefully and answer the questions given below:
Six People – A, C, D, E, F, and G live in eight different floors of building (but not necessarily in the same order). Two of the
floors in the building are vacant. The lowermost floor of the building is numbered one, the one above that is numbered
two, and so on till the topmost floor is numbered eight. Each one of them also likes a different brands of perfumes,
namely Adidas, Axe, Fogg, Gillette, Nivea and NIKE (but not necessarily in the same order).
The number of floors above F is same as the number of floors between F and D. F lives an odd numbered floor above the
floor numbered four. There are three people live between the two vacant floors. Only three floors between D and the one
who likes Gillette. The one who likes Nivea lives immediately above G. Only three floors between G and A. The one who
likes NIKE lives immediately above the one who likes Adidas. C lives immediately above the one who likes Fogg. The
number of floors between F and the one who likes AXE is only one. The one who likes AXE perfume immediately below the
one of the vacant floors. The floor number of the vacant floors are even – number. Only two floors are there between the
one who likes NIKE and Gillette. C lived immediately below the one of the vacant floors and not on the ground floor. C
neither lived on floor number 5 nor floor number 3. The one who likes Axe live on one of the floors below the floor
number 4.
11. Which of the following Statements is true with respect to the given information?
A. G lives immediately above the one who likes Gillette
B. E lives immediately above C
C. Only three people live between F and the one who likes NIKE.
D. D likes Fogg.
E. All the given statements are true.
12. Who amongst the following lives exactly between the fourth floor and the one who likes Adidas Perfume?
A. B, C B. G, D C. F, G D. A, B E. E, A
14. Four of the following five are alike in a certain way and so form a group. Which one of the following does not
belong to the group?
A. G – Nivea B. C – Gillette C. F – Nivea D. E – Adidas E. D – AXE
161 | P a g
e
A. Four B. Three C. None D. Five E. Two
Directions (16-20): Study the following information carefully and answer the questions given below:
Ten students namely viz P, Q, R, S, T, U, V, W, X and Y of ten different colleges but not necessarily in the same order have
exam on five different days starting from Monday to Friday of the same week. Each student have exam at two different
time slots, i.e 08.00 AM or 11.00 A.M
Only two people have exam between U and Y. Neither T nor V does not have exam on Friday. X has exam on Tuesday at
08.00 A.M. W does not have exam at 11.00 AM. The number of people who have exam between V and S is same as the
number of people who have exam between R and W. S does not have exam on any one of the days after T. U does not
have exam on any of the days after W. Q has exam immediately before X. X does not have exam on any of the days before
V. The one who has exam at 08.00 A.M. immediately before Y. S has exam immediately after the day of one who has exam
on Monday. U does not have exam at 11.00 A.M. Only three people have exam between V and T.
16. How many persons have exam at 11’0 clock between T and W?
A. 5 B. 6 C. 2
D. 4 E. None of these.
18. Four among the following form a group in a certain way. Which of the following does not belong to Group?
A. Q – Tuesday B. S – Wednesday C. V – Tuesday
D. P – Friday E. W – Friday
Directions (21-25): Study the following information carefully and answer the questions given below:
Seven friends M, N, O, P, Q, R and S going for different classes, namely Singing, Drawing, Karate, Dancing, Abacus,
Swimming and Skating, but not necessarily in the same order, from Monday to Sunday .
Q attends a class on Friday. Only two persons going for classes between Q and the one who is learning Dancing. O going to
class immediately before P. Neither O nor P is learning Dancing. Only one person attends a class between O and the one
who is learning Karate. The one who is learning Karate does not have a class on Monday.
N attends a class immediately before the one who is learning Singing. Q is not going for Singing. Only one person has a
class between O and M. S is going to class for swimming at the weekends. Only two persons have classes between S and
the one who is learning Abacus. M is not going for Skating.
162 | P a g
e
23. As per the given arrangement, P is related to Friday, M is related to Tuesday, then Which of the following is related to
O?
A. Monday B. Saturday C. Friday
D. Thursday E. None of these
25. Four of the following five form a group as per the given arrangement. Which of the following does not belong to that
group?
A. S- Sunday B. Q-Karate C. P-Thursday
D. Monday – Drawing E..Saturday -M
Directions (26-30): Study the following information carefully and answer the questions given below:
Five friends Agil, Arun, Ashwin, Gopal and Hani are working in 5 different Companies Polaris, CTS, Infosys, TCS and IBM and
they earn different salaries Rs 9000, Rs 12000, Rs 15000, Rs 18000 and Rs 21000 and they all are of different ages i.e. 26,
27, 28, 29 and 30years not necessarily in same order. Person who is 29 year old earns highest salary but does not work at
Polaris and CTS. Hani does not work at TCS and CTS and his age is not 30. The salary of Gopal is less than Rs 15000 and not
working in IBM. The person who is 27 years old earns Rs 18000 per month. Agil earns Rs 12000 per month but does not
work at Infosys and TCS. Arun works at Polaris and earns more than Rs 15000. Person who is 28 years old works at IBM.30
years old person earns at least Rs 15000.
29. Acccording to the given information Gopal is related to 9000, Arun is related to 18000, in the same way Ashwin is
related to
A. 12000 B. 15000 C. 21000
D. CNB E. None of these
Directions (31-35): Study the following information carefully and answer the questions given below:
A, B, C, D, E, F and G are seven people live on seven different floors of a building but not necessarily in the same order. The
lower most floor of the building is numbered 1, the one above that is numbered 2 and so on till the topmost floor is
numbered 7. Each one of them earn different amount per month. i.e. 20000, 15000, 10000, 25000,30000, 35000 and
40000. (But not necessarily in the same order.)
The one who earn Rs.20000 lives immediately above the one who earns Rs.40000. Only one person lives between B and E.
B lives on one of the floors above E. Neither C nor A earns Rs.25000. E does not earn Rs. 10000. A lives on an odd
numbered floor but not on the floor numbered three. The one who earns Rs.30000 lives immediately above A. Only two
people live between A and the one who earns Rs.10000. The one who earns Rs.15000 lives on one of the odd numbered
163 | P a g
e
floors above D. Only three people live between C and the one who earns 15000. The one who earn Rs.10000 lives
immediately above C. F earns 10000.
32. Which of the following combination is true as per the given arrangement?
A. A – 15000 B. C – 30000 C. C. B – 35000 D.
D. F – 20000 E. None of these.
34. Four among the following form a group in a certain way. Which of the following does not belong to Group ?
A. G – 15000 B. A – 40000 C. B – 20000
D. F – 40000 E. E – 10000
Directions (36-40): Study the following information carefully and answer the questions given below:
Seven students namely viz A, B, C, D, E, F and G of seven different colleges have seminar on seven different days, namely
viz Monday, Tuesday, Wednesday, Thursday, Friday, Saturday and Sunday of the same week but necessarily in the same
order. Each student stays in a hostel in different floor. The lower most floor of the building is numbered 1, the one above
that is numbered 2 and so on till the topmost floor is numbered 7.
G stayed in the second floor and has a seminar on Wednesday. The one who stayed in the first floor has a seminar on
Saturday. B has a seminar immediately before E. B does not have seminar on any of the days after G. The one who stayed
in the seventh floor does not have a seminar on any of the days on or before Friday. The one who stayed in the third floor
has a seminar immediately after C. E does not stay in the fifth floor. The one who stays in sixth floor does not have a
seminar immediately before or after G. F does not have any seminar on Sunday and does not live in third floor. D does not
have seminar on any one of the days before A.
38. Four among the following form a group in a certain way. Which of the following does not belong to Group ?
A. D – Monday B. C – Tuesday C. A – Wednesday D. G – Saturday E. E – Sunday
Directions (41-45): Study the following information carefully and answer the questions given below:
Seven different food boxes P, Q, R, S, T, U and V contain seven different dishes namely viz., Mutton Briyani, Sandwich, Veg
Fried Rice, Pizza, Burger, Egg Noodles and Chicken Fried Rice are arranged one above the other. The box at the bottom of
164 | P a g
e
arrangement is numbered 1, the above box is numbered 2 and so on. Box V does not contain Mutton Briyani. Q is
immediately above T. The box which contains the Pizza is immediately below P. Only one box is between the box that
contain Mutton Briyani and U. V is immediately above the box that contain Egg Noodles. More than two boxes are above
the box that contains Mutton Briyani. Only two boxes are between the box that contain sandwich and the box that
contains Chicken Fried Rice. Only two boxes are between the box which contains Pizza and the box which contains Mutton
Briyani. The box filled with Veg Fried Rice is neither at the top nor at the bottom of the arrangement. Only one box is
between Q and the box which contain Sandwich. Q is placed above the Sandwich box. R is placed immediately above U.
Neither R nor V contains Pizza.
41. As per the given arrangement, S is related to Burger and U is related to Egg Noodles in a certain way. To which of the
following is V related to the same way?
A. Sandwich B. Egg Noodles C. Pizza D. Veg Fried Rice E. None of the Above
42. Which of the following pairs of people occupy the top, middle and bottom positions of the arrangement?
A. R, P, T B. R, T, U C. P, T, V D.Q, U, S E. R, T, S
165 | P a g
e
SELECTION
Directions (1-5): Answer the questions on the basis of the information given below :
A,B,C,D,E,F,G,H,I and J are the only ten members in a department. There is a proposal to form a team from within
the members of the department, subject to the following conditions:
1. A team must include exactly one among E, G and H.
2. A team must include either C or F, but not both.
3. B and D cannot be members of the same team.
4. B and I cannot be members of the same team.
5. If a team includes A, then it must also include B and vice-versa.
6. If a team includes one among H, I and J, then it must also include the other two.
The size of a team is defined as the number of members in the team.
A. 2 or 3 B. 2 or 4 C. 3 or 4 D. Only 2 E. Only 4
3: In how many ways a team can be constituted so that the team includes D?
A. 2 B. 3 C. 4 D. 5 E. 6
A. A B. B C. C D. E E. G
Directions (6-9): Answer the questions on the basis of the information given below.
Prof. Biswas has been tracking the number of visitors to his homepage. His service provider has provided him with
the following data on the country of origin of the visitors and the museum they belong to:
Number of visitors
166 | P a g
e
6: Museum 1 can belong to
A. Australia B. Poland C. Pakistan D. Germany
8: Visitors from how many museums from Germany visited Prof. Biswas’s homepage in the three days?
A. 1 B. 2 C. 3 D. 4
9: Which among the listed countries can possibly host three of the eight listed museum?
A. None B. Both China and Germany C. Only Germany D. Only China
Directions for questions no. (10-14): A circular field, with inner radius of 10 meters and outer radius of 20
meters, was divided into five successive stages for ploughing. The ploughing of each stages was handed
over to a different farmer.
1. Farmers are referred to by following symbols: S1, S2, S3, S4, S5.
2. The points between different stages of project are referred to by the following symbols: A1, A2, A3, A4, A5, not
necessarily in the same order.
3. Farmer S4 was given the work of the fourth stage.
4. Stage 3 finished at point A1, and the work of which was not given to farmer S1.
5. Farmer S3 was given work of stage ending at point A5.
6. Farmer S5 was given the work of ploughing stage starting at point A4.
7. The stage from point A5 to point A3 was not the first stage.
10: Which were the starting and the finish points of stage 2?
A. A2 and A5 B. A5 and A3 C. A3 and A1 D. A5 and A4 E. A3 and A2
Directions for questions no. (15-17): Four married couples competed in a singing competition. Each couple had
a unique team name. Points scored by the teams were 2, 4, 6 and 8. The “Suhani couple” won 2 points. The
“Hadippa Singers” won two more points than Ram’s team. Munna’s team won four points more than Lavina’s team,
but Lavina’s team didn’t score the least amount of points. “Just Singing” won 6 points. Wadia wasn’t on the team
called “Old singers”. Sanjay’s team won 4 points. Simran wasn’t on the “Hadippa Singers” team. Tarun and Soniya
were on the same team, but it wasn’t the “Suhani couple”.
167 | P a g
e
16: The teams arranged in the ascending order of points are:
A. Hadippa Singers, Just Singing, Old singers, Suhani couple
B. Suhani couple, Old singers, Just Singing, Hadippa Singers
C. Old singers, Suhani couple, Hadippa Singers, Just Singing
D. Suhani couple, Hadippa Singers, Just Singing, Old singers
E. Just Singing, Hadippa Singers, Suhani couple, Old singers
17: The combination which has the couples rightly paired is:
A. Munna, Lavina B. Munna, Wadia C. Sanjay, Simran
D. Sanjay, Lavina E. Sanjay, Wadia
Question No. 18-20 are based on a set of conditions. In answering some of the questions, it may be useful to
draw a rough diagram. Choose the response that most accurately and completely answers each question.
In a local pet store, seven puppies wait to be introduced to their new owners. The puppies, named Ashi, Burma,
Cumin, Darton, Emeri, Faza, and Galzar, are all kept in two available pens. Pen 1 holds three puppies, and pen 2
holds four puppies.
If Galzar is kept in pen 1, then Darton is not kept in pen 2.
If Darton is not kept in pen 2, then Galzar is kept in pen 1. If Ashi is kept in pen 2, then Burma is not kept in pen 2. If
Burma is kept in pen 1, then Ashi is not kept in pen 1.
18 : If Emeri and Faza are in different pens, then which of the following must NOT be true?
A. Faza shares a pen with Cumin.
B. Galzar shares a pen with Ashi.
C. Emeri is in a higher-numbered pen than Burma.
D. Burma shares pen 2 with Emeri and Darton.
E. Cumin is in a higher-numbered pen than Faza.
20 : If Emeri shares a pen with Faza, then which of the following MUST be true?
A. Galzar is in pen 1 with Darton.
B. Cumin is in pen 2.
C. Burma is in pen 2 and Faza is in pen 1.
D. Emeri is in pen 1.
E. Galzar snares a pen with Burma.
Directions (Qs. 21-23): Amily Company has its office at the Fourth floor of a multi-storied building in Delhi. There
are 5 rooms to be allotted to 5 managers (designated N1 to N5), each of whom will occupy one room. Each room
has its own advantages and disadvantages. Some have the sea view, while others are closer to either the lift or the
dining room, while some are more spacious. Each of the five managers was asked to rank the room preferences
amongst the rooms 401, 402, 403, 404 and 405. Their preferences were recorded as follows:
168 | P a g
e
Some managers did not indicate any preference for some rooms, as they did want to be there under any condition.
The company decided to allot rooms to managers in such a way that so that the managers get rooms as per their
best preference or close to that.
21 : If manager X gets his/her 1st choice, then his / her preference ranking is 1 and so on. Management decided
to allot rooms so that the sum of the preference rankings of all the managers is minimized, what is the total
preference ranking for the rooms allotted to all the managers?
A. 5 B. 6 C. 7 D. 8 E. 9
22 : How many managers would get their rooms as per their best preference?
A. 1 B. 2 C. 3 D. 4 E. 5
23 : Suppose that Manager N2 decides not to join the new zonal office and Manager N6 takes his place. Manager N6
has the following preference ranking in decreasing order: 401,403,402,404 – in this case what would be the sum of
the preference rankings allotted to all the five managers?
A. 5 B. 6 C. 7 D. 8 E. 9
Directions for questions 24 to 28: Read the following information and Tables and answer the questions.
Ahmedabad, Jaipur, Bangalore, Indore, Mumbai and Lucknow are 6 major Indian cities. For some reason people use
only a certain mode of transport between a pair of cities. The modes of transport are provided in Table 1, while in
Table 2 the distances between different pairs of cities are given. Table 3 provides the speed of the mode of
transport and the cost associated with each of them.
Table 1: Mode of Transport between Cities
24 : Mr. Ranjith lives in Mumbai and wants to travel to Indore. However, the train services are on halt due to
laying of track for bullet trains across the country. In this scenario, which of the following is the least cost route to
169 | P a g
e
reach Indore?
A. Mumbai-Ahmedabad-Indore
B. Mumbai – Jaipur – Indore
C. Mumbai – Bangalore – Indore
D. Mumbai – Lucknow- Indore
27 : A school in Jaipur is planning for an excursion tour for its students. They want to show them Bangalore,
Lucknow, and Ahmedabad, not necessarily in the same order. What is the minimum travel cost (in Rs.) the school
should charge from each of the student for the entire tour?
A. Rs. 4300 B. Rs. 5000 C. Rs. 7500 D. Rs. 6800
28 : Which of the following cities can be reached from Ahmedabad in least time?
A. Jaipur B. Bangalore C. Mumbai D. Lucknow
Directions for questions 29 to 32: During a four-week period each one of seven products – P,Q,R,S,T,U and V will
be advertised,. A different pair of these products will be advertised each week. Exactly one of the products will be a
members of two of these four pairs. None of the other products gets repeated in any pair.
Further, the following constraints must be observed: R is not advertised during a given week unless Q is advertised
during the immediately preceding week. The product that is advertised twice is advertised during week 4 but is not
advertised during week 3. P is not advertised during a given week unless either R or V is also advertised that week.
S is advertised during one of the first two weeks. V is one of the products advertised during week 3.
29 : If T is the product that is advertised during two of the weeks, which one of the following is a product that
MUST be advertised during one of the weeks in which T is advertised?
A. P B. Q C. U D. S E. R
30: Which one of the following could be the schedule of the advertisements?
A. Week 1: P, R; week 2 : S, T; week 3: V,U; week 4: Q, T
B. Week 1: Q, S; week 2: R, P; week 3: V, T;week 4: U, S
C. Week 1: Q, S; week 2: R, U; week 3: V, T;week 4: P, U
D. Week 1: Q, T; week 2: R, U; week 3: V, P;week 4: S, T
E. Week 1: S, U; week 2: Q, R; week 3: V, P;week 4: T, U
31 : Which one of the following is a pair of products that could be advertisement during the same week?
A. U and V B. P and Q C. Q and R D. Q and V E. S and V
32 : Which one of the following is a product that could be advertised in any of the four weeks?
A. Q B. T C. S D. R E. V
170 | P a g
e
NUMBER PUZZLE
Direction for the questions (1 to 10): Which of the following option will replace the question mark?
1.
A. 5 B. 12 C. 13 D. 26 E. None of these
3.
A. F B. M C. H D. G E. None of these
4.
A. 10 B. 15 C. 9 D. 8 E. None of these
5.
1 2 3
4 5 6
7 8 9
27 38 ?
A. 49 B. 50 C. 51 D. 52 E. None of these
6.
18 24 32
12 14 16
3 ? 4
72 112 128
A. 2 B. 3 C. 4 D. 5 E. None of these
7.
24 144 384
6 36 ?
2 12 32
1 6 16
A. 80 B. 85 C. 91 D. 96 E. None of these
171 | P a g
e
8.
12 (47) 21
10 (52) 4
64 (?) 24
A. 16 B. 40 C. 62 D. 83 E. None of these
9.
2 2 256
3 2 ?
4 2 46656
A. 2765 B. 3125 C. 8796 D. 30008 E. None of these
10.
31 17 58 87
68 19 61 56
91 22 70 50
10 142 11 ?
A. 3 B. 6 C. 7 D. 9 E. None of these
11. In each of the following questions, a matrix of certain characters is given. These characters follow a certain
trend, row wise or column wise. Find out this trend and choose the missing character accordingly.
12. In each of the following questions, a matrix of certain characters is given. These characters follow a certain
trend, row wise or column wise. Find out this trend and choose the missing character accordingly.
14. In each of the following questions, a matrix of certain characters is given. These characters follow a certain
trend, row wise or column wise. Find out this trend and choose the missing character accordingly.
15. In each of the following questions, a matrix of certain characters is given. These characters follow a certain
trend, row wise or column wise. Find out this trend and choose the missing character accordingly.
172 | P a g
e
(A) 25 (B) 27 (C) 32 (D) 37
16. In each of the following questions, a matrix of certain characters is given. These characters follow a certain
trend, row wise or column wise. Find out this trend and choose the missing character accordingly.
17. In each of the following questions, a matrix of certain characters is given. These characters follow a certain
trend, row wise or column wise. Find out this trend and choose the missing character accordingly.
19. In each of the following questions, a matrix of certain characters is given. These characters follow a certain
trend, row wise or column wise. Find out this trend and choose the missing character accordingly.
20. In each of the following questions, a matrix of certain characters is given. These characters follow a certain
trend, row wise or column wise. Find out this trend and choose the missing character accordingly.
173 | P a g
e
REVISION
1. You must either (a)/ be regular with your studies (b)/ and study for longer period before the examination. (c)/ No error
(d)
2. The new taxation rates (a)/ announced by the government (b)/are bound to effect the export sector. (c)/ No error (d)
3. These days, job opportunities are not as better (a)/ as they used to be (b)/ in the early 70's. (c)/ No error (d)
4. When viewed with his point of view, the (a) / entire episode assumes (b)/ a different colour altogether. (c)/ No error (d)
5. On many occasions (a)/ we did helped the poor (b)/ people by way of giving them food to eat and clothes to put on. (c)/
No error (d)
6. Unless it is accepted to both the parties, an (a)/ arbitrator would be of no (b)/ use to settle this dispute. (c)/ No error (d)
7. Although the manager was keen on getting the work (a)/ done through Sudhir yesterday, (b)/ he tries to avoid it (c)/ No
error (d)
8. The various consequences of(a)/ the decision taken by the (b)/ finance ministry was not foreseen by the bureaucrats.
(c)/ No error (d)
9. I never considered him to be a person who would (a)/ go back on his promise and (b)/ then do not even apologise. (c)/
No error (d)
10. Having finished at school (a)/Raghu thought/ of going to Bombay in (b)/ search some job. (c)/ No error (d)
11. When shall we (a)/ arrive (b)/to our destination ? (c)/ No error. (d)
12. Based on the newspaper reports(a)/ we can conclude that (b)/many accidents caused by reckless driving.(c)/ No error.
(d)
13. Females (a)/ are not appointed (b)/ in our college. (c)/ No error. (d)
14. The officer (a)/ is angry on the clerk (b)/ for not attending to the work. (c)/ No error. (d)
15. No sooner (a)/ I had spoken, (b) than he left. (c)/ No error. (d)
16. Computer education (a)/ in universities and colleges (b) leaves much to be desired. (c)/ No error. (d)
17. You will be prosecuted (a)/ for bringing seeds (b)/ into Australia. (c)/ No error. (d)
18. You must either tell me ( 1) / the whole story or, at least (b)/ the first half of it. (c)/ No error. (d)
19. Our new neighbours (a)/ had been living in Arizona (b)/ since ten years before moving to their present house. (c)/ No
error. (d)
20. The patient (a)/ was accompanied (b)/ with his friend. (c)/ No error. (d)
21. A city dweller finds it difficult (a)/to pass away the time (b)/ in a village. (c)/ No error. (d)
22. Visitors (a)/ were not permitted (b)/ entering the park (c)/ after dark. No error (d)
23. The fifth and final act (a)/ of Macbeth contain (b)/ the sleep- walking scene. (c)/ No error. (d)
24. One of the terrorists (a)/ of the Kashmir valley (b)/ are shot dead. (c)/ No error (d)
25. Ten kilometres (a)/ is (b)/ a long distance to walk. (c) / No error (d)
26. I saw him (a)/a couple of times (b)/ since May(c). /No error (d)
27. Tea (a)/which I am drinking (b)/is hot (c). /No error (d)
28. Although the police officer sympathised with poor (a)/ he refused to (b) /take an action against the rich man (c). /No
error (d)
29. We were looking forward (a)/ to hear news (b)/about the missing fishermen (c)./ No error (d)
30. The actress (a)/was shocked (b)/by the news of her dog’s death (c). /No error (d)
31. One of the questions (a)/ he asked me was ( 2) / “Who did you travel with (c) ?” /No error (d)
32. I know (a)/a doctor (b)/you are referring to (c)./No error (d)
33. The introduction of job-oriented courses (a)/in the self-financing colleges (b)/ attract many students (c)./ No error (d)
34. It is better (a)/to keep one’s head in the face of danger than (b)/losing one’s courage (c). /No error (d)
35. The short story (a)/should not exceed (b)/more than two hundred words (c)./No error (d)
36. To die with honour (a)/is betterthan (b)/live with dishonour.(c)/ No error. (d)
37. It is I (a)/who is to blame.(b)/for this bad situation(c)./ No error.(d)
174 | P a g
e
38. Gowri told me(a)/his name after(b)/he left. (c)/ No error. (d)
39. John would have told (a)/you the truth (b)/ if you had asked him. (c)/ No error. (d)
40. My sister (a)/has read (b)/pages after pages of the Bible. (c)/ No error. (d)
41. Your success in the IAS examinations depends not only on (a)/ what papers you have selected (b)/but on how you have
written them. (c) No error.(d)
42. Heavy rain (a)/prevented us (b)/ to go to the cinema. (c)/No error. (d)
43. If majority of the individuals in a State (a)/prosper (b)/the State itself would prosper. (c) /No error(d)
44. If motorists do not observe the traffic regulations (a)/they will be stopped, ticketed (b)/and have to pay a fine. (c)/ No
error. (d)
45. He asked (a)/supposing if he fails (b)/ what he would do. (c)/ No error. (d)
46. We had a lot of difficulty (a)/to find (b)/the house. (c)/ No error. (d).
47. Patience as well as perseverance (a)/are necessary (b)/ for success. (c)/ No error/(d).
48. The passer-by told us (a)/where was the marriage hall (b)/and even led us to it. (c)/ No error/(d)
49. The increase in consumption is directly (a)/proportional to the increase (b)/in income. (c)/No error. (d).
50. In Singapore (a)/my brother-in-law with his wife (b)/were present at the function. (c)/No error. (d).
175 | P a g
e
PREPOSITIONS
1. (a) He took/ (b) leave of/ (c) four days/ (d) No error.
2. (a) Children/ should always/ (b) listen the advice of their elders/ (c) and well wishers./ (d) No error.
3. (a) He will not/ (b) listen/ (c) what you say./ (d) No error.
4. (a) Nobody denies/ (b) that my ideas/ (c) are different than yours./ (d) No error.
5. (a) It was I who was responsible of/ (b) making all the arrangements for the/ (c) successful completion of his studies./
(d) No error.
6. (a) She was in the courtyard/ (b) when the burglars/ (c) entered into her house./ (d) No error.
7. (a) Our teacher/ (b) emphasized on/ (c) the use of correct grammar./ (d) No error.
8. (a) On the time/ (b) of the opening ceremony of the theatre/ (c) a large crowd had assembled/ (d) No error.
9. (a) While they were returning/ (b) from school,/ (c) a stalker attacked on them with a knife./ (d) No error.
10. (a) The decline of his moral values/ (b) has caused a lot/ (c) of pain to his parents/ / (d) No error.
11. (a) Without thinking/ (b) for a moment he/ (c) entrusted me in all the responsibilities. / (d) No error.
12. (a) Sudha fell in/ (b) the well and nobody / (c) tried to save him./ (d) No error.
13. (a) I / (b) prefer coffee/ (c) than tea/ (d) No error.
14. (a) Suresh is busy / (b) in his work/ (c) for his presentation/ (d) No error.
15. (a) While crossing the road/ (b) an old man was/ (c) run out by a bus/ (d) No error.
16. (a) Amphibians / (b) can live / (c) in water as well as land./ (d) No error.
17. (a) My mother is fond off/ (b) cooking different / (c) types of dishes/ (d) No error.
18. (a) Our teacher/ (b) cannot/ (c) control on the students./ (d) No error.
19. (a) The lawyer has been waiting/ (b) for the prisoner / (c) since two hours / (d) No error.
20. (a) Mr. Bacon has / (b) great affection to/ (c) his family./ (d) No error.
21. (a) Rekha has a great/ (b) enmity for her/ (c) brother's friend./ (d) No error.
22. (a) He threw the bucket/ (b) into the river/ (c) and returned home without any water./ (d) No error.
23. (a) There appears/ (b) to be very little/ (c) understanding among the two brothers / (d) No error.
24. (a) She was/ (b) angry on me because I/ (c) had not invited her to party./ (d) No error.
25. (a) He got a prestigious job though/ (b) he was not worthy/ (c) for it./ (d) No error.
26. (a) He described about/ (b) the incident/ (c) in a very interesting way./ (d) No error.
27. (a) My father/ (b) deals/ (c)with garments/ (d) No error.
28. (a) I cannot / (b) deal from/ (c) those unruly students/ (d) No error.
29. (a) Despite of / (b) working hard/ (c) he failed/ (d) No error.
30. (a) Ashok/ (b) married with/ (c) Rekha last month./ (d) No error.
31. (a) The earth's atmosphere/ (b) comprises of/ (c) three layers/ (d) No error.
32. (a) The court held/ (b) the local MLA responsible/ (c) for the loss or damage to any public property./ (d) No error.
33. (a) He was debarred to attend/ (b) the monsoon session/ (c)of the Parliament./ (d) No error.
34. (a) We will have to await for/ (b) the result/ (c) as the management is on strike./ (d) No error.
35. (a) He should refrain/ (b) to associate himself with any party/ (c) because people have faith in his integrity./(d) No
error.
36. (a) The songs of / (b) the old movies are/ (c) worth listening to./ (d) No error.
37. (a) It should be obvious to you/ (b) that if you persist bothering him,/ (c) he will get angry with you/ (d) No error.
38. (a) I certainly/ (b) differ with you/ (c) in this matter (d) No error.
39. (a)He had a suspected f racture, / (b) so he was/ (c)admitted into the hospital/ (d) No error.
40. (a) If you put your / (b) heart to it,/ (c) you will be a winner/ (d) No error.
176 | P a g
e
CONJUNCTIONS
1. (a) You are quite cynical/ (b) when you say that the reason why we have/ (c) sucha large turnout is because we are serving
refreshments./ (d) No error.
2. (a) That store/ (b) hadn’t hardly/ (c) any of those goods./(d) No error.
3. (a) I needed that money/ (b) so desperately, it was/ (c) like manna from heaven when it arrived./ (d) No error.
4. (a) The period/ (b) between 1980 to 1990/ (c) was very significant in my life./ (d)No error.
5. (a) No sooner had the hockey match started/ (b) when it began/ (c) to rain./(d) Noerror.
6. (a) She is very/ (b) beautiful/ (c) but intelligent./ (d) No error.
7. (a) Your success in the IAS examinations depends not only on/ (b) what papers you have selected/ (c) but on how you have
written them/ (d) No error.
8. (a) No sooner had/ (b) he arrived then/ (c) he was asked to leave again./ (d) No error.
9. (a) I haven’t been/ (b) to New York before and/ (c) neither my sister./ (d) No error.
10. (a) Scarcely had/ (b) I arrived than/ (c) the train left./ (d) No error.
11. (a) The reason why/ (b) he was rejected/ (c) was because he was too young./ (d)No error.
12. (a) Unless you do not give/ (b) the keys of the safe/ (c) you will be shot./ (d) No error.
13. (a) None of the diplomats at the conference/ (b) was able either to/ (c) comprehend or solve the problem./ (d) No error.
14. (a) I have found that he is/ (b) neither willing/ (c) or capable./ (d) No error.
15. (a) We are extremely pleased/(b) for excited as well to invite you/(c) to attend the meeting./(d) No error.
16. (a) When her son got a job/ (b) then she was/ (c) very happy./ (d) No error.
17. (a) Bread and butter/ (b) is/ (c) all we want./ (d) No error.
18. (a) The cost of the new/ (b) machines is likely to/ (c) be so high as ten/ (d) times the existing ones./ (e) No error.
19. (a) He walked as faster/ (b) as he could so that/ (c) he would not miss the train to work./ (d) No error.
20. (a) She was running/ (b) a very high fever/(c) and thus her mother takes her to the doctor./(d) No error.
21. (a) No sooner the teacher/ (b) enter the class/ (c) than the students stood up./ (d) No error.
22. (a) Unless/ (b) you will work hard,/ (c) you cannot pass./ (d) No error.
23. (a) Keep him at an arm's length/ (b) lest you may not repent/(c) in the long run./(d)No error.
24. (a) The briefing/ (b) will be held/ (c) between 2 P.M. to 3 P.M./ (d) No error.
25. (a) Unless you do not meet/ (b) all the requirements/ (c) your application will be rejected./ (d) No error.
26. (a) How do you manage to speak/ (b) to her with/ (c) so great patience./ (d) No error.
27. (a) No sooner she had realized/ (b) her blunder than she began/ (c) to take corrective measures./ (d) No error.
28. (a) She was not/ (b) so well versed in/ (c) English that we had expected./ (d) No error.
29. (a) Mr. Sinha, my friend, philosopher and guide not only stopped coming/ (b) here but also going to any place/ (c) which
is related to his wife’s life./ (d) No error.
30. (a) Neither the doctor nor the nurses/ (b) were asleep when/ (c) the injured was brought to the hospital./ (d) No error.
31. (a) As soon as the peon rings the bell, then all/ (b) the students come to the assembly room/ (c) for prayer./ (d) No error.
32. (a) Hardly had he come out of the bus/ (b) then the bomb exploded/ (c) and shattered the bus into pieces./ (d) No error.
33. (a) Scarcely had I bought/ (b) the ticket when the train/ (c) left the platform with a hissing sound./ (d) No error.
34. (a) Many of the freedom fighters/ (b) are so respectful as Gandhiji/ (c) if not more. /(d) No error.
35. (a) This is perhaps the same place who/ (b) was chosen by us/ (c) for the picnic./(d) No error.
36. (a) She asked me that why I was/ (b) not preparing for the/ (c) Civil Service Examinations./ (d) No error.
37. A) Run fast/ (b) lest you will/ (c) lose the race/ (d) No error.
38. (a) As Amir Khan is a perfectionist,/ (b) so he always insists/ (c) on retakes till he is satisfied with the shot./ (d) No error.
39. (a) You must either/ (b) work hard/ (c) else be prepared to lead a miserable life./(d) No error.
40. (a) I don’t know if any of the students/ (b) of our school is going/ (c) to pay any heed to the principal’s order or not./(d)
No error.
177| P a g e
CRITICAL REASONING
Direction for questions 1 to 15: Select the correct alternative from the given choices.
1. Which of the following best completes the passage below? In a survey of job applicants, two-fifths admitted to
being at least a little dishonest. However, the survey may underestimate the proportion of job applicants who are
dishonest, because.
A. some dishonest people taking the survey might have claimed on the survey to be honest
B. some generally honest people taking the survey might have claimed on the survey to be dishonest
C. some people who claimed on the survey to be at least a little dishonest may be very dishonest
D. some people who claimed on the survey to be dishonest may have been answering honestly
E. some people who are not job applicants are probably at least a little dishonest
2. The average life expectancy for the United States population as a whole is 73.9 years, but children born in Hawaii
will live an average of 77 years, and those born in Louisiana, 71.7 years. If a newlywed couple from Louisiana were to
begin their family in Hawaii, therefore, their children would be expected to live longer than would be the case if the
family remained in Louisiana.
Which of the following, if true, would most seriously weaken the conclusion drawn in the passage?
A. Insurance company statisticians do not believe that moving to Hawaii will significantly lengthen the average
Louisianan's life.
B. The governor of Louisiana has falsely alleged that statistics for his state are inaccurate.
C. The longevity ascribed to Hawaii's current population is attributable mostly to genetically determined factors.
D. Thirty percent of all Louisianans can expect to live longer than 77 years.
E. Most of the Hawaiian Islands have levels of air pollution well below the national average for the United States
3. The average life expectancy for the United States population as a whole is 73.9 years, but children born in Hawaii
will live an average of 77 years, and those born in Louisiana, 71.7 years. If a newlywed couple from Louisiana were to
begin their family in Hawaii, therefore, their children would be expected to live longer than would be the case if the
family remained in Louisiana.
Which of the following statements, if true, would most significantly strengthen the conclusion drawn in the
passage?
A. As population density increases in Hawaii, life expectancy figures for that state are likely to be revised downward.
B. Environmental factors tending to favor longevity are abundant in Hawaii and less numerous in Louisiana.
C. Twenty-five percent of all Louisianans who move to Hawaii live longer than 77 years.
D. Over the last decade, average life expectancy has risen at a higher rate for Louisianans than for Hawaiians.
E. Studies show that the average life expectancy for Hawaiians who move permanently to Louisiana is roughly equal
to that of Hawaiians who remain in Hawaii.
4. Insurance Company X is considering issuing a new policy to cover services required by elderly people who suffer
from diseases that afflict the elderly. Premiums for the policy must be low enough to attract customers. Therefore,
Company X is concerned that the income from the policies would not be sufficient to pay for the claims that would
be made.
Which of the following strategies would be most likely to minimize Company X's losses on the policies?
A. Attracting middle-aged customers unlikely to submit claims for benefits for many years.
B. Insuring only those individuals who did not suffer any serious diseases as children
C. Including a greater number of services in the policy than are included in other policies of lower cost
D. Insuring only those individuals who were rejected by other companies for similar policies
E. Insuring only those individuals who are wealthy enough to pay for the medical services
5. A program instituted in a particular state allows parents to prepay their children's future college tuition at current
rates. The program then pays the tuition annually for the child at any of the state's public colleges in which the
child enrolls. Parents should participate in the program as a means of decreasing the cost for their children's college
178 | P a g
e
education.
Which of the following, if true, is the most appropriate reason for parents NOT to participate in the program?
A. the parents are unsure about which public college in the state the child will attend.
B. The amount of money accumulated by putting the prepayment funds in an interest-bearing account today will be
greater than the total cost of tuition for any of the public colleges when the child enrolls.
C. The annual cost of tuition at the state's public colleges is expected to increase at a faster rate than the annual
increase in the cost of living.
D. Some of the state's public colleges are contemplating large increases in tuition next year.
E. The prepayment plan would not cover the cost of room and board at any of the state's public colleges.
6. Company Alpha buys free-travel coupons from people who are awarded the coupons by Bravo Airlines for flying
frequently on Bravo airplanes. The coupons are sold to people who payles for the coupons than they would pay by
purchasing tickets from Bravo. This making of coupons results in lost revenue for Bravo. To discourage the buying
and selling of free-travel coupons, it would be best for Bravo Airlines to restrict the
7. In parts of South America, vitamin-A deficiency is a serious health problem, especially among children. In one region,
are attempting to improve nutrition by encouraging farmers to plant a new variety of sweet potato called
SPK004 that is rich in beta-carotene, which the body converts into vitamin A. The plan has good chances of
success, since sweet potato is a staple of the region’s diet and agriculture, and the varieties currently grown
contain little beta- carotene.
Which of the following, if true, most strongly supports the prediction that the plan will succeed?
A. The growing conditions required by the varieties of sweet potato currently cultivated in the region are conditions
in which SPK004 can flourish.
B. The flesh of SPK004 differs from that of the currently cultivated sweet potatoes in color and texture, so traditional
foods would look somewhat different when prepared from SPK004.
C. There are no other varieties of sweet potato that are significantly richer in beta-carotene than SPK004 is.
D. The varieties of sweet potato currently cultivated in the region contain some important nutrients that are lacking
in SPK004.
E. There are other vegetables currently grown in the region that contain more beta-carotene than the currently
cultivated varieties of sweet potato do
8. The prairie vole, a small North American grassland rodent, breeds year around, and a group of voles living together
consists primarily of an extended family, often including two or more litters. Voles commonly live in large groups
from late autumn to winter; from spring thru early autumn, however, most voles live in far smaller groups. The
seasonal variation in groups size can probably be explained by a seasonal variation in mortality among young
voles. Which of thefollowing, if true, provides the strongest support for the explanation above?
A) It is the spring and in the early summer that prairie vole communities generally contain the highest proportion of
young voles.
B) Prairie vole populations vary dramatically in size from year to year.
C) The prairie vole subsists primarily on broad-leaved plants that are abundant only in spring.
D) Winters in prairie vole's habitat are often harsh, with temperatures that drop well below freezing.
179 | P a g
e
E) Snakes, a major predator of young prairie voles, or active only from spring thru early autumn.
9. Extinction is a process that can depend on a variety of ecological, geographical, and physiological variables. These
variables affect different species of organisms in different ways, and should, therefore, yield a random pattern of
extinctions. However, the fossil record shows that extinction occurs in a surprisingly definite pattern, with many
species vanishing at the same time.
Which of the following, if true, forms the best basis for at least a partial explanation of the patterned
extinctions revealed by the fossil record?
(A) Major episodes of extinction can result from widespread environmental disturbances that affect numerous
different species.
(B) Certain extinction episodes selectively affect organisms with particular sets of characteristics unique to their
species.
(C) Some species become extinct because of accumulated gradual changes in their local environments.
(D) In geologically recent times, for which there is no fossil record, human intervention has changed the pattern of
extinctions.
(E) Species that are widely dispersed are the least likely to become extinct
10. Most household appliances use electricity only when in use. Many microwave ovens, however, have built-in clocks
and so use some electricity even when they are not in Use. The clocks each consume about 45 kilowatt-hours per
year. Therefore, households whose microwave oven has no built-in clock use 45 kilowatt-hours per year less ,on
average,than do comparable households microwave oven is otherwise similar but has a built-in clock.
11. Recent studies show that people between the ages of 13 and 55 produce 65 pounds more garbage per year than
they did in 1995. This increase has led to a higher percentage of the total garbage produced by all the age groups.
This age group constitutes a growing percentage of the population, so it partially explains this rise.
12. Early in the twentieth century, Lake Konfa became very polluted. Recently fish populations have recovered as
release of industrial pollutants has declined and the lak waters have become cleaner. Fears are now being voiced
that the planned construction of an oil pipeline across the lake’s bottom might revive pollution and cause the fish
population to decline again. However, a technology for preventing leaks is being installed. Therefore, provided this
technology is effective, those fears are groundless.
180 | P a g
e
The argument depends on assuming which of the following?
A. Apart from development related to the pipeline, there will be no new industrial development around the lake
that will create renewed pollution in its waters.
B. Other than the possibility of a leak, there is no realistic pollution threat posed to the lake by the pipeline’s
construction.
C. There is no reason to believe that the leak-preventing technology would be ineffective when installed in the
pipeline in Lake Konfa.
D. Damage to the lake’s fish populations would be the only harm that a leak of oil from the pipeline would cause.
E. The species of fish that are present in Lake Konfa now are the same as those that were in the lake before it was
affected by pollution.
13. Some geologists argue that if oil is as common in unsampled areas of the world as it is in those already sampled, our
current estimate of reserves that exist underground must be multiplied by a factor of 10,000. From this we can
conclude that we can meet the oil needs of the entire world for at least five centuries, even assuming that future
consumption grows at an accelerating rate.
To reach the stated conclusion, the author must assume which of the following?
(A) It is possible to recover the oil contained in unexplored areas of the world
(B) The consumption rate for oil will not grow rapidly
(C) Oil will remain an important energy source for at least 500 years
(D) The world will achieve and maintain zero population growth
(E) New technology will make oil discovery and drilling more feasible than ever before
14. Commentator: The theory of trade retaliation states that countries closed out of any of another country’s markets
should close some of their own markets to the other country in order to pressure the other country to reopen its
markets. If every country acted according to this theory, no country would trade with any other.
15. In the past the country of Siduria has relied heavily on imported oil. Siduria recently implemented a program to
convert heating systems from oil to natural gas. Siduria already produces more natural gas each year than it burns,
and oil production in Sidurian oil fields is increasing at a steady pace. If these trends in fuel production and usage
continue, therefore, Sidurian reliance on foreign sources for fuel should decline soon.
16. Homeowners aged 40 to 50 are more likely to purchase ice cream and are more likely to purchase it in larger
amounts than are members of any other demographic group. The popular belief that teenagers eat more ice cream
than adults must, therefore, be false.
181 | P a g
e
(B) does not supply information about homeowners in age groups other than 40 to 50
(C) depends on popular belief rather than on documented research findings
(D) does not specify the precise amount of ice cream purchased by any demographic group
(E) discusses ice cream rather than more nutritious and healthful foods
17. Because no employee wants to be associated with bad news in the eyes of a superior, information about serious
problems at lower levels is progressively softened and distorted as it goes up each step in the management
hierarchy. The chief executive is therefore, less well informed about problems at lower levels than are his or her
subordinates at those levels.
18. A company is considering changing its policy concerning daily working hours. Currently, this company requires all
employees to arrive at work at 8 a.m. The proposed policy would permit each employee to decide when to arrives
from as early as 6 a.m. to as late as 11 a.m. The adoption of this policy would be most likely to decrease employees
productivity
19. Manufacturers are now required to make all cigarette lighters child-resistant by equipping them with safety levers.
But this change is unlikely to result in a significant reduction in the number of fires caused by children playing with
lighters, because children given the opportunity can figure out how to work the safety levers and ………..
Which of the following, if true, most logically completes the argument below?
(A) the addition of the safety levers has made lighters more expensive than they were before the requirement was
instituted.
(B) adults are more likely to leave child-resistant lighters than non-child-resistant lighters in places that are
accessible to children.
(C) many of the fires started by young children are quickly detected and extinguished by their parents
(D) unlike child-resistant lighters, lighters that are not child-resistant can be operated by children as young as two
years old.
(E) approximately 5,000 fires per year have been attributed to children playing with lighters before the safety levers
were required.
20. Burglar alarm systems can be inaccurate in two ways: they can fail to detect an intruder when one is present, or
they can falsely report the presence of an intruder. To detect the presence of an intruder most accurately,
homeowners should purchase the system that is least likely to falsely report the presence of an intruder.
If true, which of the following would most strengthen the above recommendation?
(A) The police respond to automatic alerts from burglar alarms more quickly than they do to telephone calls
reporting the presence of an intruder.
182 | P a g
e
(B) The burglar alarm system that is least likely to sound its alarm when there is no intruder present is equal in price
to all other major brands of alarm systems.
(C) Local police forces vary in their response rates to burglar alarm systems.
(D) All burglar alarm systems have the same rate of mistaking an animal for a burglar.
(E) All burglar alarm systems have the same rate of failing to detect an intruder when one is present.
21. Studies have shown that elderly people who practice a religion are much more likely to die immediately after an
important religious holiday period than immediately before one. Researchers have concluded that the will to live
can prolong life, at least for short periods of time.
Which of the following, if true, would most strengthen the researchers' conclusion?
(A) elderly people who practice a religion are less likely to die immediately before or during an important religious
holiday than at any other time of the year
(B) elderly people who practice a religion appear to experience less anxiety at the prospect of dying than do other
people
(C) some elderly people who do practice a religion live much longer than most elderly people who do not
(D) most elderly people who participate in religious holidays have different reasons for participating than young
people do
(E) many religious have important holidays in the spring and fall, seasons with the lowest death rates for elderly
people
22. Sania: The newest workers in the workforce are the most effective digital marketing employees because they are
more likely to use social networking websites and tools themselves.
Carlos: But effective digital marketing also requires very technical expertise, such as search engine optimization, that
is best learned on the job via prolonged exposure and instruction.
23. State X recently decided to cut state funding for the public library system in County X. To help counteract this cut in
funding, the county library system has increased library late fees from $.10 per day to $.12 per day. Since the fee
increase, library administrators have seen no decline in the number of overdue books. The director of the county
library system concludes that the fee hike has helped to counteract the cut in state funding.
Which of the following statements, if true, most strengthens the director’s claim?
(A) Since the fee increase, library administrators have noted a significant decrease in the number of books borrowed
each day.
(B) The library system incurred minor costs to make its visitors aware of the late fee increase.
(C) Since the fee increase, there has been no significant change in the average number of days that books are
overdue before they are returned.
(D) The library system in County X tracks its books through a very advanced database system, allowing library
administrators to have access to very accurate statistics on the number of overdue books at any given time.
(E) Since the reduction in state funding, the library system in County X has eliminated 10% of its staff, creating a
2%reduction in costs.
24. A report on acid rain concluded, “Most forests in Canada are not being damaged by acid rain.” Critics of the report
insist the conclusion be changed to, “Most forests in Canada do not show visible symptoms of damage by acid rain,
such as abnormal loss of leaves, slower rates of growth, or higher mortality.” Which of the following, if true,
provides the best logical justification for the critics’ insistence that the report’s conclusion be changed?
183 | P a g
e
(A) Some forests in Canada are being damaged by acid rain.
(B) Acid rain could be causing damage for which symptoms have not yet become visible
(C) The report does not compare acid rain damage to Canadian forests with acid rain damage to forests in other
countries.
(D) All forests in Canada have received acid rain during the past fifteen years.
(E) The severity of damage by acid rain differs from forest to forest.
25. A report on acid rain concluded, “Most forests in Canada are not being damaged by acid rain.” Critics of the report
insist the conclusion be changed to, “Most forests in Canada do not show visible symptoms of damage by acid rain,
such as abnormal loss of leaves, slower rates of growth, or higher mortality.”
Which of the following, if true, provides the best logical justification for the critics’ insistence that the report’s
conclusion be changed?
(A) Some forests in Canada are being damaged by acid rain.
(B) Acid rain could be causing damage for which symptoms have not yet become visible.
(C) The report does not compare acid rain damage to Canadian forests with acid rain damage to forests in other
countries.
(D) All forests in Canada have received acid rain during the past fifteen years.
(E) The severity of damage by acid rain differs from forest to forest.
26. On average, the number of speeding tickets issued in County X every year is three times greater than the number of
speeding tickets issued in County Y during the same period. Therefore, the number of people who exceed the speed
limit must be higher in County X than in County Y. Which of the following describes a flaw in the reasoning above?
(A) The argument fails to take into account that the speed limit may be different in the two counties.
(B) The argument fails to take into account that the number of tickets issued in County X may reflect a lower
proportion of drivers overall in that county.
(C) The argument fails to take into account that a single driver can receive more than one ticket in a given year.
(D) The argument fails to take into account that residents of County Y may be more law-abiding overall.
(E) The argument fails to take into account that residents of County X may not be aware of the speed limit in that
county.
27. The last members of a now-extinct species of a European wild deer called the giant dear lived in Ireland about
16,000 years ago. Prehistoric cave paintings in France depict this animal as having a large hump on its back. Fossils of
this animal, however, do not show any hump. Nevertheless, there is no reason to conclude that the cave paintings
are therefore inaccurate in this regard, since .
28. Because it was long thought that few people would watch lengthy televised political messages, most televised
political advertisements, like commercial advertisements, took the form of short messages. Last year, however, one
candidate produced a half-hour-long advertisement. During the half hour the advertisement was aired, a substantial
portion of the viewing public tuned into the advertisement. Clearly, then, many more people are interested in
watching lengthy televised political messages than was previously thought.
184 | P a g
e
(D) Most people who are interested in political issues watch television regularly.
(E) Most of the viewers who tuned in to the candidate’s half-hour-long advertisement last year did not change
channels after the first few minutes.
29. Gortland has long been narrowly self-sufficient in both grain and meat. However, as per capita income in Gortland
has risen toward the world average, per capita consumption of meat has also risen toward the world average, and it
takes several pounds of grain to produce one pound of meat. Therefore, since per capita income continues to rise,
whereas domestic grain production will not increase, Gortland will soon have to import either grain or meat or both.
30. Mayor: Migrating shorebirds stop at our beach just to feed on horseshoe-crab eggs, a phenomenon that attracts
tourists. To bring more tourists, the town council plans to undertake a beach reclamation project to double the area
available to crabs for nesting. Birdwatcher: Without a high density of crabs on a beach, migrating shorebirds will go
hungry because shorebirds only eat eggs that a crab happens to uncover when it is digging its own nest. Which of
the following, if true, would provide the mayor with the strongest counter to the birdwatcher’s objection?
31. If highways were restricted to cars and only those trucks with capacity of less than 8 tons, most of the truck traffic
would be forced to run outside highways. Such a reduction in the amount of truck traffic would reduce the risk of
collisions on highways.
The conclusion drawn in the Iast sentence depends on which of the following assumptions?
(A)The roads outside highway would be as convenient as highway for most drivers of trucks..
(B)Most of the roads outside highways are not ready to handle truck traffic.
(C)Most trucks that are currently running in highway have a capacity of more than 8 tons.
(D)Cars are at greater risk of being involved in collisions than are trucks.
32. If highways were restricted to cars and only those trucks with capacity of less than 8 tons, most of the truck traffic
would be forced to run outside highways. Such a reduction in the amount of truck traffic would reduce the risk of
collisions on highways.
Which of the following, if true, would most strengthen the conclusion drawn in the second sentence?
(A) Highways are experiencing overcrowded traffic mainly because of sharp increases in car traffic
(B) Many drivers of trucks would rather buy trucks with a capacity of less than 8 tons than be excluded from highways.
(C)The number of collisions that occur near highways has reduced in recent years
(D)Trucks that have a capacity of more than 8 tons cause a disproportionately large number of collisions on highways
33. In contrast to the earlier predictions, demand for sugarcane has not risen in recent years, yet, even though
production amounts and price have also been stable during the last 3 years. Last year, sugarcane growers increased
their profits by more than 10 percent over the previous year’s level.
Any of the following statements, if true, about last year, helps explain the rise in profits EXCEPT
(A) Many sugar factories that are large consumers of sugarcane increase their production of sugarcane-based ethanol,
yet their overall consumption of sugarcane decreased.
185 | P a g
e
(B) Rainfall in sugarcane-growing regions was higher than it had been during previous year, allowing the growers to
save money on expensive artificial irrigation.
(C) Sugarcane growers have saved money on wages by switching from paying laborers a daily wage to paying them
by the amount harvested.
(D) Many small sugarcane growers joined together to form an association of sugarcane producers and began to buy
supplies at low group rates.
34. Pharmaceutical manufacturers have long claimed that one of the main reasons they give doctors free drug samples
is so that doctors can pass the medicine along to poor patients. However, a new study shows that high-income, well
insured individuals receive considerably more prescription drug samples than do low-income, poorly insured
individuals. This is because doctors favor affluent people with health insurance.
Which of the following, if true, most seriously jeopardizes the validity of the explanation for why high-income
individuals receive more free prescription drug samples than low-income individuals do?
(A) Independent medical clinics not affiliated with large hospitals receive only a small percentage of the free drug
samples distributed by pharmaceutical manufacturers.
(B) Because of the associated costs, low-income people see doctors less often, if at all.
(C) Some medical offices refuse to treat individuals for conditions that are not critical or life-threatening if the
individual does not have the means to pay for the treatment.
(D) Once the free drug sample supply in a doctor’s office is gone, the patients are forced to pay for their
prescriptions.
(E) Though they claim to give doctors free drug samples in order to help poor patients, the real reasons
pharmaceutical companies do it are to increase brand awareness and to influence the doctor to prescribe these drugs
more often.
35. In the early 20th century, ivory poaching led to the near extinction of the black rhino and the African elephant. As a
result, numerous African nations supported a complete ban on all ivory sales. This ban has been in effect since 1989.
The governments of South Africa, Botswana, and Namibia have recently put up for auction thousands of tons of
confiscated ivory horns and tusks, in spite of the continued moratorium. However, the three governments have the
full support of the same conservationists who helped impose the 1989 international ban on ivory sales.
Which of the following, if true, contributes most to an explanation of why conservationists support South Africa’s,
Botswana’s, and Namibia’s auction of ivory?
(A) The international demand for ivory has decreased significantly since 1989.
(B) Most wild black rhinos and African elephants live outside of South Africa, Botswana, and Namibia.
(C) Once the tons of confiscated ivory are auctioned, the market will be flooded with ivory, making poaching
economically impractical.
(D) If it were not for the auction, the confiscated ivory could never be used and would have to remain in government
warehouses.
(E) Due to major conservation efforts, black rhino and African elephant populations have slowly but steadily
increased in the last few years.
36. Recent market research has indicated that, because Global tech’s current line of cell phones has remained the
sameover the last few years without adding any cutting-edge features that consumers demand, it has lost
considerable market share in the industry. So, in an attempt to regain market share, Global tech has proposed a new
line of
186 | P a g
e
cell phones with the capability of synchronizing various household appliances such as computers, light switches, and
even toasters.
Which of the following, if true, provides the strongest reason to expect that the proposed line of cell phones will
be successful in regaining market share?
(A) Engineers have only recently been able to discover a cost-effective way to remotely control household appliances
through cell phones.
(B) An increasing number of countries throughout the world now have more cell phones than people.
(C) The number of appliances that can be remotely controlled through a cell phone signal has more than tripled in
thelast few years.
(D) Global tech’s market share began decreasing at the same time as it stopped adding new features to its
cell phones.
(E) In today’s economy, the new time-saving ability to remotely control appliances is being demanded by more and
more consumers.
37. Researchers have recently discovered that approximately 70% of restaurant lemon wedges they studied were
contaminated with harmful microorganisms such as bacterial and fungal pathogens. The researchers looked at
numerous different restaurants in different regions of the country. Most of the organisms had the potential to cause
infectious disease. For that reason, people should not order lemon wedges with their drinks.
Which of the following, if true, would most weaken the conclusion above?
(A) The researchers could not determine why or how the microbial contamination occurred on the lemon wedges.
(B) The researchers failed to investigate contamination of restaurant lime wedges by harmful microorganisms.
(C) The researchers found that people who ordered lemon wedges at restaurants were equally likely to contract the
diseases caused by the discovered bacteria as were people who did not order lemon wedges.
(D) Health laws require lemons to be handled with gloves or tongs, but the common practice is for waiters and
waitresses to handle them with their bare hands.
(E) Many factors that have nothing to do with lemons affect the chance of an individual contracting a disease by
coming into contact with bacteria. These factors include such things as the health and age of the individual as well as
the status of his or her immune system.
38. The administrative budget in the Central Valley school district is proportional to the value of the valley’s property
tax base, the chief source of funding for the school district. As revenue from property taxes increases, each budget
segment of the school district is increased proportionately.
Which of the following statements, if true, is the best basis for a criticism of he Central Valley’s budgeting policy as
an economically sound budgeting method for school districts?
(A) The school district might continue to pay for past inefficient allocation of funds.
(B) The revenue from property taxes has remained relatively unchanged for the last decade.
(C) Student performance is affected by fluctuations in the overall school district budget.
(D) Many Central Valley taxpayers have complained about the high property tax rates in the area.
(E) The current budgeting system has little impact on whether parents decide to take their children to non-district
funded classes.
39. A newspaper article in the Smithville Herald argued that the strength of unions was declining. The article’s evidence
was the decreasing number and size of strikes as though the reason for the unions’ existence was to organize strikes.
187 | P a g
e
Surely, in a modern industrial society, the calling of a strike is evidence that the negotiating position of the union was
too weak. Strong unions do not need to call strikes. They can concentrate their efforts on working with others in the
labor market to achieve common goals, such as profitable and humane working conditions.
The argument criticizing the newspaper article employs which one of the following strategies?
(A) Questioning the accuracy of the statistical evidence that the newspaper article uses
(B) Detailing historical changes that make the newspaper articles’ analysis outdated
(C) Reinterpreting evidence that the newspaper article uses as indicating the opposite of what the newspaper
Concludes
(D) Arguing that the newspaper article’s conclusion is motivated by a desire to change the role of unions
(E) Pointing to common interests among unions and management
40. Widespread use of lectures in class-rooms in business schools leads to severe negative consequences.
The first consequence is theoretically knowledgeable graduates who cannot apply theory to solve real
world problems. The more serious consequence is that lectures encourage a feeling of total omniscience
among them which persists for quite some time after graduating. This feeling prevents "them from
learning from their subordinates and colleagues.
Which of the following can best help to reduce these negative consequences among the students in a
business school?
(A) Use illustrations of real life problems in classrooms.
(B) Send the students to find business problems so that it can be discussed in classrooms.
(C) Business education to be given to students, who have work experience.
(D) Modify the pedagogy to have knowledge of theory and application in parallel.
(E) Removing theoretical inputs from the curriculum altogether; only practical problems to be discussed in class
rooms.
41. “The sum of behaviour is to retain a man’s dignity without intruding upon the liberty of others”, stated
Sir Francis Bacon. If this is the case, then not intruding upon another’s liberty is impossible.
42. The most successful coaches in Indian club football tournaments are from Latin American countries. In
most of the Latin American countries, football is more popular sport than cricket.
188 | P a g
e
43. This season will pass. The Prime Minister may not win LokSabha elections, or she may; she may not
continue as Prime Minister, or she may. The country will survive whatever the texture of politics in this
decade or the next.
Which of the following, IF TRUE, will BEST reinforce the author’s view?
(A) The survival of any Prime Minister is dependent on the country’s economics growth.
(B) The country has a vibrant young working population.
(C) The survival of the country depends on a dynamic, growth-oriented Prime Minister, not on the texture of politics.
(D) The previous season had also witnessed similar political uncertainty.
(E) The survival of the Prime Minister is dependent on the political texture of the country
44. Ranu is an ordinary sportsperson. In the last two university sprint events, her performances in the heats
were pathetic. Which of the following, if true, weakens the above argument the most?
(A) She had participated in the college swimming competition and finished last.
(B) She is a national shot -put champion.
(C) The last two times, Ranu had to compete with national level runners. Had she been in other heats, she would
have reached quarterfinals.
(D)Ranu was the only player who represented her college in the sprint events.
(E) In the college sprint events, Ranu always won
45. “On a wing and a prayer the Indian team landed in Heathrow to take on their formidable oppenents”.
From the above sentence it can be inferred that the Indian team was
(A)high in spirits
(B)well prepared
(C)over confident
(D)under prepared
(E)buoyant
189 | P a g
e
READING COMPREHENSION
PASSAGE – 1
"I Have a Dream" is a public speech delivered by American civil rights activist Martin Luther King Jr. during the March on
Washington for Jobs and Freedom on August 28, 1963, in which he calls for an end to racism in the United States and
called for civil and economic rights. Delivered to over 250,000 civil rights supporters from the steps of the Lincoln
Memorial in Washington, D.C., the speech was a defining moment of the civil rights movement.
Beginning with a reference to the Emancipation Proclamation, which freed millions of slaves in 1863, King observes that:
"one hundred years later, the Negro still is not free". Toward the end of the speech, King departed from his prepared
text for a partly improvised peroration on the theme "I have a dream", prompted by Mahalia Jackson's cry: "Tell them
about the dream, Martin!" In this part of the speech, which most excited the listeners and has now become its most
famous, King described his dreams of freedom and equality arising from a land of slavery and hatred. Jon Meacham
writes that, "With a single phrase, Martin Luther King Jr. joined Jefferson and Lincoln in the ranks of men who've shaped
modern America". The speech was ranked the top American speech of the 20th century in a 1999 poll of scholars of
public address.
3) From the last paragraph, give one word for “to leave”
a) Departed
b) Proclamation
c) Improvised
d) Address
PASSAGE – 2
Today I Rabindranath Tagore complete eighty years of my life .As I look back on the vast stretch of years that lie behind
me and see in clear perspective the history of my early development, I am struck by the change that has taken place
both in my own attitude and in the psychology of my countrymen -- a change that carries within it a cause of profound
tragedy.
190 | P a g
e
Our direct contact with the larger world of men was linked up with the contemporary history of the English people
whom we came to know in those earlier days. It was mainly through their mighty literature that we formed our ideas
with regard to these newcomers to our Indian shores. In those days the type of learning that was served out to us was
neither plentiful nor diverse, nor was the spirit of scientific enquiry very much in evidence. Thus their scope being strictly
limited, the educated of those days had recourse to English language and literature. Their days and nights were eloquent
with the stately declamations of Burke, with Macaulay’s long-rolling sentences; discussions centered upon
Shakespeare's drama and Byron's poetry and above all upon the large-hearted liberalism of the nineteenth-century
English politics.
At the time though tentative attempts were being made to gain our national independence, at heart we had not lost
faith in the generosity of the English race. This belief was so firmly rooted in the sentiments of our leaders as to lead
them to hope that the victor would of his own grace pave the path of freedom for the vanquished. This belief was based
upon the fact that England at the time provided a shelter to all those who had to flee from persecution in their own
country. Political martyrs who had suffered for the honour of their people were accorded unreserved welcome at the
hands of the English.
I was impressed by this evidence of liberal humanity in the character of the English and thus I was led to set them on the
pedestal of my highest respect. This generosity in their national character had not yet been vitiated by imperialist pride.
About this time, as a boy in England, I had the opportunity of listening to the speeches of John Bright, both in and
outside Parliament. The large-hearted, radical liberalism of those speeches, overflowing all narrow national bounds, had
made so deep an impression on my mind that something of it lingers even today, even in these days of graceless
disillusionment.
191 | P a g
e
PASSAGE - 3
Sportsmanship can be conceptualized as an enduring and relatively stable characteristic or disposition such that
individuals differ in the way they are generally expected to behave in sports situations. In general, sportsmanship refers
to virtues such as fairness, self-control, courage, and persistence, and has been associated with interpersonal concepts
of treating others and being treated fairly, maintaining self-control if dealing with others, and respect for both authority
and opponents. Sportsmanship is also looked at as being the way one reacts to a sport/game/player.
The four elements of sportsmanship are often shown being good form, the will to win, equity and fairness. All four
elements are critical and a balance must be found among all four for true sportsmanship to be illustrated. These
elements may also cause conflict, as a person may desire to win more than play in equity and fairness and thus resulting
in a clash within the aspects of sportsmanship. This will cause problems as the person believes they are being a good
sportsman, but they are defeating the purpose of this idea as they are ignoring two key components of being sportsman
like. When athletes become too self-centred, the idea of sportsmanship is dismissed.
Today's sporting culture, in particular the base of elite sport, places great importance on the idea of competition and
winning and thus sportsmanship takes a back seat as a result. In most, if not all sports, sportsmen at the elite level make
the standards on sportsmanship and no matter whether they like it or not, they are seen as leaders and role models in
society.
Since every sport is rule driven, the most common offence of bad sportsmanship is the act of cheating or breaking the
rules to gain an unfair advantage. A competitor who exhibits poor sportsmanship after losing a game or contest is often
called a "sore loser", while a competitor who exhibits poor sportsmanship after winning is typically called a "bad
winner". Sore loser behavior includes blaming others for the loss, not accepting responsibility for personal actions that
contributed to the defeat, reacting to the loss in an immature or improper fashion, making excuses for the defeat, and
citing unfavourable conditions or other petty issues as reasons for the defeat. A bad winner acts in a shallow fashion
after his or her victory, such as by gloating about his or her win, rubbing the win in the face(s) of the opponent(s), and
lowering the opponent(s)'s self-esteem by constantly reminding the opponent(s) of "poor" performance in comparison
(even if the opponent(s) competed well). Not showing respect to the other team is considered to being a bad sportsman
and could lead to demoralizing effects; as Leslie Howe describes: "If a pitcher in baseball decides to pitch not to his
maximum ability suggest that the batter is not at an adequate level, [it] could lead to the batter to have low self-
confidence or worth.
3) If one does not accept responsibility for one’s defeat, one is called a:
a) Sore loser
b) Bad winner
c) Good sportsman
d) Prudent sportsman
4) From the last paragraph, give the opposite of the word ‘deep’:
a) Competitor
b) Pitch
c) Immature
d) Shallow
192 | P a g
e
5) When does the spirit of sportsmanship die?
a) When the sportsman becomes too self-centered
b) When the player loses the will to play
c) When the sportsman behaves badly
d) None of the above
PASSAGE – 4
Artificial intelligence (AI), sometimes called machine intelligence, is intelligence demonstrated by machines, in contrast
to the natural intelligence displayed by humans and other animals. In computer science AI research is defined as the
study of "intelligent agents": any device that perceives its environment and takes actions that maximize its chance of
successfully achieving its goals. Colloquially, the term "artificial intelligence" is applied when a machine mimics
"cognitive" functions that humans associate with other human minds, such as "learning" and "problem solving".
The scope of AI is disputed: as machines become increasingly capable, tasks considered as requiring "intelligence" are
often removed from the definition, a phenomenon known as the AI effect, leading to the quip, "AI is whatever hasn't
been done yet." For instance, optical character recognition is frequently excluded from "artificial intelligence", having
become a routine technology. Capabilities generally classified as AI as of 2017 include successfully understanding human
speech, competing at the highest level in strategic game systems (such as chess and Go), autonomous cars, intelligent
routing in content delivery network and military simulations.
Artificial intelligence was founded as an academic discipline in 1956, and in the years since has experienced several
waves of optimism, followed by disappointment and the loss of funding (known as an "AI winter"), followed by new
approaches, success and renewed funding. For most of its history, AI research has been divided into subfields that often
fail to communicate with each other. These sub-fields are based on technical considerations, such as particular goals
(e.g. "robotics" or "machine learning"), the use of particular tools ("logic" or artificial neural networks), or deep
philosophical differences. Subfields have also been based on social factors (particular institutions or the work of
particular researchers).
The traditional problems (or goals) of AI research include reasoning, knowledge representation, planning, learning,
natural language processing, perception and the ability to move and manipulate objects. General intelligence is among
the field's long-term goals. Approaches include statistical methods, computational intelligence, and traditional symbolic
AI. Many tools are used in AI, including versions of search and mathematical optimization, artificial neural networks, and
methods based on statistics, probability and economics. The AI field draws upon computer science, mathematics,
psychology, linguistics, philosophy and many others.
3) Leaning and problem solving by machines is a colloquial way of understanding artificial intelligence?
a) Yes
b) No
c) Artificial intelligence means acquiring knowledge through books
d) It cannot be defined colloquially
193 | P a g
e
a) Yes
b) No
c) As of 2017, they are
d) None of the above
PASSAGE – 5
Like their ancient toga-wearing counterparts, modern philosophers continue to disagree on the nature of freewill. Do we
really have any control over the choices we make and the things we desire, and if so, to what degree?
Theories of freewill vary, but the ancient words of Plato still line up with our modern perceptions of temptation and
willpower. The revered Greek philosopher argued that the human experience is one of constant struggle between the
intellect and the body, between rationality and desire. Along these lines, true freedom is only achievable when
willpower unchains us from bodily, emotional, instinctual slavery.
You can find similar sentiments throughout world religions, most of which offer a particular and often difficult path to
rise above our darker natures.
And science? Well, science mostly agrees with all of this. Willpower is all about overcoming your natural impulses to eat
cupcakes, skip your morning workout, flirt with the waiter, hit the snooze alarm and check your e-mail during a funeral.
Your willpower, however, is limited. If life were a video game, you'd see a glowing "willpower" or "ego" meter at the top
of the screen next to your "life" meter. Successfully resist one temptation, and the meter depletes a little. The next
temptation depletes the "willpower" meter even more, until there's nothing left at all.
Our modern scientific understanding of willpower in large part stems from a 1996 research experiment involving
chocolate and radishes. Psychologist Roy Baumeister led a study in which 67 test subjects were presented with tempting
chocolate chip cookies and other chocolate-flavored treats before a persistence-testing puzzle. Here's the catch: The
researchers asked some of the participants to abstain from sweets and snack on radishes instead.
Baumeister's results told a fascinating story. The test subjects who resisted the sweet stuff in favor of radishes
performed poorly on the persistence test. They simply didn't have the willpower left to resist slacking off.
The resulting paper, "Ego Depletion: Is the Active Self a Limited Resource?" inspired more than a thousand additional
studies discussing everything from the influence of positive messages to the ego-sapping power of daily decisions
Studies also show that cognitive capacity also affects our ability to hold out against temptation. Cognitive capacity is
essentially your working memory, which you employ when resisting a temptation ... or holding a string of numbers in
your head. A 1999 study from the University of Iowa professor Baba Shiv found that people tasked with remembering a
two-digit number held out better than people remembering a seven-digit number when tempted with chocolate cake.
3) In the second paragraph, what does the expression ‘line up’ signify?
194 | P a g
e
a) Align with
b) disagree with
c) Differ from
d) In discussion with
PASSAGE – 6
Born on Jan 12, 1863 in an affluent Bengali family, Narendra Natha Datta was a precocious child who was what we call
nowadays, an all-rounder, excelling in music, studies and athletics. His father Vishwanatha Datta was a well-known
attorney. However, he took the spiritual route instead and introduced Hinduism to the world in 1893 when he spoke at
the World's Parliament of Religion (probably one of the most epic things any Indian has done abroad!).
The historic speech was given on September 11, 1893 by Swami Vivekananda. Here's the full text of his opening and
closing address:
Sisters and Brothers of America,
It fills my heart with joy unspeakable to rise in response to the warm and cordial welcome which you have given us. I
thank you in the name of the most ancient order of monks in the world; I thank you in the name of the mother of
religions, and I thank you in the name of millions and millions of Hindu people of all classes and sects.
My thanks, also, to some of the speakers on this platform who, referring to the delegates from the Orient, have told you
that these men from far-off nations may well claim the honor of bearing to different lands the idea of toleration. I am
proud to belong to a religion which has taught the world both tolerance and universal acceptance.
We believe not only in universal toleration, but we accept all religions as true. I am proud to belong to a nation which
has sheltered the persecuted and the refugees of all religions and all nations of the earth. I am proud to tell you that we
have gathered in our bosom the purest remnant of the Israelites, who came to Southern India and took refuge with us in
the very year in which their holy temple was shattered to pieces by Roman tyranny. I am proud to belong to the religion
which has sheltered and is still fostering the remnant of the grand Zoroastrian nation. I will quote to you, brethren, a few
lines from a hymn which I remember to have repeated from my earliest boyhood, which is every day repeated by
millions of human beings: "As the different streams having their sources in different paths which men take through
different tendencies, various though they appear, crooked or straight, all lead to Thee."
The present convention, which is one of the most august assemblies ever held, is in itself a vindication, a declaration to
the world of the wonderful doctrine preached in the Gita: "Whosoever comes to me, through whatsoever form, I reach
him; all men are struggling through paths which in the end lead to me." Sectarianism, bigotry, and its horrible
descendant, fanaticism, have long possessed this beautiful earth.
195 | P a g
e
b) Swami Vivekananda
c) Both
d) None
3. Give the opposite of the word “Occident” from the second passage of the speech
a) Delegate
b) Universal
c) Orient
d) Toleration
4. In the phrase: “all lead to Thee”, to whom does the word ‘Thee’ refer?
a) All religions
b) The delegates present
c) Universal brotherhood and peace
d) God
5. In the phrase: “I am proud to belong to a nation” – what nation is the speaker referring to?
a) India
b) Southern India
c) Rome
d) America
PASSAGE – 7
Almost everything has implications for the poor, whether it is education, health, family planning, employment, or the
environment, or the environment: they are all fields where current performance hurts the poor in particular. Policies
that enhance economic growth, and agriculture growth, will favour reduction of poverty. The lives of poor people are
most threatened by potential shortages of water and low water quality, and by air pollution; pressures on common pool
resources also bear most heavily on the poor, and on women in particular.
Since half of India’s poor are in the persistent category, the country’s anti-poverty programmes can at best, have only a
limited impact. They are excessively scattered and unrelated to basic processes in the economy. They focus on regions
with particular problems, such as drought prone areas; subsidy programmes such as the Targeted Public Distribution
System for food; a number of employment-related schemes, and social security measures for the very poor. Most of
them have been shown to be inefficient in terms of the proportion of expenditure actually reaching the poor, and in
terms of their lasting effects. But greater impacts on poverty could be made by improved performance in the social
sectors and the environment generally, concentrating on the states with the greatest backlogs. Better health and
nutrition and better education for all, and clean air and water are the best anti-poverty programmes. To these must be
added reproductive health services and family planning, again most needed in the poorest states : they too are pro poor
measures in themselves for high fertility households.
The majority of the poor remain in rural areas, and measures to redress poverty must concentrate on enhancing both
agricultural growth and non-farm employment. Yet India, for all its anti-poverty commitment, has not seen the increases
in the key investments--irrigation, rural roads, and agriculture research-- that would help to achieve this. Anti-poverty
strategies all over the world rightly give an important place to the empowerment of the poor, and their involvement in
the design and management of schemes intended for their benefit. India has made considerable strides in these
directions : in water, moving to farmer managed watershed development; in a range of activites devolving budgets and
management to panchayats and pushing decentralization. Much of this experience has been positive; but much has not.
Instead of empowering the poor, it has empowered local-vested interests. Like, markets, devolution and
decentralization cannot be guaranteed to help the poor. There is an indispensable role for the state. But frequently
public action by state institutions has not delivered either.
Policies to redress poverty require the positive engagement of central and state governments as well as NGOs, local
communities, and group of beneficiaries. With greater account ability and transparency India’s own resources will be
adequate to overcome poverty. Accountability is needed most particularly in public services that affect the poor : health,
education and the police perhaps more than any other, as well as the general working of bureaucracy. If resources are
196 | P a g
e
not invested or distributed where they are needed, and policies are not framed to benefit the poor or to ensure that the
poor receive the benefits meant for them, this projection of slow poverty decline could become a reality. It is ultimately
down to politics. If more of India’s politicians see electoral promise in genuinely addressing the needs of the poor,
poverty can and will decline much faster.
197 | P a g
e
d) Management by imitating policies of other countries
e) None of these
8. According to the passage what should be the fundamental role of the state?
a) Monitoring working of bureaucracy
b) Implementation of anti-poverty strategies
c) Reducing role of panchayats
d) Empowering local-vested interest
e) None of these
9. How will India’s own resources and systems be helpful to diminish poverty?
a) Positive engagement of central and state government
b) With higher accountability and transparency
c) General working of bureaucracy
d) Devolving budgets and managements to panchayats
e) None of these
Directions: Choose the word/phrase which is most nearly the same in meaning as the word printed in bold as used in
the passage.
10. IMPLICATIONS
a) Impacts
b) Advantages
c) Production
d) Expenses
e) Arrangements
11. CURRENT
a) Fresh
b) Recent
c) Shock
d) Hidden
e) Voltage
12. PRONE
a) Affected
b) Unassailable
c) Incited
d) Liable
e) Vulnerable
Directions: Choose the word/phrase which is most opposite in meaning as the word printed in bold as used in the
passage.
198 | P a g
e
13. EXCESSIVELY
a) Closely
b) Absurdly
c) Meagerly
d) Profusely
e) Moderately
14. LASTING
a) Temporary
b) First
c) Final
d) Timely
e) Concluding
15. HEAVILY
a) Animated
b) Thickly
c) Relaxing
d) Lightly
e) Delicately
199 | P a g
e
PARAJUMBLES
Direction: Rearrange the following sentences in a proper sequence to form a meaningful paragraph and mark the correct
sequence.
1.
(A) 197A war changed the political geography of the subcontinent
(B) Despite the significance of the event. There has been no serious book about the conflict
(C) Surrender at Dacca aims to fill this gap
(D) It also profoundly altered the geo-strategic situation in South-East Asia
(a) ACBD (b) CADB (c) BADC (d) ADBC
2.
(A) Nonetheless, Tocqueville was only one of the first of a long line of thinkers to worry whether such rough equality could
survive in the face of a growing factory system that threatened to create divisions between industrial workers and a new
business elite.
(B)"The government of democracy brings the nation of political rights to the level of the humblest citizens. He wrote," Just
as the dissemination of wealth brings the notion of property within the reach of all the members of the community".
(C) Tocqueville was far too shrewd an observer to be uncritical about the US, but his verdict was fundamentally positive.
(D)No visitor to the US left a more enduring record of his travels and observations than the French writer and political
theorist Alexis de Tocqueville, whose ‘Democracy in America’, first published in A8CE, remains one of the most trenchant
and insightful analyses of American social and political practices.
(a) DACB (b) BACD (c) DCBA (d) DBAC
3.
(A). The potential exchanges between the officials of IBBF and the Maharashtra Body-Building Association has all the
trappings of a drama we are accustomed to.
(B). In the case of sports persons, there is room for some sympathy, but the apathy of the administrators, which has even led
to sanctions from international bodies, is unpardonable.
(C). A case in the point is the hefty penalty of US $A0,000 slapped on the Indian Body-Building Federation for not fulfilling its
commitment for holding the Asian Championships in Mumbai in October.
(D). It is a matter of deep regret and concern that the sports administrators often cause more harm to the image of the
country than sportsmen and sportswomen do through their dismal performances.
(a) CABD (b) DBCA (c) DABC (d) CDBA
4.
(A). Over the years, I have had the opportunities to observe and understand the thought processes behind the ads that have
been flooding both the print and the TV media.
(B). Although there is a huge shift in the quality of ads that we come across on a daily basis-- thanks essentially to
improvement in technology--I somehow can't help but feel that the quality of communication of the message has
become diluted.
(C). Proportionally, the number of ads that lack in quality, have gone up exponentially as well!!
(D). There is an increasing attempt by most companies to be seen as cool and funky.
(E). Another reason could be the burgeoning number of companies, which means an exponential increase in the number
ofads that are being made.
(a) DCABE (b) DCEAB (c) ABDEC (d) BADCE
5.
(A). His political career came to an abrupt end with China's military operation.
(B). He attracted as as repelled.
(C). He was responsible for the debacle.
(D). A man of paradoxes, Menon remained an enigma.
(a) DCAB (b) ACDB (c) DBAC (d) DACB
200 | P a g
e
6.
(A). Thus begins the search for relief: painkillers, ice, yoga, herbs, even surgery
(B). Most computer users develop disorders because they ignore warnings like tingling fingers, a numb hand or a sore
shoulder
(C). They keep pointing and dragging until tendons chafe and scar tissue forms, along with bad habits that are almost
impossible to change
(D). But cures are elusive , because repetitive stree injuries present a bag of ills that often defy easy diagnosis.
(a) BDAC (b) BADC (c) BCAD (d) ABCD
7.
(A). Let us take a look at the manner in which the traditional bank adds value to the customer.
(B). The ability to retain deposits, in itself, is not enough to ensure long-term survival and growth.
(C). The ability to deploy invested funds into productive economic activity at a higher rate of return, hence contributing to
the prosperity of both the economy and the institution, is the other loop in the banking cycle.
(D). Further, as only a small portion of the actual deposit base is retained with the bank in a liquid form, the very survival of
the bank lies in building enough trust with its clientele so as to prevent the occurrence of a sizeable chunk of
simultaneous customer withdrawal (a run on the bank).
E). The bank's basic job is risk absorption- it takes money, which has a lot of attached risk, and provides the customer an
assured rate of return.
(a) CADBE (b) EABCD (c) BDAEC (d) AEDBC
8.
(A). What came out was very large garland made out of currency notes.
(B). The unsuspecting governor opened the box in full view of the gathering
(C). When the RBI governor came to inaugurate the new printing press, the local unit of the BJP handed him a gift wrapped
box
(D). There was a twist – the notes were all as tattered as notes could get
(a) DACB (b) CABD (c) CBAD (d) DCAB
9.
(A). Otherwise the Congress would not have opposed PSU disinvestment today.
(B). It is clear that there is not consensus on economic reform.
(C). Nor would allies of ruling NDA opposes privatisation.
(D). All this would stop India from becoming the next superpower.
10.
(A). I suggested that Ford should buy up a company called NCP, which owned most of the car parks in the city centres
throughout the UK.
(B). We were discussing competing in the European market.
(C). If NCP became a Ford company, a notice could be placed at the entrance to all city centres car parks indicating that only
Ford cars could use them
(D). At one time I was giving a seminar for the British marketing department of Ford, the biggest Ford operation outside of
Detroit.
(a) BACD (b) DBAC (c) BDCA (d) BCAD
11.
(A). By the time he got to Linjeflug four years later, he had learned many lessons, in fact, he began his second stint as top
dog by calling the entire company together in a hanger and asking for help, a far cry from his barking out commands just
D8 months back.
(B). At SAS, he arrived at a time crisis.
201 | P a g
e
(C). This book is chock-a-block full of intrusive stories and practical advice, describing Carton’s activities at Vingresor (where
he assumed his first presidency at age C(B), Linjeflug, and SAS in particular.
(D). He began at Vingresor as an order giver, not a listener – neither to his people nor to his customers and made every
mistake in the book.
(a) BADC (b) BACD (c) CBAD (d) CDAB
12.
(A). They argue that it is this, which has led to the bankruptcy in many states.
(B). Here was a commission whose members worked very hard, did exemplary research and homework, before coming up
with a list of recommendations that balanced economic efficiency with safety nets for disadvantaged labour.
(C). It reminds us of the political shenanigans during the implementation of the Fifth pay Commission.
(D). How many times have you heard experts, politicians and the finance minister refer to the implementation of the pay
hikes following the commission's report as the singular cause for the increase in government expenditure?
E). Barring P. Chidambram, who was then the finance minister, every single political party and politician opposed the
implementation of the recommendations and are directly responsible for the current fiscal crises in the Centre and the
states.
(a) DBEAC (b) CDABE (c) BEADC (d) DEBAC
13.
(A). arrived here on Sunday by an early morning flight.
(B). Dubai-based Win Gautam who is the
(C). accused in the Rs E0 crore Before guns kickback case.
(D). He is scheduled to appear in the trial court By Wednesday.
(a) BACD (b) BCAD (c) ADCB (d) ACDB
14.
(A). the main difference is that efficiency is a ration and effectiveness is not.
(B). But they reach efficiency in a different way than American businesses.
(C). The Japanese are very efficient and such concepts as "just in time" are a witness to their efficiency.
(D). They reach efficiency through the route of effectiveness.
(a) CBDA (b) ADCB (c) CADB (d) CDAB
15.
(A). The general impressions that skilled negotiators seem to convey is they are people who keep their cards close to their
chest and do not reveal their feelings.
(B). Hence, they used a surrogate method- they countered the number of times that the negotiators talked about their
feelings or motives.
(C). This contrasts sharply with the amount of information given about external events such as facts, clarifications and
general expressions of opinion.
(D). The results showed that contrary to the general impressions, skilled negotiators are more likely to give information
about internal events than are average negotiators.
E). Feelings are in themselves not observable and Huthwaite's researchers could not measure them directly.
(a) AEBDC (b) AEDCB (c) ABEDC (d) ABECD
16.
(A). To get a head start, early the next morning the farmer started covering ground quickly because he wanted to get as
much land as he could.
(B). Late in the afternoon he realized the condition he had to fulfil to get the land was to get back to the starting point by
sundown.
(C). Even though he was tired, he kept going all afternoon because he did not want to miss this once in a lifetime
opportunity to gain more wealth.
(D). There is a story about a wealthy who was once offered all the land he could walk on in a day, provided he come back by
sundown to the point where he started.
202 | P a g
e
(a) DCAB (b) CABD (c) DACB (d) DABC
17.
(A). Sony has been valued at around Rs 800 crore.
(B). IBM is a leading consultancy firm.
(C). This valuation has been done by IBM.
(D). They have relied on the excess value approach.
(a) DABC (b) BADC (c) ABCD (d) ACBD
18.
(A). Even as Indians leftists think Bill Clinton is coming to take over India, Indian companies are preparing to take over
American ones on a gargantuan scale.
(B). Now Infosys and Wipro propose of Rs 54,000 crore each.
(C). To put this in perspective, recall that when Chandan sold his Parle brands to Coca-Cola amidst much swadeshi wringing
of hands, he got a repoted Rs 200 crore.
(D). Infosys and Wipro, our two most glamorous infotech companies, both want automatic permission from FIPB to take
over foreign companies worth - hold your breath - $ AE billion each.
(a) BCDA (b) ADCB (c) ACDB (d) BDAC
19.
(A). Mr D Gautam's personality sets him apart the rest.
(B). Nothing is too small for his attention
(C). He has a fanatical devotion to detail.
(D). This is what makes him a different guy.
(a) ACBD (b) ABCD (c) BCDA (d) BACD
20.
(A). In his first inaugural address he concluded with an eloquent plea; "Ask not what your country can do for you--- ask what
you can do for your country."
(B). John F. Kennedy, Democratic victor in the election of A960, was at DC, the youngest man ever to win the presidency.
(C). On television, in a series of debates with opponent Richard Nixon, he appeared able, articulate and energetic.
(D). In the campaign, he spoke of moving aggressively into the new decade, for 'the New Frontier is here whether we seek it
or not'.
(a) DABC (b) BCDA (c) CABD (d) BACD
21.
(A) Rana is one of a growing number of professionals who are channeling their passion into food.
(B) This is not a new phenomenon, of course — we have had IT professionals swapping codes for farms, bloggers hosting
secret suppers, and a MasterChef-inspired generation of Instagram chefs.
(C) However, food is not an easy industry to be in.
(D) We talk to a few new foodpreneurs who have switched career paths and are addressing the needs of this evolved, more
conscious market.
(E) As the food landscape evolves, newer examples are cropping up.
(a) ABCDE (b) ACBDE (c) ABECD (d) ADCEB (e) AEDBC
22.
(A) The era of sharing space for designated slots of time is here.
(B) Then came renting and leasing. But even that is passé.
(C) There were times when properties were bought and companies raised grand edifices to house their offices with
paraphernalia.
(D)With nearly two decades of the new millennium behind us today, the new technology is urging compaction of time and
space.
(E) The future lies in aggregation of time and space.
203 | P a g
e
(a) EBCDA (b) ECBDA (c) EADCB (d) ECDBA (e) EADBC
23.
(A) With the modernization of our homes, cooking is no longer done behind four walls
(B) the kitchen needs to be open to attract attention and is often the hub of the home.
(C) and have transformed into aesthetically pleasing spaces.
(D) In any home, the kitchen is considered as one of the most important spaces.
(E) Our kitchens thereby have moved from being only utilitarian
(a) DCBAE (b) DCEAB (c) DBAEC (d) DEBCA (e) DABEC
24.
(A) apart from the potential for upholding human values, is our ability to make choices, to take control of our destiny.
(B) Unless we go with nature, we won’t be able to survive any longer, in Stephen Hawking’s words, “the next thousand
years” on this fragile planet”.
(C) It all comes down to karma, what goes around comes around. Humans have committed mass deforestation, nature has
rewarded us with climate change, unexpected storms, wildfires and sea levels.
(D) That’s not just why we must be careful in terms of the lifestyle we choose. What differentiates us humans from other
species,
(E) Today it all comes down to our lifestyle choices, each of which, corporations have ensured have strong repercussions on
nature.
(a) CDAEB (b) CDEAB (c) CEADB (d) CADBE (e) CBDEC
25.
(A) The boys have queued up, the overseer has inspected their dhoti-kurtas,
(B) I am at the ashram gate, wondering if the priest will let me in.
(C) I walk in hesitantly, but the monk smiles warmly and points to an empty chair.
(D) and they are now waiting patiently for instructions from the priest , who is on the phone.
(E) Everything is ready for the evening prayer at the ashram.
(a) EDBCA (b) EADBC (c) ECBDA (d) ECDAB (e) EBDCA
26.
(A) for the entire length of the concert in anticipation of what these great
(B) The three-day The Hindu November Fest opened with ‘Classical and Beyond’, a coming together of the stars of
Hindustani music.
(C) the stunningly gifted Kaushiki Chakraborty, the extraordinary Satyajit Talwalkar, Rakesh Chaurasia, Purbayan Chatterjee,
and Sudhanshu Gharpure.
(D) The auditorium was jam packed with an attentive and benevolent audience sitting with undiminished enthusiasm
(E) masters would unfold. The star–studded stage had the inimitable maestro Taufiq Qureshi,
(a) BEDAC (b) BCDEA (c) BCAED (d) BDAEC (e) BADEC
27.
(A) Scriptures have always perceived the holistic value pervading creation and speak of the close ties between human
beings and nature,
(B) and of the need to live in harmony with the environment. Drawing from these and taking into account the body-mind
complex that constitutes each human being,
(C) the wise have shown that the most precious gifts for an individual are water, food and good advice, pointed out
Velukkudi Sri Krishnan in a discourse.
(D) Water and food are for the upkeep of the body and good advice, ‘subhashitani,’ for the welfare of the Atma. Good advice
helps one to remain healthy in both mind and body.
(E) Indian schools of thought have given much thought to explain the nature and function of the human mind. It is seen as an
instrument endowed with the functions to think, decide and act.
(a) ABDCE (b) ACDBE (c) AEDBC (d) AEBDC (e) ABCDE
204 | P a g
e
28.
(A) Legend has it that he was close to despair and hiding in a cave during a storm when he saw an animal in the process of
building something.
(B) It tried six times and failed and he realized he too had fought against the English six times and failed.
(C) This inspired him to continue fighting the English and he eventually won in ACAD.
(D) On the seventh attempt it succeeded.
(E) On March BE, AC06, Robert the Bruce became the King of Scotland. King Edward of England waged war against him and
made him an outlaw.
(a) EBCDA (b) EABDC (c) EBDCA (d) EACBD (e) ECDAB
29.
(A) If we do a movement, some hundreds of times, we can then repeat it easily with little awareness.
(B) Yoga is a way of bringing positive changes in our life. Positive change requires opposing past habits.
(C) But awareness is the key to sustaining transformation; awareness helps us recognize what we need to change and
prevents us from slipping back after we change.
(D) Movement becomes mechanical because our brain learns movement patterns.
(E) But Asanas can also become habits. If we practice asana like riding a bicycle, we lose a large part of its transformative
potential.
(a) BEDAC (b) BADEC (c) BCDEA (d) BDECA (e) BACDE
30.
(A) It made me want to dig into culture to get a grasp of men’s thinking process.
(B) Even when the laws were made more stringent in India, it did nothing to deter crimes against women.
(C) that allows men to think and do whatever they pleased with a woman — to her body, mind, thoughts or psyche.
(D) I think it shook the collective conscience of the nation. I started looking at the culture
(E) When Nirbhaya happened, it shook something in me.
(a) ECBDA (b) EADCB (c) EBCDA (d) EDCBA (e) EABCD
31.
(A) where many international students lived.
(B) most memorable of all, discover the hedonistic pleasures of red wine and inhale the indefinable aroma of freshly baked
baguettes.
(C) The aftershocks were still rippling out when I reached Paris to study comparative literature, hang around the Latin
Quarter and,
(D) With age on our sides, we had the energy and will to walk, after the last Métro had gone, to the Cité Universitaire
(E) It was a heady place, where impassioned and cheap wine-fuelled conversations lingered on well after the moon was high
in the sky.
(a) CADEB (b) CDBEA (c) CBEDA (d) CADBE (e) CAEBD
32.
(A) the game’s heart even as they enhanced many a fast bowler’s ability to extract reverse-swing.
(B) That it has been called ‘a gentleman’s game’ suggests that it is held to high standards. Yet, like a few other things wrong
with the game, ball-tampering remains one of its murkiest secrets.
(C) the use of nails or abrasive dust from the turf, and in some cases the use of bottle openers have plunged a knife into
(D) Cricket is a sport, but it is also a code of honour. The phrase ‘it’s not cricket’ refers to any act that is not fair.
(E) The seemingly innocuous application of saliva and sweat, and more interventionist acts such as pressing chewed
lozenges, throwing the ball hard on the surface,
(a) DBECA (b) DEBCA (c) DCAEB (d) DBCAE (e) DBACE
33.
(A) dire warnings about Mr. Yameen’s emergency in the past month have led to the Maldives cancelling its participation in
the Indian Navy’s “Milan” exercises. Even in Bangladesh,
(B) with Bangladesh’s Home Minister Asaduzzaman Khan describing the remarks as untrue, unfounded and not helpful.
205 | P a g
e
(C) the Indian Army chief, General Bipin Rawat’s tough talking last week about immigration has drawn ire there,
(D) the required changes in the government’s treatment of the opposition, and New Delhi’s
(E) Mr. Modi’s decision to abruptly cancel his visit to Male in 2015 did not yield
(a) EADCB (b) EBCDA (c) EDACB (d) EACDB (e) EDCBA
34.
(A) Genetic discrimination (GD) is understood to be differential treatment of those not showing symptoms
(B) Such conditions covered those with disability, who were poor, had mental health problems, were promiscuous, were
dwarfs, and so on.
(C) that made sterilization compulsory for those who expressed a range of conditions believed to be inherited.
(D) There were robust policies of eugenics in the U.S. in the A900s. These led to laws in many States
(E) but who are nevertheless treated differently on the basis of any real or assumed genetic characteristics. We must
recognize that GD is nothing new.
(a) ABCDE (b) ADCBE (c) AECDB (d) ACBDE (e) AEDCB
35.
(A) With supporting studies in the form of well-planned lab tests, this work has the potential to improve health care and
enhance drug discovery.
(B) Plants secrete various special chemicals to ward off predators, fight pathogens and survive in difficult situations. Some of
these so-called phytochemicals
(C) have been used to prepare traditional medicines and also poisons. While there are extensive databases of
phytochemicals of Chinese herbs, there has no similar work in India.
(D) The use of Indian medicinal plants for drug discovery and therapeutics just received a boost.
(E) For the repository, the scientists sourced information from several texts including those that documented tribal
medicine.
(a) DACBE (b) DEABC (c) DBCEA (d) DECBA (e) DCBEA
Directions (36-40): Rearrange the following five sentences (A), (B), (C), (D) and (E) in the proper sequence to form a
meaningful paragraph and then answer the questions given below.
(A) In the face of this apparent inequality, is it even possible for an outcome that favours the outnumbered minority?
(B) On the one side are some 8,000 Dongria Kondh and other tribals, sometimes referred to as “primitive” tribes.
(C) For in those thickly forested hills, where under the rich and diverse plant and animal life above the soil lie valuable
deposits of bauxite underneath, an environmental battle of an epic scale is being fought.
(D) On the other is the State of Odisha and one of the most powerful multi-national companies in the world, Vedanta.
(E) I have never been to Niyamgiri in Odisha. But what is playing out there since July 18 is a drama that has people like me
mesmerised even at this distance.
36 (1). Which of the following will be the FIFTH sentence after the rearrangement?
(a) A (b) B (c) D (d) C (e) E
36 (2). Which of the following will be the THIRD sentence after the rearrangement?
(a) A (b) B (c) D (d) C (e) E
(A). American President Donald Trump last week announced that his administration would soon impose tariffs on the import
of steel and aluminum into the U.S. for an indefinite period of time.
(B) The European Union, one of the largest trading partners of the U.S., has since vowed to return the favour through
retaliatory measures targeting American exporters.
(C) After all, they had learned their lessons from the global trade war of the 1930s which deepened and prolonged the Great
Depression, or so it was thought.
(D) The EU is expected to come out with a list of over 100 items imported from the U.S. that will be subject to scrutiny.
(E) World leaders did well to avoid protectionist trade policies in the aftermath of the Great Recession of 2008.
206 | P a g
e
37. Which of the following will be the SECOND sentence after the rearrangement?
(a) A (b) B (c) D (d) C (e) E
(A) We do need to guard against unfair trade practices, such as goods made in China being routed through some countries
with which India has an FTA, flouting all rules of origin and local value-addition norms.
(B) This is integral to the ongoing process of eliminating from Indian business assorted means of enrichment that have little
do with efficient creation of value.
(C). India can hope to end its present exclusion from global value chains — across various industry segments — through
membership of RCEP.
(D) At the same time, the government needs to appreciate that global trade and exposure to import competition constitute
a sure method of raising Indian industry’s competitiveness.
(E) In parallel, there’s the need to put in place clear-cut safeguards measures to prevent dumping of goods, especially from
China.
38. Which of the following will be the FOURTH sentence after the rearrangement?
(a) A (b) B (c) D (d) C (e) E
39.
(A). The Indian Meteorological Department just issued a warning saying that the storm has turned 'very severe'.
(B). Fani, a cyclonic storm brewing in the Bay of Bengal, is set to hit India's eastern coast in the state of Odisha.
(C). Fani can even worsen into an "extremely severe cyclone" by tomorrow.
(D). Ministry of Home Affairs has ordered to release financial assistance, in advance to four states that stand threatened by
the natural disaster.
(E). This will prompt the government to put the National Disaster Response Force and the Indian Coast Guard on high alert.
40.
The World Health Organization has declared India as polio-free since no new polio case has been reported in the country in
the last couple of years.
(A). It also gives an idea regarding the effective implementation of the government schemes in the country so that they give
the desired result
(B). Without participation from the general public, it would not have been possible to achieve this tremendous feat with the
government schemes only.
(C). India can take heart from this success and can replicate the same model for eradication of other diseases also from the
country.
(D). The thrust should be on educating the mass regarding the harmful effects of insects and the reasons for the growth of
such insects.
(E). This underlines the efforts by the Ministry of Health and Family Welfare along with the staff members at the ground
level.
Which among the following will be followed by the fifth sentence of the paragraph after the rearrangement?
(a) A (b) B (c) D (d) C (e) E
207 | P a g
e
IDIOMS & PHRASAL VERBS
1. By leaps and bounds
A. very fast B. very slow C. in details D. aimlessly
2. At daggers drawn
A. real cause B. to be puzzled C. at enmity D. at friendship
4. A man of straw
A. A puppet B. Influential C. One who has no influence D. To struggle in vain
6. Above board
A. boasting person
B. honest and straightforward
C. a man with arrogance
D. a dishonest person
8. Bandy words
A. to argue B. to request C. to give respect D. to be polite
14. Cut both ends B. very hard worker C. in favour of both sides D. against both sides
A. very big loss
15. Carrot and stick policy
A. reward and punishment B. dishonest C. arrogant D. selfish policy
16. When the thief saw the sentry at the main gate, he managed to get off from the attack.
208 | P a g
e
A. Dodge away B. Overpowered C. Escape D. Hide himself
26. We haven’t been able to decide on who is going to —- our baby while we are on the business trip next week.
A. fall out B. come to C. get by D. look after
27. In order not to lose its market share, our company must —- the latest technological developments.
A. hold up B. drop in C. fall apart D. Keep up with
28. It is obvious that the new law has —- some revolutionary changes in the current tax system but there are still some
points to be revised.
A. brought about B. broken away C. taken in D. pulled out
29. Before —- for Los Angeles, it is urgent that I get my car serviced today.
A. holding on B. letting down C. giving out D. setting off
30. A lot of plant and animal species are in danger of — at the moment.
A. dying out B. paying off C. turning on D. setting out
209 | P a g
e
SYNONYMS/ANTONYMS
Direction: In the following questions choose the word which best expresses the meaning of the given word.
1. Poignant
A. Showy B. Sad C. Silly D. Snobbish
2. Querulous
A. Critical B. Curious C. Quarrelsome D. Ambiguous
3. Audacious
A. Brilliant B. Powerful C. Bold D. Frightening
4. Perilous
A. Hazardous B. Rigorous C. Resilient D. Requisite
5. Reverie
A. Determination B. Day-dream C. Reality D. Realization
6. Genial
A. Cordial B. Selfish C. Careful D. Specific
7. Accrue
A. Accumulate B. Accommodate C. Grow D. Suffice
8. Loquacious
A. Talkative B. Slow C. Content D. Unclear
9. Vindictive
A. Imaginative B. Accusative C. Spiteful D. Aggressive
10. Inclement
A. Selfish B. Active C. Unfavorable D. Inactive
11. Anonymity
A. Reputation B. Publicity C. Being unknown D. Wrong address
12. Ephemeral
A. Disturbing B. Filthy C. Short-lived D. Poverty-stricken
13. Battered
A. Damaged B. Awarded C. Intact D. Wholesome
14. Extravagant
A. Hardworking B. Good C. Extraordinary D. Spendthrift
15. Redundant
A. Superfluous B. Uninterested C. Introspective D. Knowledgeable
210 | P a g
e
Direction: In the following questions choose the word which best expresses the opposite meaning of the given word.
16. Plausible
A. Inplausible B. Unplausible C. Implausible D. Displausible
17. Frailty
A. Weak B. Intensity C. Vehemence D. Strength
18. Flair
A. Talent B. Inability C. Spotlight D. Taste
19. Collapse
A. Rise B. Handicap C. Crush D. Crumble
20. Dormant
A. Acute B. Active C. Able D. Ablaze
21. Anxious
A. Crafty B. Light C. Carefree D. Careless
22. Hazy
A. Plain B. Light C. Clear D. Dull
23. Thrifty
A. Clean B. Loyal C. Wasteful D. Reverent
24. Fantastic
A. Old B. Ordinary C. Classic D. Rational
25. Innovate
A. Sell B. Buy C. Close D. Copy
26. Asceticism
A. Comfort B. Luxury C. Anti-semitism D. Humility
27. Enduring
A. Fleeting B. Painful C. Permanent D. Long lasting
28. Dissolution
A. Retribution B. Establishment C. Persuasion D. Compliance
29. Progressive
A. Repressive B. Retrogressive C. Repulsive D. Aggressive
30. Unnerved
A. Confident B. Nervous C. Hopeful D. Anxious
211 | P a g
e
PSEUDO CODE
A. 0 B. 3 C. 1 D. 2
A. 3 B. 5 C. 8 D. None of these
3. Consider the following recursive function fun(x, y). What is the value of fun (4, 3)
def fun(x,y):
if x==0:
return y
else:
return fun(x-1,x+y)
print(fun(4, 3))
A. 13 B. 12 C. 9 D. 10
212 | P a g
e
p=p+2
if(p > r)
Continue
End if
p=1
if(p > s)
Jump out of the loop
End if
End for
End for
Print p + q
A. 13 B. 24 C. 35 D. 20
A. 0#1#2#3#4#5#6#7#8#9#10
B. 0#1#2#3#4#5#6###
C. #0#1#2#3#4#5##7#8#9#10
D. #0#1#2#3#4#5#
A) 5 3 9 B) 6 14 17 C) 6 4 14 D) 6 4 16
213 | P a g
e
if (p[0] && P[4])
m = p[1]
End if
if (p[2] && p[3])
m = p[2]
End if
Print m
A. 18 B. 1 C. -5 D. 4
A. 15 B. 4 C. 5 D. 8
214 | P a g
e
A. test1 B. test2 C. test1testcompleted D. test2test completed
a. 56 b. 78 c. 72 d. 68
a. Error b. 3 c. 4 d. 5
215 | P a g
e
Break
Default: Print “UGICampus”
a. UGI b. Campus c. UGICampus d. Compile Time Error
a. 1 3 6 10 15 b. 1 2 3 4 5 c. 4 6 8 10 d. 11 2 3 5
216 | P a g
e
19. What will be the output of the following pseudo?
Integer a, b,c,d
Set d=6
For(each a from 1 to 2)
For(each b from a to 2)
For(each from b to 2)
print d
END For
End For
End For
a. 6 6 6 6 6 6 b. 6 6 c. 6 6 6 d. 6 6 6 6
fun(1000)
217 | P a g
e
p=1
if(p > s)
Jump out of the loop
End if
End for
End for
Print p + q
A. 13 B. 24 C. 35 D. 20
A. 21 B. 17 C. 37 D. 20
24. What will be the output of the following pseudo code for a = 4, b = 6?
Integer funn(Integer a, Integer b)
if(a > 1)
return a * funn (b - 6, a - 4)
Else
return 1
End if
return a + b
End function funn()
A. 0 B. 17 C. 4 D. 7
218 | P a g e
A. 17 B. 14 C. 22 D. 12
a. 763
b. 963
c. 678
d. 769
a. %0
b. 0x100
c. %x
d. error
28. What will be the number of " * " printed by the given pseudocode when input is 25?
Write "Please enter a number"
Read input
Repeat while input > 0
if( input > 0 and input <=10 )
Write *
else if ( input >10 and input <=20 )
Write **
else if ( input >20 and input< = 30 )
Write ***
input --
end if
end while
a. 55
b. 45
c. 35
d. 25
29. You have written the pseudo-code given alongside for performing a binary search in an array of elements sorted
in ascending order. Which step is to be followed in ‘A’ to execute binary search successfully?
1. Compare x with the middle element.
2. If x matches with the middle element, we return the mid index.
3. A
4. Else ( x is smaller ) recur for the left half
219 | P a g e
a. Else if x is greater than the mid element, then x can only lie in left half subarray after the mid element. So we
recur for left
b. Else if x is greater than the mid element, then x can only lie in right half subarray after the mid element. So we
recur for right
c. Else if x is less than the mid element, then x can only lie in right half subarray after the mid element. So we recur
for right
d. None of the given options
a. SevenTwoHello
b. OneHello
c. SevenTwo
d. Seven
a. 15
b. 1
c. 2
d. 8
220 | P a g e
33. What will be the output of the following pseudocode:
function main()
set float go = 5.0, integer num = 1*10;
do
{
num /= go;
} while(go--);
display num;
return 0;
}
a. Floating point exception
b. Compile time error
c. 3 6 7
d. No output
a. 12345
b. 54321
c. 13524
d. 53124
a. Pseudocode
b. Test
c. Competition
d. PseudocodeTest
e. PseudocodeCompetition
f. Error
Function value:integer
Return(a++)
End-function
Function main()
221 | P a g e
Int a=value(10)
Display –a
End-function
a. 8
b. 9
c. Error
d. 11
e. 12
f. 10
a. 4
b. 5
c. 3
d. Error
e. 8
f. 7
a. Infinte
b. 4
c. 3
d. 2
e. 1
f. 0
39.
Set integer array=[2,4,0,1,6,0]
for m=1 to 6 step 1 do
arr[m]++
end-for
a. [2,5,1,2,7,0]
b. [2,5,1,2,7,1]
222 | P a g e
c. [3,5,1,2,7,1]
d. Error
e. [2,5,1,2,6,0]
f. [3,6,1,2,7,0]
a. error
b. f
c. P
d. q
e. fq
f. pfq
a. 1
b. Garbage
c. Error
d. 4
e. 6
f. 0
a. 3
b. 0
c. 4
d. 1
e. 2
f. Infinite
43. using Bubble Sort, how many swaps are required to sort the given array [2,5,1,3,4]?
223 | P a g e
For all elements of array
If array[i]> array[i+1]
Swap(array[i],array[i+1])
End if
End for
a. 4
b. 5
c. 6
d. 7
e. 8
a. b,a
b. c,b
c. a,b
d. a,a
e. b,b
a. Error
b. {1,3,5,7,8}
c. {1,3,7,8}
d. 1,3,5,7,10}
a. 11
b. 10
224 | P a g e
c. 9
d. Error
e. Garbage
a. pq
b. nullpq
c. pnullq
d. nullpnullq
a. 4
b. 6
c. 3
d. 5
function fun(int x)
if(x>0)
fun(--x)
show (x)
fun(--x)
endfunction
function main()
fun(4)
endfunction
a. 4 3 2 1 1 2 3 4
b. 4 2 0 0 2 4
225 | P a g e
c. 0 1 2 0 3 0 1
d. 0 1 2 3 2 1 0
a. 11111
b. 12345
c. 55555
d. 54321
e. 554321
a. 6
b. 7
c. 5
d. 8
a. 45
b. 48
c. 50
d. 52
e. 47
Set integer x = 0;
Function int f()
if (x == 0)
return x + 1;
else
return x - 1;
endif
endfunction
function int g()
return x++;
endfunction
function int main()
int i = (f() + g()) | g(); //bitwise or
int j = g() | (f() + g()); //bitwise or
show (i,j);
endfunction
a. 3,5
b. 1,7
c. 5,9
d. 10,20
a. 2
b. 3
c. 0
d. 1
227 | P a g e
ANSWER KEY 6TH SEMESTER
1. DATA INTERPRETATION
1.D 6.D 11.B 16.E 21.D 26.A 31.A 36.D
2.B 7.A 12.E 17.A 22.E 27.B 32.C 37.C
3.D 8.B 13.B 18.E 23.A 28.E 33.B 38.A
4.A 9.D 14.A 19.A 24.D 29.C 34.D 39.C
5.A 10.C 15.C 20.B 25.B 30.D 35.B
2. DATA SUFFICIENCY
1.E 6.E 11.E 16.A 21.D 26.C
2.C 7.D 12.E 17.D 22.E 27.E
3.A 8.D 13.C 18.C 23.B 28.D
4.D 9.C 14.D 19.E 24.E 29.E
5.E 10.C 15.E 20.D 25.C 30.C
3. CRYPT ARITHMETIC
1.C 6.B 11.B 16.A
2.C 7.B 12.A 17.B
3.A 8.B 13.E 18.C
4.B 9.B 14.E 19.A
5.B 10.A 15.B 20.E
5. PROBABILITY
1.B 6.C 11.B 16.C 21.E 26. A 31. A
2.A 7.D 12.C 17.E 22.A 27. B 32. D
3.A 8.C 13.E 18.B 23. A 28. B 33. A
4.A 9.B 14.D 19.D 24.A 29. C 34. A
5.A 10.B 15.B 20.C 25.B 30. A 35. C
6. STATISTICS
1. B 2. A 3. B 4. C 5. D 6. A 7. A 8. D 9. B 10. B
11. A 12. B 13. A 14. C 15. A 16. B 17. A 18. C
228 | P a g e
8. SYLLOGISM
1. D 6. D 11. C 16. B 21. C 26. B 31. B 36. A
2. B 7. D 12. B 17. D 22. C 27. A 32. E 37. B
3. A 8. B 13. D 18. D 23. E 28. B 33. A 38. A
4. E 9. A 14. B 19. B 24. B 29. C 34. B 39. B
5. E 10. C 15. C 20. D 25. E 30. A 35. A 40. E
9. SEATING ARRANGEMENT
1. A 6.D 11.C 16.C 21.B 26.A 31.B 36.D 41.C 46.A
2. A 7. E 12.E 17.B 22.C 27.E 32.C 37.D 42.A 47.E
3. E 8.D 13.E 18.C 23.C 28.C 33.D 38.C 43.A 48.A
4.D 9. B 14.B 19.B 24.D 29.B 34.E 39.B 44.B 49.C
5.E 10.B 15.E 20.D 25.B 30.A 35.D 40.E 45.D 50.A
10. PUZZLE
1. C 6. C 11. A 16. C 21. D 26. E 31. D 36. A 41. D
2. C 7. D 12. E 17. B 22. B 27. B 32. A 37. B 42. E
3. D 8. B 13. C 18. E 23. D 28. D 33. C 38. E 43. E
4. D 9. D 14. A 19. B 24. C 29. B 34. D 39. C 44. C
5. A 10. E 15. E 20. D 25. E 30. C 35. B 40. D 45. B
11. SELECTION
1. D 6. C 11. E 16. D 21. C 26. B 31. A
2. E 7. A 12. E 17. D 22. C 27. A 32. B
3. E 8. B 13. A 18. E 23. E 28. E
4. A 9. A 14. A 19. D 24. B 29. C
5. C 10. B 15. E 20. B 25. A 30. B
13. REVISION
1. C 6.A 11.C 16.D 21.B 26.A 31.C 36.C 41.C 46.B
2. C 7. B 12.C 17.A 22.C 27.A 32.B 37.B 42.C 47.B
3. A 8.C 13.D 18.A 23.B 28.D 33.C 38.C 43.D 48.B
4.A 9. C 14.B 19.C 24.C 29.B 34.C 39.D 44.C 49.D
5.B 10.C 15.B 20.C 25.D 30.C 35.C 40.C 45.B 50.C
14. PREPOSITIONS
1. B 6.C 11.C 16.C 21.B 26.A 31.B 36.C
2. C 7.B 12.A 17.A 22.B 27.C 32.C 37.B
3. B 8.A 13.C 18.C 23.C 28.B 33.A 38.B
4. C 9.C 14.B 19.C 24.B 29.A 34.A 39.C
5.A 10.A 15.C 20.B 25.C 30.B 35.B 40.B
229 | P a g e
15. CONJUNCTIONS
1. C 6. C 11. A 16. B 21. B 26. C 31.A 36. A
2. B 7. C 12. A 17. C 22. B 27.A 32. B 37. B
3. B 8. B 13. B 18. C 23. C 28.C 33. D 38. B
4. B 9. C 14.C 19. A 24. C 29.A 34. C 39. C
5. B 10. B 15. B 20. C 25. A 30.D 35. A 40. A
EXPLANATION
1. (C) ‘Because’ doesn’t come with ‘the reason why’ because then the sentence become superfluous
2. (B) had hardly any
3. (B)after DESPERATELY use THAT
4. (B) ‘between’ is followed by ‘and’ and not ‘to’.
5. (B)No sooner ................... than
6. (C) and
7. (C) NOT ONLY..BUT ALSO
8. (B)No sooner ................... than
9. (C) NEITHER HAS MY SISTER
10. (B)Scarcely ................... when
11. (A) The reason.. and.. why will not be used together
12. (A) unless is negative word so don’t use NOT With it
13. (B) able to EITHER comprehend or solve the problem
14. (C) NEITHER..NOR
15. (B)REMOVE for USE and
16. (B) remove then, we don’t use THEN with WHEN, USE COMMA AFTER REMOVING then
17. (C) use THAT after all
18. (C) SO..AS>> NEGATIVE SENTENCE / AS. .. AS>> POSITIVE SENTENCE
19. (A) AS FAST AS
20. (C) change "THUS HER MOTHER TAKES" TO "SO HER MOTHER TOOK"
21. (B) after no sooner use helping verb..so no sooner did the teacher
22. (B) UNLESS YOU WORK HARD
23. (C) LEST..SHOULD
24. (C) BETWEEN..ANDFROM. TO
25. (A) UNLESS negative word so remove not
26. (C) question mark
27. (A) NO SOONER+HELPING VERB..( no sooner had)
28. (C) remove THAT use AS
29. (A) STOPPED NOT ONLY
30. (D) No error
31. (A) with AS SOON AS we dont use THEN
32. (B) HARDLY,,,,WHEN
33. (D)NO ERROR
34. (C) SO...AS. ..NEGATIVE SENTENCE
35. (A)THE PLACE nonliving things so use WHICH.BUT here the word,,THE SAME,, requires THAT only,,so use THAT
36. (A) with WH FAMILY we must not use THAT
230 | P a g e
37. (B) LEST...SHOULD
38. (B) When AS is used for denoting some reason,, we dont use SO
39. (C)EITHER ... TAKEN OR,,NOT ELSE,,,SO USE OR
40. (A) we use OR with WHETHER but not with IF,, so ,, USE whether
1. A 2. C 3. A 4. C 5. C 6. B 7. C 8. A 9. C 10. D
11. C 12. A 13. C 14. C 15. A 16. C 17. B 18. A 19. A 20. D
21. A 22. A 23. A 24. C 25. C 26. D 27. D 28. A 29. D 30. A
20. SYNONYMS/ANTONYMS
1. B 2. C 3. C 4. A 5. B 6. A 7. A 8. A 9. C 10. C
11. C 12. C 13. A 14. D 15. A 16. C 17. D 18. B 19. A 20. B
21. C 22. C 23. C 24. B 25. D 26. B 27. A 28. B 29. B 30. A
231 | P a g e
21. PSEUDO CODE
1. A 2. A 3. A 4. B 5. D 6. D 7. D 8. B 9. B 10. D
11. C 12. C 13. B 14. D 15. C 16. A 17. D 18. A 19. D 20. B
21. B 22. D 23. D 24. C 25. B 26. A 27. C 28. B 29. B 30. A
31. D 32. A 33. D 34. D 35. F 36. D 37. F 38. D 39. B 40. D
41. F 42. F 43. A 44. E 45. A 46. D 47. D 48. B 49. C 50. E
51. C 52. B 53. B 54. A
232 | P a g e